514
Financial Accounting Paper F3 (International) Integrated Course Notes ACF3CN09(J) (INT)

f3 Course Notes

  • Upload
    aip10

  • View
    10.259

  • Download
    9

Embed Size (px)

DESCRIPTION

Financial AccountingPaper F3 (International) Integrated Course Notes ACF3CN09(J) (INT)Improving study material and removing errorsThere is a constant need to update and enhance our study materials in line with both regulatory changes and new insights into the exams. BPP appoints, from one of our experienced tutor team, a subject expert to update and improve these course notes regularly. These updates are technically checked by another experienced tutor and frequently proof read. We always a

Citation preview

Page 1: f3 Course Notes

Financial Accounting Paper F3 (International) Integrated Course Notes ACF3CN09(J) (INT)

Page 2: f3 Course Notes

Improving study material and removing errors There is a constant need to update and enhance our study materials in line with both regulatory changes and new insights into the exams. BPP appoints, from one of our experienced tutor team, a subject expert to update and improve these course notes regularly. These updates are technically checked by another experienced tutor and frequently proof read. We always aim to leave no numerical errors and narrative typos. However, given the volume of detailed information being changed in a short space of time, it is regrettable that despite our control procedures an error may slip through. We apologise sincerely for any inconvenience that this might cause.

If you find a specific error or typo please let us know at [email protected] so we can correct it immediately. In addition we would welcome any suggestions you may have to further improve these study materials.

Blank

Page 3: f3 Course Notes

3

F3 Financial Accounting (INT) Contents page Page

Introduction to the paper and the course................................................................................................................. 5 Skills bank ............................................................................................................................................................. 13 1 Introduction to accounting ............................................................................................................................ 31 2 Home study chapter – The regulatory framework ........................................................................................ 43 3 Accounting conventions................................................................................................................................ 53 4 Sources, records and books of prime entry.................................................................................................. 73 5 Ledger accounts and double entry ............................................................................................................... 91 6 From trial balance to financial statements .................................................................................................. 107 7 Sales tax..................................................................................................................................................... 135 8 Inventory..................................................................................................................................................... 149 9 Tangible non-current assets ....................................................................................................................... 169 10 Intangible non-current assets ..................................................................................................................... 201 11 Accruals and prepayments ......................................................................................................................... 213 12 Irrecoverable debts and allowances ........................................................................................................... 237 13 Provisions and contingencies..................................................................................................................... 261 14 Control accounts ........................................................................................................................................ 273 15 Bank reconciliations.................................................................................................................................... 301 16 Correction of errors..................................................................................................................................... 315 17 Home study chapter – Preparation of financial statements for sole traders ............................................... 331 18 Incomplete records..................................................................................................................................... 343 19 Partnerships ............................................................................................................................................... 359 20 Introduction to company accounting........................................................................................................... 385 21 Preparation of financial statements for companies..................................................................................... 405 22 Events after the reporting period ................................................................................................................ 427 23 Statements of cash flows............................................................................................................................ 437 24 Home study chapter – Information technology ........................................................................................... 459 25 Answers to Lecture Examples ................................................................................................................... 471

Page 4: f3 Course Notes

INTRODUCTION

4

F3 Financial Accounting (INT) Study Programme Page Introduction to the paper and the course................................................................................................................. 5 Skills bank ............................................................................................................................................................. 13 1 Introduction to accounting ............................................................................................................................ 31 2 Home study chapter – The regulatory framework ........................................................................................ 43 3 Accounting conventions................................................................................................................................ 53 4 Sources, records and books of prime entry.................................................................................................. 73 5 Ledger accounts and double entry ............................................................................................................... 91

End of Day 1 – refer to Course Companion for Home Study

6 From trial balance to financial statements .................................................................................................. 107 8 Inventory..................................................................................................................................................... 149 9 Tangible non-current assets ....................................................................................................................... 169 10 Intangible non-current assets ..................................................................................................................... 201

End of Day 2 – refer to Course Companion for Home Study

11 Accruals and prepayments ......................................................................................................................... 213 12 Irrecoverable debts and allowances ........................................................................................................... 237 13 Provisions and contingencies..................................................................................................................... 261 15 Bank reconciliations.................................................................................................................................... 301

End of Day 3 – refer to Course Companion for Home Study

7 Sales tax..................................................................................................................................................... 135 14 Control accounts ........................................................................................................................................ 273 16 Correction of errors..................................................................................................................................... 315 17 Home study chapter – Preparation of financial statements for sole traders ............................................... 331 18 Incomplete records..................................................................................................................................... 343

End of Day 4 – refer to Course Companion for Home Study

19 Partnerships ............................................................................................................................................... 359 20 Introduction to company accounting........................................................................................................... 385 22 Events after the reporting period ................................................................................................................ 427

End of Day 5 – refer to Course Companion for Home Study

21 Preparation of financial statements for companies..................................................................................... 405 23 Statements of cash flows............................................................................................................................ 437 24 Home study chapter – Information technology ........................................................................................... 459

End of Day 6 – refer to Course Companion for Home Study

25 Answers to Lecture Examples ................................................................................................................... 471

Prepare for and book your CBE! You should plan to sit your CBE within the next couple of weeks whilst the knowledge from this course is still fresh in your mind. In preparation use the Learning Media Practice & Revision Kit and i-Pass to test yourself on as many questions as you can, revising from the Course Notes and Passcards any areas of the syllabus that cause you problems.

One of the criteria for Pass Assurance is that you book your CBE with BPP, so please contact your local BPP centre as early as you can to book your CBE and good luck!

Page 5: f3 Course Notes

INTRODUCTION

5

Introduction to Paper F3 Financial Accounting (INT) Overall aim of the syllabus To develop knowledge and understanding of the underlying principles and concepts relating to financial accounting and technical proficiency in the use of double-entry accounting techniques including the preparation of basic financial statements.

The syllabus The broad syllabus headings are:

A The context and purpose of financial reporting B The qualitative characteristics of financial information and the fundamental bases of accounting C The use of double entry and accounting systems D Recording transactions and events E Preparing a trial balance F Preparing basic financial statements

Main capabilities On successful completion of this paper, candidates should be able to:

• Explain the context and purpose of financial reporting • Define the qualitative characteristics of financial information and the fundamental bases of accounting • Demonstrate the use of double entry and accounting systems • Record transactions and events • Prepare a trial balance (including identifying and correcting errors) • Prepare basic financial statements for incorporated and unincorporated entities

Links with other papers

This diagram shows where direct (solid line arrows) and indirect (dashed line arrows) links exist between this paper and other papers that may precede or follow it.

Paper F7 Financial Reporting, assumes knowledge acquired in paper F3 Financial Accounting, and develops and applies this further and in greater depth. Paper P2 Corporate Reporting, assumes knowledge acquired at the Fundamentals level including core technical capabilities to prepare and analyse financial reports for single and combined entities.

Corporate Reporting (P2)

Financial Reporting (F7)

Financial Accounting (F3)

Accountant in Business (F1)

Page 6: f3 Course Notes

INTRODUCTION

6

Assessment methods and format of the exam Examiner: Nicola Ventress

Students have the option to take either a paper-based or a computer-based exam.

Both examinations are a two hour paper and all questions are compulsory. Questions will assess all parts of the syllabus and will contain both computational and non-computational elements. Format of the Exam Marks

80 40 two mark questions 10 one mark questions 10

90

Page 7: f3 Course Notes

INTRODUCTION

7

Study Programme Aims

Achieving ACCA's Study Guide Outcomes

A The context and purpose of financial reporting

A1 The reasons for and objectives of financial reporting Chapter 1 A2 Users’ and stakeholders’ needs Chapter 1 A3 The main elements of financial reports Chapter 1 A4 The regulatory framework Chapter 2

B The qualitative characteristics of financial information and the fundamental bases of accounting

B1 The qualitative characteristics of financial reporting Chapter 3 B2 Alternative bases used in the preparation of financial information Chapter 3

C The use of double entry and accounting systems

C1 Double entry bookkeeping principles including the maintenance of accounting records and sources of information

Chapters 4 & 5

C2 Ledger accounts, books of prime entry and journals Chapters 4 & 5 C3 Accounting systems and the impact of information technology on financial reporting Chapter 24

D Recording transactions and events

D1 Sales and purchases Chapters 4, 5, 7 & 14

D2 Cash Chapters 4 & 5 D3 Inventory Chapter 8 D4 Tangible non-current assets Chapter 9 D5 Depreciation Chapter 9 D6 Intangible non-current assets and amortisation Chapter 10 D7 Accruals and prepayments Chapter 11 D8 Receivables and payables Chapter 12 D9 Provisions and contingencies Chapter 13 D10 Capital structure and finance costs Chapters 20 &

21

Page 8: f3 Course Notes

INTRODUCTION

8

E Preparing a trial balance

E1 Trial balance Chapter 6 E2 Correction of errors Chapter 16 E3 Control accounts and reconciliations Chapter 14 E4 Bank reconciliations Chapter 15 E5 Suspense accounts Chapter 16

F Preparing basic financial statements

F1 Statements of financial position Chapter 17 F2 Income statements Chapter 17 F3 Events after the reporting period Chapter 22 F4 Accounting for partnerships Chapter 19 F5 Statements of cash flows (excluding partnerships) Chapter 23 F6 Incomplete records Chapter 18

Page 9: f3 Course Notes

INTRODUCTION

9

Analysis of the pilot paper

Please note that the ACCA will not publish past exam papers for the Knowledge modules. The analysis of the Pilot Paper should therefore be used as a guide to both the areas that will be examined and the mix between narrative and computational questions.

Narra

tive

Com

puta

tiona

l

The context and purpose of financial reporting

The reasons for and objectives of financial reporting

Users’ and stakeholders’ needs

The main elements of financial reports 1

The regulatory framework 1

The qualitative characteristics of financial information and the fundamental bases of accounting

The qualitative characteristics of financial reporting 1

Alternative bases used in the preparation of financial information 4

The use of double entry and accounting systems

Double entry bookkeeping principles including the maintenance of accounting records and sources of information

Ledger accounts, books of prime entry and journals 1

Accounting systems and the impact of information technology on financial reporting 1

Recording transactions and events

Sales and purchases 1

Cash 1

Inventory 1 1

Tangible non-current assets and depreciation 1 4

Intangible non-current assets and amortisation 1

Accruals and prepayments 2

Receivables and payables 2

Provisions and contingencies 1

Capital structure and finance costs 1 2

Page 10: f3 Course Notes

INTRODUCTION

10

Narra

tive

Com

puta

tiona

l

Preparing a trial balance

Trial balance 1

Correction of errors 1 2

Control accounts and reconciliations 4

Bank reconciliations 1 1

Suspense accounts 1

Preparing basic financial statements

Statements of financial position 1 1

Income statements

Events after the reporting period 2

Accounting for partnerships 1 2

Statements of cash flows (excluding partnerships) 3

Incomplete records 2

Page 11: f3 Course Notes

INTRODUCTION

11

Key to icons

Question practice from the Study Text This is a question we recommend you attempt for home study.

Section reference in the Study Text Further reading is needed on this area to consolidate your knowledge.

Page 12: f3 Course Notes

INTRODUCTION

12

Page 13: f3 Course Notes

13

1

Skills bank

This section explains and demonstrates the key skills required to enable you to maximise your chance of

exam success. Knowledge of the syllabus is insufficient on its own. Through question practice you will develop a

set of skills that will enable you to pass this paper.

Page 14: f3 Course Notes

14

Page 15: f3 Course Notes

SKILLS BANK

15

1Key skills required to pass Our analysis of the examiner’s comments on past exams, together with our experience of preparing students for this type of exam, suggests that to pass Paper F3 you will need to develop a number of key skills.

Each of these key skills is analysed on the following pages. Examples from past exam questions are included to illustrate the importance of these skills and how these skills should be applied.

1 Learning and understanding the syllabus content

2 Time management and approaching questions logically

3 Answering questions efficiently

Page 16: f3 Course Notes

SKILLS BANK

16

Skill 1 – Learning and understanding the syllabus content

1 What do I need to know to attempt the paper? F3 is a broad syllabus that can be tested by objective testing including multiple choice questions and other means such as data entry type questions. This means that you need a broad and yet quite detailed knowledge of the syllabus as well as an understanding of a variety of calculations to apply the theory.

The type of knowledge that you have to acquire includes the following:

Practical application – e.g. calculation of numbers to be included in the financial statements; identifying errors and correcting them; producing figures to go into a statement of cash flows.

Theoretical knowledge – e.g. how accounts fit together and their key components; the purpose and roles of the regulatory bodies; the underlying concepts and assumptions that govern accounts preparation.

In this section we will look at approaches that you can take to help you learn the key elements of the knowledge in the syllabus.

2 Practical application Practical application requires you to do two main things:

1. Understand the rationale behind a topic and be able to explain it; and 2. Apply your understanding to generate figures that may be included in a set of accounts.

In this way you should be in a good position to answer most questions. They will either ask you to calculate a number from some information provided, or to use the numerical information provided to demonstrate your knowledge of the topic in some way. You should ensure that you read the requirement carefully for these questions; further tips on question approach will be covered under skill 2 [page 20].

For example, in the pilot paper Q32 asks:

Application

Alpha received a statement of account from a supplier, Beta, showing a balance to be paid of $8,950. Alpha’s payables ledger account for Beta shows a balance due to Beta of $4,140.

Investigation reveals the following:

(1) Cash paid to Beta $4,080 has not been allowed for by Beta

(2) Alpha’s ledger account has not been adjusted for $40 of cash discount disallowed by Beta.

What discrepancy remains between Alpha’s and Beta’s records after allowing for these items?

A $690

B $770

C $9,850

D $9,930 (2 marks)

This question requires you to have a good understanding of how to account for credit purchase transactions and the way control accounts work in order to be able to answer it.

Page 17: f3 Course Notes

SKILLS BANK

17

Another question type is Q46 from the pilot paper. It requires you to apply your understanding of a specific area of the syllabus.

Principles

Gareth, a sales tax registered trader purchased a computer for use in his business. The invoice for the computer showed the following costs relating to the purchase:

$ Computer 890 Additional memory 95 Delivery 10 Installation 20 Maintenance (1 year) 25 1,040 Sales tax (17.5%) 182 Total 1,222

How much should Gareth capitalise as a non-current asset in relation to the purchase?

A $1,222

B $1,040

C $890

D $1,015 (2 marks)

Note that to find the correct figure you need to understand both the principle behind what can be included in the cost of a non-current asset and what effect the sales tax may have on the cost of the asset to be included in the accounts.

Therefore you need to ensure that you understand the context of the principles of what you are learning and are able to apply them to numerical examples. To do this you should consider:

Detail – for practical application of the topics covered in the syllabus, you must know the rules for each area and understand where they have been derived from. By reviewing the overview at the beginning and the summary at the end of the relevant chapter in your course notes before attempting questions, you should be able to pick up the key points.

Application – this is where question practice is key. The more practice you have in working through the questions, the more confident you will become on using and applying the theory.

3 Theory You will also have to answer narrative questions about theory.

You can expect questions about:

1. Fact – what are the rules and requirements of the accounting standards? What are the similarities and differences between sole traders, partnerships and companies?

2. Application – how are the accounting concepts applied to different areas of the syllabus?

Here is an example from the ACCA F3 examiner on an area that students struggled with in the December 2007 paper based exam:

Page 18: f3 Course Notes

SKILLS BANK

18

Fact

Which of the following provides advice to the International Accounting Standards Board (IASB) as well as informing the IASB of the implications of proposed standards for users and preparers of financial statements?

A The Standards Advisory Council

B The International Financial Reporting Interpretations Committee

(1 mark)

This question is testing you on facts regarding the roles of the different regulatory bodies. To be able to answer this question you need to know what each body does and how they fit together. Here is a further example from the Pilot Paper

Application

Q3

In times of rising prices, what effect does the use of the historical cost concept have on a company’s asset values and profit?

A Asset values and profit both understated

B Asset values and profit both overstated

C Asset values understated and profit overstated

D Asset values overstated and profit understated

(2 marks)

This type of question is about applying the theory to a theoretical situation. This can seem tricky if there are no numbers involved – the key here is to think of some simple numbers for an asset and its depreciation policy (e.g. 10% straight line) both at cost (e.g. $100) and revalued amount (e.g. $300) and see what effect they have on asset values and profits:

Cost ($) Revalued amount ($)

Asset (cost – 1 year’s depreciation) (100 – 10) 90 270 (300- 30) Profit before depreciation 50 50 Depreciation (10) (30) Profit after depreciation 40 20

It is therefore important to make sure that you actively learn the content of the syllabus in such a way that you can answer all of the types of question that you may face in the exam.

Page 19: f3 Course Notes

SKILLS BANK

19

There are various ways to build up this level of knowledge. Here are some suggestions:

Read Passcards regularly

Get a colleague or friend to set you quizzes

Practice as many MCQs as possible

Write your own glossary of definitions

Produce a list of your common mistakes and review it before doing question practice!

Skills practice Learn the content of the syllabus actively by:

1. Reviewing the key overviews for each chapter

2. Practising as many questions as possible, moving from using your notes to completing them without any help

3. Using the study text to help only on areas you’re struggling with and to fill in gaps in your background knowledge

Page 20: f3 Course Notes

SKILLS BANK

20

Skill 2 – Time management and approaching questions logically

The F3 exam contains objective tests; it is important that you use your time wisely to gain maximum marks. Your approach will differ depending on whether you are doing a computer based exam (CBE) or a paper based exam.

In the paper based exam you will have mainly multiple choice questions with some yes/no questions. The computer based exam (CBE) may contain a wider range of question styles including selecting a number of possible correct answers from a list and entering numbers into a blank box.

You need to ensure that you have a logical approach to these questions to maximise your chances of getting the correct answers.

Below we look at how to approach each style of paper, and within that how to deal with the different types of questions you may come across.

Time management

1 Paper based exam and computer based exam

What you SHOULD NOT do

Panic! You have two hours to answer 50 questions. This equates to approximately 2.6 minutes per question or 1.3 minutes for the small 1 mark questions. For many questions you will get the answer straight away and so you are likely to have a bit more time to think about some of the others. Therefore don’t worry about your timing on each individual question, just keep track over a few (e.g. 5).

What you SHOULD do

It is important to start the exam positively. Firstly: Work through questions systematically

Start at question 1 and begin answering from there working through questions in order.

If you find a question that you don’t know the answer to and want to come back to it later then put an answer in for the moment, make a note of it and go onto the next question. If you take the paper based exam, this means you won’t fill out your answer sheet incorrectly by having left a gap on the sheet.

Try not to jump around questions otherwise you may leave some unanswered by the end.

Then: Check your paper before the end of the exam

Having answered all of the questions you should look through your answers to make sure:

1. You are happy with the options selected; and

2. You have answered all questions

If you have taken this logical and systematic approach you should have given yourself the best chance of doing well in the exam.

Page 21: f3 Course Notes

SKILLS BANK

21

2 Computer based exams (CBE) If you are doing the computer based exam you need be systematic in your approach as with the paper based exam. There are however some other guidelines to ensure that you make the most of the CBE format of the exam.

STEP

Before the exam Make sure that you are registered with the ACCA and that your exam centre has your exam booking. This is important because you can not sit the exam if you are not a registered student and you need your student card as identification on the day of your exam.

STEP

At the beginning of the exam You want to make the start of the exam as stress free as possible so make sure that you have the following available:

• Photo identification and your student number – your ACCA student card is ideal for this

• Paper supplied by your exam centre, pens and a calculator

• Details of which exam you are planning to sit

STEP

Starting the exam To start the exam you will need to do the following:

• Login – enter your ACCA student number and your date of birth – make sure that you have both to hand

• Select the exam – the system will ask you which exam you want to do (ensure you pick the International exam and not the UK stream) and then you <Confirm> that selection

• Instructions – the next three screens are instructions – read these carefully so that you know what you have to do to complete the exam

• Launch the exam – please don’t click this screen until you have been advised to by your invigilator AND you are ready because this starts the exam and starts the timer.

1

2

3

Page 22: f3 Course Notes

SKILLS BANK

22

STEP

Answering the questions The exam will start at question 1. You can progress through the questions by clicking <Next> but you can also go back by clicking <Previous>. When you answer the questions you must follow the following procedure :

• Enter your answer

• Click on <Submit> - you must do this otherwise as soon as you click <Next> to move to the next question your entry will be lost and you will have to re-enter

• If you click <Next> and have not submitted your answer to a question then you will get a reminder – clicking <OK> on the reminder does not submit your answer – you must go back and re-enter your answer and then <Submit>

• If you are unsure of an answer then just put an answer in for now, make a note of it, and revisit it later

You can see that it is very important to submit your answers as you go and to keep track of what you have done. It is probably a good idea therefore to use your pad of paper to do two things:

1. Make any notes you want to help you answer the questions; and

2. Keep a record of the status of each question

For example:

Question Status

1

2 ?

3

4 X

Where

means that the question has been answered and I am reasonably confident of the answer.

? means that the question has been answered but I want to check the answer

X means that the question has not been answered and I need to go back to it

It would be most efficient to set this up at the beginning of the exam. In this way you will have a tally of which questions really need to be checked over at the end of the exam and it reduces the chances that you will leave a question unanswered.

4

Page 23: f3 Course Notes

SKILLS BANK

23

STEP

At the end of the exam time At the end of the exam you should check your answers and ensure that you have submitted an answer for every question as well as double checking any answers you were not sure of.

You have two ways of navigating the questions:

1. Clicking <Previous> to work back through the questions one by one; or

2. Using the drop down menu which shows all the questions 1-50 indicating whether an answer has been submitted or not; clicking on the question number will take you directly to that question

Remember that if you choose to change a previously submitted answer you must <Submit> the new one otherwise your original answer will be retained instead!

STEP

Closing your exam session Once you are satisfied that you are happy with your answers, if you have time left in the exam then you have a couple of options to finish your exam session (subject to any advice you get from the invigilators in your exam centre).

1. Let the time on the on screen clock run down to zero and the exam session will end automatically; or

2. Click <Exit> - you will be asked to <Confirm> this so you can not accidentally end your session early

In either case the next thing you will see will be the Results Screen that shows your mark and whether you have passed or not.

It is important that you don’t <Exit> at this stage since you have no proof of your result!

Your invigilator will ask you to <Print> two copies and will instruct you to <Exit> once these have been printed off.

You have now finished your exam.

3 Some final tips on timings • Make use of the paper to make notes or to work out the answers to questions;

• If you find a particularly difficult question or a long calculation, move on and come back to it later in the remaining time – it is important that you do not run out of time leaving easier questions later in the paper unanswered

• Keep an eye on the clock so that you can pace yourself

• Be well prepared for the exam day so that you can concentrate on doing the exam rather than the administration around it

• Get to your exam venue in plenty of time so that you are relaxed when you get into the exam room.

5

6

Page 24: f3 Course Notes

SKILLS BANK

24

Approaching Questions Logically – Multiple Choice Questions

1 What to do if you know the answer to the question If you know the answer to a narrative question you should:

1. Locate the correct answer

2. Check the other answers

3. Read the question again to ensure you’re answering the correct question

4. Confirm that you have the correct answer

This systematic check will ensure that you do not throw away marks when you really do know the answer.

2 What to do if more than one answer appears plausible Sometimes more than one option can seem to answer the question. In this case you have to firstly ensure you’ve read the requirement carefully, as questions may be phrased in ways that are not what you’re expecting. If you still identify more than one likely option, select the “most correct” answer. The approach adopted above is useful here too but this time you have to think through the alternatives a bit more.

For example, Q45 on the Pilot Paper states:

Which of the following statements are correct?

(1) A cash flow statement prepared using the direct method produces a different figure for operating cash flow from that produced if the indirect method is used.

(2) Rights issues of shares do not feature in cash flow statements.

(3) A surplus on revaluation of a non-current asset will not appear as an item in the cash flow statement.

(4) A profit on sale of the sale of a non-current asset will appear as an item under Cash flows from Investing Activities in a cash flow statement.

A 1 and 4

B 2 and 3

C 3 only

D 2 and 4

This is testing your knowledge of cash flow statements. At first sight, it may be tricky to identify the correct answer as there are so many variations in the options.

Here are some steps to follow:

STEP

Never Firstly, identify any answers that are immediately wrong. In this question, the key thing to think about is what cash effect there may be to any transactions. Remembering that revaluations are purely an accounting adjustment means that statement (3) must be correct. So we can discard options A and D from the answers.

1

Page 25: f3 Course Notes

SKILLS BANK

25

STEP

Assess the remaining answers We now need to consider statement (2) as this will differentiate answers B and C. Remembering that it is only a bonus issue of shares that has no cash effect (rights issues give shareholders the right to buy shares at a certain price) means that statement (2) is incorrect (we do get some cash in), so in this case we can now discard answer B and have arrived at answer C.

STEP

Read the question again… Finally, we should re-read the requirement before submitting the question to ensure we are answering the correct question. The question could easily have asked: “Which of the following statements is incorrect” which would have led us to a different answer!

This systematic approach helps you to break a question down and work through to find the correct answer logically.

3 How to approach numerical questions In the exam, you will be asked to calculate numbers based on some information provided. If it is a multiple choice question rather than a data entry question, the temptation may be to look at the options first, and then “fit” your calculations to the one you think is most likely. This could lead you to answering the question incorrectly, especially if you have not read the requirement carefully.

For example, the ACCA F3 examiner identified the following question which students struggled with in the June 2008 paper based exam:

Johnsons use the imprest method for accounting for petty cash. The petty cash was counted and there was $57.22 in hand. The following petty cash slips were found for the following:

$

Stamps 16.35

Sale of goods to staff 12.00

Coffee and tea purchase 18.23

Birthday cards for stuff 20.20

What is Johnsons’ imprest amount?

A $124

B $100

C $112

D $80

Working through this question, we find that, since the $12.00 was from sale of goods to the staff (i.e. a receipt in the petty cash tin), the imprest amount is $100. It is important to understand what the question is asking and how to approach answering these types of questions.

2

3

Page 26: f3 Course Notes

SKILLS BANK

26

STEP

Read the requirement carefully With numerical questions it is really important to first understand what the question is asking, since you can easily either do too much work and waste time working out calculations that aren’t required, or answer the question you want to answer, and not the question that is actually being asked!

In the question above, many students picked option A as they assumed all slips were for amounts paid and none for receipts.

STEP

Ignore the options Once you’re certain what the question is asking, you should ignore the options given, as you may arrive at an answer that matches an option but is not what is required by the question.

Again, this is where many students tripped up above, as they saw option A was available and immediately picked that rather than remembering what the requirement was for that question.

STEP

Match your answer Once you’ve calculated your answer, match it to the options. If you’ve worked carefully and answered the question, the matching option will be available. If no option matches your calculation, re-read the requirement to ensure you’ve understood what you have to do. If you still can’t find the answer, you may want to guess the answer, make a note of the question number and return to it at the end when you can judge how much of the time remaining you can spend on it.

4 What to do if you still don’t know the answer… If you have been through the above steps and can’t identify a preferred answer then you have to guess!

What you SHOULD NOT do

Two main things to avoid:

1. Waste excessive time – time spent dithering over a single question could leave you with insufficient time for the rest of the paper.

2. Not answering – this is a common yet serious error – even if you make a wild guess you start with a 25% chance of success. Your chance of getting the 2 marks if you don’t offer an answer is zero!

What you SHOULD do

Having used the three step approaches above to narrow down your possible answers, go with the one that feels right. And move on.

If you have a flash of inspiration later in the exam go back and revisit it – but only if you are sure.

Data entry questions The above systematic approach to multiple choice questions can be applied to any questions that you come across in the exam. You may be asked to calculate a numerical figure which you then have to enter into a box in the exam. For these questions you can use steps 1 and 2 above in How to Approach Numerical Questions.

1

2

3

Page 27: f3 Course Notes

SKILLS BANK

27

Skills practice 1. Practice keeping track of the questions you have answered when doing questions from the

Practice and Revision Kit

2. Always check your answers through (if you would have time in the exam) before looking at the solutions in the back of the book

3. Practice as many multiple choice questions as possible.

4. If you don’t know the answer to a question – don’t just go to the answer at the back or just guess – use the three step approach described above.

Page 28: f3 Course Notes

SKILLS BANK

28

Skill 3 – Answering questions efficiently

Once you have grasped an area fully, the third key skill in being successful at F3 is to answer questions efficiently. If you are confident at how to arrive at an answer, you can often avoid doing extra calculations that do not impact on the final answer by knowing when to stop.

1 Knowing the basics Before you attempt any shortcuts, it is important to make sure you’re happy with the topic itself. You should work through questions as you have been taught in lecture examples, and make sure you can arrive at the right answer each time.

2 Identifying when to “Stop” Whilst you will have enough time to answer every question fully in the exam, it makes sense to develop your question technique as you revise to try to recognise at what point you’ve arrived at the answer and can stop. This means you avoid wasting time and can use those extra seconds to think about other questions.

As an example, the ACCA F3 examiner identified the following question which students struggled with in the June 2008 paper based exam:

Charles entered into the following transactions:

1. He sold goods on credit to Cody with a list price of $3,200. He

allows a 10% trade discount and a further 2% discount for payment within seven days. Cody paid within two days.

2. He made a credit sale to Mary allowing a 5% trade discount on the

list price of $640.

3. He purchased goods for $600 and paid $590, receiving a discount for

immediate cash payment.

How much discount should be recorded in the Discount Allowed account as a result of the above transactions?

A $57.60

B $10.00

C $352.00

D $409.60

Page 29: f3 Course Notes

SKILLS BANK

29

It is tempting, if you know the topic, to calculate absolutely everything here. But if you think about the requirement, all you need to work out is what the discount allowed balance is.

Remember, this is only the discount we allow our customers for prompt payment. It is not any discount we receive (so transaction 3 is irrelevant to this question!). It is also not any trade discount we allow our customers who are bulk buyers or regular customers (they will always take this discount as there are no conditions attached; we therefore record the sale initially net of this discount). Therefore transaction 2, which only deals with a trade discount, is also irrelevant.

Now, we have eliminated a lot of the work, and are left with transaction 1:

The initial price was $3,200, on which there was a trade discount of 10%. So: List price $3,200 Trade discount (10%) $(320) Amount recorded as a sale initially $2,880

We know that there’s a further settlement discount of 2% if payment is received within seven days, which it is. Therefore we can calculate the discount we allowed this customer on early payment:

Amount recorded as a sale initially $2,880.00

Further settlement discount (2%) $(57.60)

No more work is needed on this question – you now have the answer!

Spending time now working out the other trade discount or the discount received is unnecessary.

3 Building up your technique

There are several places on the syllabus where you may be able to apply this technique. As you practise questions, compare your workings to the answers and highlight at what point on your workings you arrived at the answer they wanted.

In this way you can start to pick up where you’re doing too much work to answer the question. Your goal should be to finish your calculations as soon as you’ve got the correct answer!

You may like to make a list of key common areas where you can answer questions efficiently, and look over them as part of your revision.

Skills practice 1. Practice questions building up to the exam with efficiency in mind. The more you do, the

more efficient you’ll become.

2. Identify from your question practice which areas this is likely to apply to and make a list of key areas and the relevant shortcuts.

Page 30: f3 Course Notes

SKILLS BANK

30

Page 31: f3 Course Notes

31

1

Syllabus Guide Detailed Outcomes Having studied this chapter you will be able to:

• Define and understand the principles of financial reporting.

• Identify and define the different business entities of: sole trader, partnership and limited liability company and recognise the legal differences between them.

• Identify the advantages and disadvantages of operating as each of the three types of business entity.

• Identify the users of financial statements and state and differentiate between their information needs.

• Understand and identify the purpose of each of the main financial statements.

• Define and identify assets, liabilities, equity, revenue and expenses.

Exam Context This chapter introduces the subject of accounting. Questions on this area will most likely focus on the different characteristics of the three types of business entity: sole trader, partnership and limited liability company.

Qualification Context Sole trader and partnership accounts are only examined in Financial Accounting. The Fundamentals and Professional level papers of Financial Reporting (F7) and Corporate Reporting (P2) are set in the context of a limited liability company. These papers will test your understanding of the content of financial statements and the detailed accounting rules which companies must apply.

Introduction to accounting

Page 32: f3 Course Notes

1: INTRODUCTION TO ACCOUNTING

32

Overview

Income statement Statement of financial position

Users of financial information

Introduction to accounting

Financial statements

Limited liability company

Sole trader Partnership

Concept of separate entity

Types of business entities

Page 33: f3 Course Notes

1: INTRODUCTION TO ACCOUNTING

33

1 Accounting

Definition 1.1 Accounting is a way of recording, analysing and summarising transactions of a business.

2 Proforma financial statements

Income statement 2.1 Income statement for the year ended 31 December 20X7:

$ $ Sales 200,000 Less: Cost of sales Opening inventories 40,000 Purchases 110,000 Carriage inwards 20,000 170,000 Closing inventories (50,000) (120,000) Gross profit 80,000 Sundry income 5,000 Discounts receivable 3,000 88,000 Less: Expenses Rent 11,000 Carriage outwards 4,000 Telephone 1,000 Electricity 2,000 Wages and salaries 9,000 Depreciation 7,000 Bad and doubtful debts 3,000 Motor expenses 5,000 Discounts allowable 1,000 (43,000) Profit for the year 45,000

Page 34: f3 Course Notes

1: INTRODUCTION TO ACCOUNTING

34

Statement of financial position 2.2 Statement of financial position as at 31 December 20X7:

$ $ ASSETS Non-current assets

Land and buildings 100,000 Office equipment 50,000 Motor vehicles 30,000 Furniture and fixtures 20,000

200,000 Current assets

Inventories 50,000 Trade receivables 30,000 Less: allowance for receivables (2,000) 28,000 Prepayments 5,000 Cash in hand and at bank 7,000

90,000 Total assets 290,000 CAPITAL AND LIABILITIES Capital

Capital 170,000 Profit 45,000 Less: drawings (25,000)

190,000 Non-current liabilities

Bank loans 40,000 Current liabilities

Bank overdraft 16,000 Trade payables 40,000 Accruals 4,000

60,000 Total capital and liabilities 290,000

Page 35: f3 Course Notes

1: INTRODUCTION TO ACCOUNTING

35

3 Users of financial information

Lecture example 1 Idea generation Required What information would these users of financial information be interested in?

Solution (a) Investors (b) Employees (c) Lenders (d) Suppliers (e) Customers (f) Governments and their agencies (g) Public

Page 36: f3 Course Notes

1: INTRODUCTION TO ACCOUNTING

36

4 Accounting records 4.1 In order to be able to produce an income statement and a statement of financial position a

business needs to keep a record of all its transactions.

4.2 This process is called bookkeeping.

4.3 Accounting records should be complete, accurate and valid if the information produced is to be useful for the users of financial information.

4.4 The mechanics of bookkeeping and the accounting records a business should keep will be covered in Chapters 4, 5 and 6.

5 Types of business entities 5.1 Businesses fall into three main types:

(a) Sole trader

(b) Partnership

(c) Limited liability company

The sole trader is the simplest of these forms.

6 The concept of business entity (separate entity) 6.1 A business is considered to be a separate entity from its owner and so the personal

transactions of the owner should never be mixed with the business transactions.

6.2 When considering a limited liability company this distinction is laid down in law – the company has a separate legal identity.

6.3 In preparing accounts, any type of business is treated as being a separate entity from its owner(s).

Quick Quiz Q2

Sections 2.3, 2.4

Page 37: f3 Course Notes

1: INTRODUCTION TO ACCOUNTING

37

7 Chapter summary Section Topic Summary 1 Accounting Accounting is a way of recording, analysing and

summarising a business’ transactions. 2 Proforma financial

statements Companies must follow a prescribed format when producing their financial statements, there is however no set format for a sole trader’s income statement and statement of financial position.

3 Users of financial information

Financial statements are used by a wide variety of users, each with different information needs. Satisfying the investors’ needs will mean that the majority of other users’ needs are also met.

4 Accounting records All businesses must keep sufficient accounting records in order to be able to produce accurate information about the entity’s activities.

5 Types of business entities

There are three main types of businesses. For sole traders and partnerships the owners have unlimited liability and bear all the risks and reap all the rewards of being in business. For a limited liability company the shareholders' liability is limited to the extent of their investment.

6 The concept of business entity

The business entity concept states that a business is a separate entity from its owners.

Page 38: f3 Course Notes

1: INTRODUCTION TO ACCOUNTING

38

Chapter Summary

Income statement Statement of financial position

Users of financial information

Introduction to accounting

Financial statements

Limited liability company

Sole trader Partnership

Concept of separate entity

Types of business entities

• Shows the income and expenses for a business over a period of time, usually a year

• Shows the assets and liabilities of the business at a point in time

• Investors • Employees • Lenders • Suppliers • Customers • Governments and

their agencies • Public

'An individual sets up business on their own' • All the risks and rewards are

borne by the sole trader

'More than one individual enter into business together' • Risks and rewards are shared

between the partners

'A separate legal entity from the owners' • The company bears the risks and

rewards • The owners have limited liability

• A business is a separate entity from its owner • Personal transactions must be recorded separately (drawings)

Page 39: f3 Course Notes

39

Chapter 1: Questions

Page 40: f3 Course Notes

1: QUESTIONS

40

1.1 In a sole trader and a partnership the owners are personally liable if the business cannot meet its debts.

Is this statement true or false?

A True

B False (1 mark)

1.2 If a limited liability company goes into liquidation will the shareholders have to make a financial contribution to help the company pay its creditors?

A Yes

B No (1 mark)

1.3 Which of the following statements most accurately defines the business entity concept?

A The business must be treated as being separate from its owners.

B A business must be set up as a separate legal entity. (1 mark)

Page 41: f3 Course Notes

41

Chapter 1: Answers

Page 42: f3 Course Notes

1: ANSWERS

42

1.1 A

1.2 B

1.3 A

END OF CHAPTER

Page 43: f3 Course Notes

43

2

Syllabus Guide Detailed Outcomes Having studied this chapter you will be able to:

• Understand the role of the regulatory system including the roles of the

– International Accounting Standards Committee Foundation (IASCF)

– International Accounting Standards Board (IASB)

– Standards Advisory Council (SAC)

– International Financial Reporting Interpretations Committee (IFRIC)

• Understand the role of International Financial Reporting Standards (IFRS)

Exam Context Questions on this chapter will be knowledge based and so it is important that you are familiar with the role of each body. The role of IFRIC was tested in the Pilot Paper.

Qualification Context Financial Accounting introduces the International Accounting Standards Board's role in issuing IFRSs and paper F3 examines some key standards. All of these standards are built upon in the Fundamentals level paper Financial Reporting (F7) and the Professional level paper Corporate Reporting (P2).

Home study chapter - The regulatory framework

Page 44: f3 Course Notes

2: HOME STUDY CHAPTER - THE REGULATORY FRAMEWORK

44

Overview

IASB SAC IFRIC

IASCF

Regulatory framework

Issue IFRS

Page 45: f3 Course Notes

2: HOME STUDY CHAPTER - THE REGULATORY FRAMEWORK

45

1 Introduction 1.1 Financial statements are produced by an entity's managers in order to show its owners how

the entity has performed over a period of time.

1.2 Company financial statements particularly need to show a true and fair view. This means a system of regulation is necessary to ensure that financial statements are produced to a high standard and are comparable across different companies.

2 Regulatory system 2.1

Standards Advisory Council(SAC)

International Financial ReportingInterpretations Committee

(IFRIC)

International Accounting Standards Board

(IASB)(14 Board members)

International AccountingStandards Committee Foundation

(IASCF)(22 Trustees)

Key:AppointsReports toAdvises

International Accounting Standards Committee Foundation (IASCF) 2.2 The IASCF is a not-for-profit organisation based in the United States which heads up the

regulatory system. Its Trustees appoint members to the IASB, IFRIC and SAC. They also oversee the regulatory system and raise the finance necessary to support it. It has no involvement in the standard setting process.

International Accounting Standards Board (IASB) 2.3 The IASB's principal aim is to develop a single set of high quality accounting standards:

International Financial Reporting Standards (IFRS). It also liaises with national accounting standard setters (for example the UK's ASB) to achieve convergence in accounting standards around the world.

Section 1.4.1

Page 46: f3 Course Notes

2: HOME STUDY CHAPTER - THE REGULATORY FRAMEWORK

46

International Financial Reporting Interpretations Committee (IFRIC) 2.4 The IFRIC issues guidance on both how to apply existing IFRSs in company financial

statements and how to account for new financial reporting issues where no IFRS exists. It reports to the IASB.

Standards Advisory Council (SAC) 2.5 The SAC's principal role is to advise the IASB on a range of issues which include:

• The IASB's agenda and timetable for developing IFRSs • Advising the IASB of areas that may need to be considered by IFRIC.

3 The role of International Financial Reporting Standards (IFRS)

3.1 IFRSs provide guidance as to how items should be shown in a set of financial statements both in terms of their monetary amount and any other disclosure. For example: IAS 2: Inventory states at what amount a company should value its inventory and also requires that the financial statements breakdown the inventory figure between its components such as raw materials, work in progress and finished goods.

3.2 If a company follows the relevant accounting standards its financial statements should show a true and fair view.

Lecture example 1 Exam standard question for 1 mark What is the role of the International Accounting Standards Committee Foundation? A To appoint members of the IASB B To advise the IASB on new accounting standards they should consider issuing.

Solution

Page 47: f3 Course Notes

2: HOME STUDY CHAPTER - THE REGULATORY FRAMEWORK

47

Lecture example 2 Exam standard question for 1 mark Which of the following bodies is involved is trying to achieve convergence of global accounting standards? A IASB B IFRIC

Solution

4 Chapter summary Section Topic Summary 1 Introduction Financial statements are relied on by many different

user groups to make economic decisions. A system of regulation is therefore necessary to ensure that the information produced is of a high standard.

2 Regulatory system The IASCF appoints members to the IASB, IFRIC and SAC.

The IASB issues International Financial Reporting Standards.

The IFRIC issues guidance on how to apply accounting standards. The SAC advises the IASB on its agenda.

3 The role of international financial reporting standards

International financial reporting standards give guidance as to how transactions should be recorded in the accounts.

.

Page 48: f3 Course Notes

2: HOME STUDY CHAPTER - THE REGULATORY FRAMEWORK

48

Chapter Summary

IASB SAC IFRIC

IASCF

Regulatory framework

Issue IFRS

• 22 Trustees who: - appoint members to the IASB, IFRIC and SAC - oversee the regulatory system - raise finance to support the system • Not involved in standard setting process

• Aim: to advise the IASB on - their agenda and timetable for

developing IFRS - areas that may need to be considered

by IFRIC

• Aim to develop a single set of high quality accounting standards (IFRS)

• Liaises with national accounting standard setters (for example the UK's ASB)

• Issues guidance on how to apply existing IFRS

Page 49: f3 Course Notes

49

Chapter 2: Questions

Page 50: f3 Course Notes

2: QUESTIONS

50

2.1 Accounting standards are prepared by

A the IASB

B the IASC Foundation

C the IAASB (1 mark)

2.2 Which of the following best describes the role of The International Financial Reporting Interpretations Committee?

A Issues International Financial Reporting Standards.

B Provides advice on the development of standards.

C Interprets International Financial Reporting Standards. (1 mark)

Page 51: f3 Course Notes

51

Chapter 2: Answers

Page 52: f3 Course Notes

2: ANSWERS

52

2.1 A

2.2 C

END OF CHAPTER

Page 53: f3 Course Notes

53

3

Syllabus Guide Detailed Outcomes Having studied this chapter you will be able to:

• Define, understand and apply accounting concepts and qualitative characteristics.

• Understand the balance between qualitative characteristics.

• Identify and explain the main characteristics of alternative valuation bases (for example net realisable value).

• Understand the advantages and disadvantages of historical cost accounting.

• Understand the provision of International Financial Reporting Standards governing financial statements regarding changes in accounting policies.

• Identify the appropriate accounting treatment if a company changes a material accounting policy.

Exam Context Questions on this chapter are likely to test your understanding of the qualitative characteristics of information. For example, the Pilot Paper required you to identify the factors that make information reliable. Questions may also ask you to define accounting conventions.

Qualification Context Your understanding of the remaining chapters of IASB Framework will be developed in the Fundamentals level paper Financial Reporting (F7). You should also expect to see more detailed calculations on IAS 8 tested in Paper F7.

Accounting conventions

Page 54: f3 Course Notes

3: ACCOUNTING CONVENTIONS

54

Overview

The objective of financial statements

Accounting conventions

Underlying assumptions

Qualitative characteristics of financial information

Elements of financial statements

IASB Framework

Other issues

Concepts and conventions Alternative valuation bases

IAS 8: Accounting policies, changes in accounting estimates and errors

Page 55: f3 Course Notes

3: ACCOUNTING CONVENTIONS

55

1 Introduction 1.1 As noted in Chapter 2 financial statements should show a true and fair view of, or present

fairly, the entity's activities. They are produced to provide information to the entity's owners.

1.2 In order for this information to be useful it must possess certain characteristics.

2 The IASB's Framework for the Preparation and Presentation of Financial Statements

Conceptual framework 2.1 The IASB's Framework is not an accounting standard.

2.2 It is a set of principles which underpin the foundations of financial accounting.

2.3 Whenever a new accounting standard is issued it will be based on the principles of the IASB Framework. Furthermore its principles should be applied to account for any item where no accounting standard exists.

2.4 The Framework is divided into seven sections.

Only sections 1 – 4 are examinable at Paper F3.

The objective of financial statements 2.5 To provide information about the financial position, financial performance and changes

in financial position of an entity that is useful to a wide range of users in making economic decisions.

1) The objectiveof financialstatements

2) Underlyingassumptions

3) Qualitative characteristics of financialinformation

6) Measurement of the elements of financial statements

5) Recognition infinancial statements

4) The elementsof financialstatements

Framework7) Concepts of capital and capitalmaintenance

Page 56: f3 Course Notes

3: ACCOUNTING CONVENTIONS

56

Underlying assumptions 2.6 Accruals basis

The effects of transactions and other events are recognised when they occur (and not as cash or its equivalent is received or paid) and they are recorded in the accounting records and reported in the financial statements of the period to which they relate. Going concern The financial statements are normally prepared on the assumption that an entity is a going concern and will continue in operation for the foreseeable future. If this is not appropriate, then additional disclosure about the basis of preparation must be made in the financial statements.

Qualitative characteristics of financial information

2.7

Quick Quiz Q3

Quick Quiz Q4,

Q10

• For same entity over different periods: consistency

• Between different entities: disclosure of accounting policies

• Information should be readily understandable by users who are assumed to have reasonable knowledge

Understandability Comparability

Relevance Reliability

• Assist users in evaluating past and predicting future events

• Materiality

• Faithful representation • Substance over form • Neutrality • Prudence • Completeness

Page 57: f3 Course Notes

3: ACCOUNTING CONVENTIONS

57

2.8 The elements of financial statements The five elements of financial statements and their definitions are listed below.

Asset A resource controlled by an entity as a result of past events and from which future economic benefits are expected to flow to the entity.

Liability A present obligation of the entity arising from past events, the settlement of which is expected to result in an outflow of economic benefits.

Equity The residual interest in the assets of an entity after deducting all its liabilities, so EQUITY = NET ASSETS = SHARE CAPITAL + RESERVES

Income Increases in economic benefits during the accounting period in the form of inflows or enhancements of assets or decreases of liabilities that result in increases in equity, other than those relating to contributions from equity participants.

Expenses Decreases in economic benefits during the accounting period in the form of outflows or depletions of assets or increases of liabilities that result in decreases in equity, other than those relating to distributions to equity participants.

3 Alternative valuation bases

Historic cost 3.1 Financial statements are generally produced using the historical cost convention where

items are recorded at their historic cost. For example, if an entity purchased a building in 20X6 for $1 million then the building would be recorded as an asset at $1 million. The $1 million asset would then be depreciated to reflect the wearing out of the building. However, in reality, the value of the building may appreciate over time depending on market values.

Page 58: f3 Course Notes

3: ACCOUNTING CONVENTIONS

58

Lecture example 1 Idea generation What are the advantages and disadvantages of recording the building at its historic cost of $1 million (consider the Framework's qualitative characteristics)?

Solution Advantages of historic cost (1) (2) (3) Disadvantages of historic cost (1) (2)

3.2 Note that in times of rising prices using the historical cost convention will lead to asset values being too low and profits too high in a set of financial statements.

3.3 Due to the limitations of historic cost, alternative valuation bases exist. They are: • replacement cost • net realisable value • economic value

Replacement cost 3.4 Assets are carried at the amount it would cost to acquire an equivalent asset today.

Liabilities are shown at the amount that would be required to settle the obligation today. Replacement cost is also known as 'current cost'.

Net realisable value 3.5 This values items at their expected selling price less any costs that need to be incurred

before the item can be sold. Inventory should always be shown in the financial statements at the lower of cost (historic cost) and net realisable value.

Page 59: f3 Course Notes

3: ACCOUNTING CONVENTIONS

59

Lecture example 2 Preparation question A Ltd has 100 items in inventory at the year end. The following information is available: $ Total cost of items to date 1,000 Expected selling price per item 11 Costs which still need to be incurred per item before item can be sold 2 Required (a) What is the historic cost of the inventory? $ (b) What is the net realisable value of the inventory? $ (c) What value for inventory should be shown in the financial statements? $

Workings

Economic value 3.6 This is the value of an item derived from its ability to generate net cash flows. It can also be

known as 'present value'. For example, the economic value of a machine would be calculated by determining the value in today's prices, of the future cash inflows from selling items produced by the machine less the related cash outflows.

Quick Quiz Q6

Page 60: f3 Course Notes

3: ACCOUNTING CONVENTIONS

60

Page 61: f3 Course Notes

3: ACCOUNTING CONVENTIONS

61

Additional Notes

Page 62: f3 Course Notes

3: ACCOUNTING CONVENTIONS

62

4 Other examinable concepts and conventions 4.1 In addition to the concepts and conventions set out in the Framework, the following are also

relevant in the preparation of financial statements.

The business entity concept 4.2 This recognises the distinction between the business and its activities and the owners or

managers themselves. This is particularly important when considering sole trader or partnership accounts as these businesses are not separately identified by law.

The money measurement concept 4.3 Only items which are capable of being measured in monetary terms should be recognised

in the financial statements. For example, even though a loyal workforce may be of benefit to a business this value cannot be measured in monetary terms and is therefore not included on the balance sheet.

The duality concept 4.4 Every transaction has two effects. This is the underlying principle of the double entry

accounting system.

The historical cost convention 4.5 Assets and liabilities are recorded at their historical cost. This is the amount recorded when

the transaction took place.

5 IAS 8: Accounting policies, changes in accounting estimates and errors

Accounting policies – definition 5.1 Accounting policies are the significant principles, bases, conventions, rules and practices

applied by an entity in preparing and presenting the financial statements. It is the way the entity has decided to treat an item in its financial statements, for example whether non-current assets are carried at historic cost or a revalued amount.

Changes in accounting policy 5.2 Changes in accounting policy will only arise if:

(a) There is a new accounting standard or statutory requirement. (b) Using the new policy makes the financial statements more relevant and reliable.

Page 63: f3 Course Notes

3: ACCOUNTING CONVENTIONS

63

Accounting treatment 5.3 Financial statements contain two years worth of figures. For example a company whose

year end is 31 December 20X7 will show information for 20X7 and 20X6. The current year figures (20X7) will be produced using the new accounting policy. In order for the financial statements to be comparable over time the comparative figures (20X6) will be restated. This means they will be reproduced and drawn up using the 20X7 accounting policies.

5.4 Disclosure The following disclosure should be made: (a) The nature of the change in accounting policy (b) The reasons for the change (c) The amount of the adjustment in the current period and the comparative period.

Illustration: Airport Parking purchased a plot of land in 20X1 for $250,000. Airport Parking uses this plot of land as a car park and 'rents out' spaces to passengers using the local airport. The company deems the land to have an unlimited useful life and so does not depreciate it. Airport Parking has always held the land at historic cost but now, in 20X7, wants to change its accounting policy and revalue the land. Its current valuation is $800,000. The current value at the end of 20X6 was $700,000. The 20X7 financial statements must be produced using the new accounting policy (revaluation). The 20X6 financial statements must be restated to show the land using the revaluation policy so that the financial statements are comparable.

Page 64: f3 Course Notes

3: ACCOUNTING CONVENTIONS

64

The original 20X6 financial statements showed: 20X6 $'000 Land 250 Other assets 600 850 Share capital 100 Retained earnings 250 Liabilities 500 850 The 20X7 financial statements will show: 20X7 20X6 Restated $'000 $'000 Land 800 700 Other assets 600 600 1,400 1,300 Share capital 100 100 Revaluation reserve (450 + (800 – 700)) 550 450 Retained earnings 250 250 Liabilities 500 500 1,400 1,300 The 20X7 financial statements will disclose: (1) The nature of the change in accounting policy (2) The reasons for the change (3) The amount of the adjustment in the current period and the comparative period. A suitable disclosure might say: 'During 20X7 Airport Parking changed its accounting policy for land. Land was previously held at historic cost but the directors have now decided to revalue the land and show it at its current value. This is in order to show more relevant information in the financial statements. The 20X6 financial statements have been restated and are produced using the policy of revaluation. The land has been increased to its value in 20X6 of $700,000 and a revaluation reserve of $450,000 has been established. This increases restated net assets by $450,000. In 20X7 the land was revalued by a further $100,000 in order to include the 20X7 valuation in the financial statements'.

(700 – 250)

Page 65: f3 Course Notes

3: ACCOUNTING CONVENTIONS

65

Errors 5.5 These are material omissions from, or misstatements in, the financial statements that ought

to have been identified before the financial statements were finalised. An error is accounted for in exactly the same way as a change in accounting policy. For example, an entity may discover a material error in the 20X6 figures whilst producing the 20X7 financial statements. When the 20X7 financial statements are produced the 20X6 comparatives should be restated and the error corrected.

Illustration: Capital Co depreciates its non-current assets on the reducing balance basis using a rate of 25%. The 20X6 financial statements showed the following information: 20X6 $'000 Non-current assets – net book value 500 Other assets 400 900 Share capital 50 Retained earnings 600 Liabilities 250 900 During the preparation of the 20X7 financial statements it was discovered that the accountant in charge of non-current assets had forgotten to charge depreciation in 20X6. The 20X6 financial statements should be restated to correct this error. The 20X7 financial statements will show: 20X7 20X6

$'000 Restated

$'000 Non-current assets – net book value (375 – 93.75) 281.25 375 Other assets 400 400 681.25 775 Share capital 50 50 Retained earnings (475 – 93.75) 381.25 475 Liabilities 250 250 681.25 775

5.6 Note that changes in accounting estimates are not examinable.

(500 – 125)

(600 – 125)

Page 66: f3 Course Notes

3: ACCOUNTING CONVENTIONS

66

6 Chapter summary Section Topic Summary

2 The IASB’s Framework for the Preparation and Presentation of Financial Statements

The IASB Framework provides a set of principles on which financial accounting is based.

The objective of financial statements is to provide information on an entity’s financial position, financial performance and financial adaptability. The accruals basis requires that transactions are recognised when they occur rather than when any cash is received or paid. The going concern basis assumes that the entity will continue in operation for the foreseeable future. In order for the information in the financial statements to be useful it should posses the qualitative characteristics of understandability, relevance, reliability and comparability.

3 Alternative valuation bases

Most transactions are recorded at their historic cost - whilst this is a reliable figure it can soon become out of date, especially when prices are rising. Alternative valuation bases exist and these include: replacement cost, net realisable value and economic value (present value).

4 Other examinable concepts and conventions

The business entity concept states that a business is a separate entity from its owners. The money measurement concept states that only items which can be measured in monetary terms can be shown in the financial statements. The duality concept forms the basis of double entry bookkeeping and states that every transaction has two effects. The historic cost convention records items at their value when the transaction took place.

5 Accounting policies, changes in accounting estimates and errors (IAS 8

An entity should apply its accounting policies consistently from year to year. If an entity changes its accounting policy this is applied retrospectively and the comparative figures are restated and drawn up using the current year policies. The correction of an error also requires the comparative figures to be restated to correct the error.

Page 67: f3 Course Notes

3: ACCOUNTING CONVENTIONS

67

Chapter Summary

The objective of financial statements

Accounting conventions

Underlying assumptions

Qualitative characteristics of financial information

Elements of financial statements

IASB Framework

Other issues

Concepts and conventions Alternative valuation bases

IAS 8: Accounting policies, changes in accounting estimates and errors

'To provide information about the financial position, financial performance and changes in financial position of an entity'

• Accruals • Going concern

• Understandability • Relevance • Reliability • Comparability

• Asset • Liability • Equity • Income • Expenses

• Business entity • Money measurement • Duality

• Historic cost • Replacement cost • Net realisable value • Economic value

• Changes in accounting policies • Errors

Page 68: f3 Course Notes

3: ACCOUNTING CONVENTIONS

68

Page 69: f3 Course Notes

69

Chapter 3: Questions

Page 70: f3 Course Notes

3: QUESTIONS

70

3.1 The Framework for the Preparation and Presentation of Financial Statements identifies two assumptions which are the bedrock of accounting. What are they?

A Consistency and prudence

B Accruals and going concern

C Materiality and separate entity (1 mark)

3.2 In which of the following circumstances can a change of accounting policy be made?

(i) If the directors want to improve the value of the statement of financial position (ii) If required by an accounting standard (iii) If it results in reliable and more relevant information

A (ii) only

B (i) and (ii)

C (ii) and (iii)

D (i), (ii) and (iii) (2 marks)

Page 71: f3 Course Notes

71

Chapter 3: Answers

Page 72: f3 Course Notes

3: ANSWERS

72

3.1 B

3.2 C

END OF CHAPTER

Page 73: f3 Course Notes

73

4

Syllabus Guide Detailed Outcomes Having studied Chapters 4 and 5 you will be able to:

• Identify and explain the function of the main data sources in an accounting system and how the accounting system provides useful information.

• Outline the contents and purpose of different types of business documentation such as an invoice.

• Identify the main types of business transactions, for example, sales, purchases, payments and receipts.

• Understand and apply the concept of double entry accounting, the duality concept and the accounting equation.

• Identify the main types of ledger account and illustrate how to balance and close a ledger account.

• Understand and illustrate the uses of journals and the posting of journal entries into ledger accounts.

• Identify correct journals from given narrative.

• Record credit sale, credit purchase and cash transactions in ledger accounts and day books.

• Understand and record sales and purchase returns.

• Understand the need for a record of petty cash transactions and security over the petty cash system.

• Describe the features and operation of a petty cash imprest system.

• Account for petty cash using imprest and non-imprest methods.

Exam Context Questions are unlikely to feature solely on this chapter; however, you should have a good understanding of what constitutes an asset, liability, capital, income and expense. You should also be aware of the principal contents of each book of prime entry and the purpose of the memorandum ledgers.

Qualification Context These topics are only examined in Financial Accounting.

Sources, records and books of prime entry

Page 74: f3 Course Notes

4: SOURCES, RECORDS AND BOOKS OF PRIME ENTRY

74

Overview

Income statement

Memorandum ledgers

Sources, records and books of prime entry

Statement of financial position

Books of prime entry

Cash book Sales day book

Purchase day book

Petty cash book

Journal book

Page 75: f3 Course Notes

4: SOURCES, RECORDS AND BOOKS OF PRIME ENTRY

75

1 Statement of financial position 1.1 An individual could prepare a list of everything they own and everything they owe.

Lecture example 1 Idea generation Required List out everything you own and owe.

Solution (a) Own (b) Owe

1.2 For a business, this list is formalised as a statement of financial position and show the entity's assets and liabilities. (a) Asset: is a resource controlled by the entity as a result of past events and from

which future economic benefits are expected to flow to the entity. (b) Liability: is a present obligation of the entity arising from past events, the

settlement of which is expected to result in an outflow of economic benefits.

Page 76: f3 Course Notes

4: SOURCES, RECORDS AND BOOKS OF PRIME ENTRY

76

Proforma statement of financial position – sole trader 1.3 Statement of financial position as at 31 December 20X7:

$ $ ASSETS Non-current assets

Land and buildings 100,000 Office equipment 50,000 Motor vehicles 30,000 Furniture and fixtures 20,000

200,000 Current assets

Inventories 50,000 Trade receivables 30,000 Less: allowance for receivables (2,000) 28,000 Prepayments 5,000 Cash in hand and at bank 7,000

90,000 Total assets 290,000 CAPITAL AND LIABILITIES Capital

Capital 170,000 Profit 45,000 Less: drawings (25,000)

190,000 Non-current liabilities

Bank loans 40,000 Current liabilities

Bank overdraft 16,000 Trade payables 40,000 Accruals 4,000

60,000 Total capital and liabilities 290,000

Key features 1.4 (a) Always headed as at, for the date of the statement of financial position.

(b) Non-current assets - assets held and used in the business over the long-term (i.e. more than one year).

(c) Current assets - not non-current assets! Conventionally listed in increasing order of liquidity (i.e. closeness of assets to cash).

(d) Capital - what the business owes the proprietor/owner. In this case the sole trader owns all of the business, i.e. its total net worth. ∴ CAPITAL = ASSETS - LIABILITIES = NET ASSETS

Page 77: f3 Course Notes

4: SOURCES, RECORDS AND BOOKS OF PRIME ENTRY

77

(e) Don't include a caption (item heading) if there isn’t a value for it. The statement of financial position is a snapshot of the business at one point in time.

2 The income statement

Profit – example 2.1 Suppose a business buys three books for $10 each. Then it sells them for $15 each:

$ Sales 45 Income Cost of sales (30) Expenditure Gross profit 15 Profit is the excess of total income over total expenditure. NOTE: The business may have other expenses such as rent, telephone bills, etc. to take off before the ‘true’ profit is shown.

Proforma income statement - sole trader 2.2 Income statement for the year ended 31 December 20X7:

$ $ Sales 200,000 Less: Cost of sales Opening inventories 40,000 Purchases 110,000 Carriage inwards 20,000 170,000 Closing inventories (50,000) (120,000) Gross profit 80,000 Sundry income 5,000 Discounts receivable 3,000 88,000 Less: Expenses Rent 11,000 Carriage outwards 4,000 Telephone 1,000 Electricity 2,000 Wages and salaries 9,000 Depreciation 7,000 Bad and doubtful debts 3,000 Motor expenses 5,000 Discounts allowable 1,000 (43,000) Profit for the year 45,000

Page 78: f3 Course Notes

4: SOURCES, RECORDS AND BOOKS OF PRIME ENTRY

78

Key features 2.3 (a) Headed up with the period for which the income and expenses are being included.

(b) The top part Sales X Cost of sales (X) Gross profit X

is called the trading account as it records just the trading activities (buying and selling) of the business.

(c) Sundry income includes items like bank account interest. (d) Do not include nil value captions. The income statement is a summary of the business' performance over a period of time – think of it as a DVD!

3 Relationship between the statement of financial position and the income statement

3.1 Statement of financial position – shows the worth of business at a point in time. Income statement – shows the trading activities over a period of time (financial performance).

3.2 The accounting period is the period for which the income statement was prepared. This is usually a year.

3.3 Therefore, there will be a statement of financial position at the beginning of the year (prior year end) and at the end of the accounting period. The income statement is for the intervening period.

Income statement for the year ended 31.12.X7

Statement of financial

position as at

31.12.X6

Statement of financial

position as at

31.12.X7

Page 79: f3 Course Notes

4: SOURCES, RECORDS AND BOOKS OF PRIME ENTRY

79

4 From business transactions to financial statements 4.1 A business will enter into a number and variety of transactions during an accounting period:

CASH TRANSACTIONS

Sales Purchases Wages Stationery Acquisition of non-current assets

CREDIT TRANSACTIONS Sales Purchases

Ultimately all of these transactions must be summarised in the business' financial statements (ie the statement of financial position and income statement).

4.2 This is achieved by having accounting records to record each stage of the process:

Assorted transactions (eg invoices) Categorised (in Books of Prime Entry) Summarised (eg nominal ledger, trial balance) FINANCIAL STATEMENTS (eg Statement of Financial Position and Income Statement)

Page 80: f3 Course Notes

4: SOURCES, RECORDS AND BOOKS OF PRIME ENTRY

80

5 Books of prime entry 5.1 The business' transactions are categorised with other similar transactions in the books of

prime entry.

5.2

Cash book 5.3 (a) Records receipts and payments into and out of the bank.

(b) For exam purposes often assumed to be two books, one for receipts, one for payments.

Cash book (receipts) 5.4 Example:

Date Narrative Total $

Capital $

Sales $

Receivables$

2.1.X7 F. Bloggs 4,000 4,000 5.1.X7 J. Spalding 200 200 6.1.X7 J. Smith 500 500 4,700 4,000 500 200

total cash

received reason why cash was

received

Books of prime entry

Cash book Sales day book

Purchase day book

Petty cash book

Journal book

Receipts

Cash transactions Credit purchases

Payments

Small cash transactions

Adjustments and errors

Credit sales

Page 81: f3 Course Notes

4: SOURCES, RECORDS AND BOOKS OF PRIME ENTRY

81

Cash book (payments) 5.5 Example:

Date Narrative Total $

Purchases$

Van $

Rent $

Payables $

Petty cash $

Drawings$

6.1.X7 Manley & Co.

350 350

6.1.X7 Petty Cash 50 50 8.1.X7 Digby Co 1,000 1,000 1,400 1,000 350 50

total cash payment

reason why payment

was made

Sales day book 5.6 Lists all sales made on credit, i.e. each individual invoice raised.

5.7 Example:

Date Customer $ 3.1.X7 J. Spalding 200 5.1.X7 G. McGregor 400 8.1.X7 J. Spalding 400 14.1.X7 G. McGregor 300 TOTAL 1,300

Purchase day book 5.8 Lists all purchases made on credit, i.e. each individual invoice received.

5.9 Example:

Date Supplier $ 1.1.X7 Tewson Co. 400 4.1.X7 Manley & Co. 350 16.1.X7 Manley & Co. 200

TOTAL 950

Page 82: f3 Course Notes

4: SOURCES, RECORDS AND BOOKS OF PRIME ENTRY

82

Petty cash book 5.10 (a) Records the movement of physical cash (kept on the premises) in and out of the

petty cash tin. (b) Used for small incidental expenses.

5.11 Example:

Receipts Payments

Date Narrative Total $

Date Narrative Total $

Stationery$

Travel$

6.1.X7 Cheque cashed

50 7.1.X7

City Stationers

10

10

8.1.X7 F. Bloggs Metro fare

2

2

12 10 2

Controlling petty cash – the imprest system An imprest system acts as an accounting control by having a set amount of petty cash. 5.12 (a) Pre-set limit, say $50.

(b) Voucher filled in when money is taken out to pay expenses. (c) At any time, vouchers + cash = pre-set limit. (d) At the end of the week/month, the petty cash book is filled in from the vouchers. (e) The amount needed to bring the balance back up to the pre-set limit = money spent.

Journal book 5.13 Certain transactions do not ‘fit’ in the main books, for example:

(a) period end adjustments (b) correction of errors The journal book lists these sundry transactions.

6 Memorandum ledgers

Purpose 6.1 To know how much is owed by a particular customer or to a certain supplier at a point in

time. For example, the sales day book shows the sales made on credit to all customers and the cash book receipts shows the cash received from all sources. J. Spalding owes the business $400 but this cannot be seen from the books of prime entry without trawling back through the detailed information. A separate memorandum ledger is kept to show this information.

Quick Quiz Q5, Q6

Page 83: f3 Course Notes

4: SOURCES, RECORDS AND BOOKS OF PRIME ENTRY

83

6.2 There are two types of memorandum ledgers kept by the business: (a) Receivables ledger – showing how much is owed by each individual customer. (b) Payables ledger – showing how much is owed to each individual supplier.

6.3 The entries in these ledgers are made by rearranging the information in the day books into individual customer and supplier accounts.

Receivables ledger 6.4 Example:

J. Spalding (Customer)

Date Narrative Sales $

Cash $

Total $

3.1.X7 5.1.X7 8.1.X7

Invoice 1032 Cash received Invoice 1101

200

400

200

200 –

400

G. McGregor (Customer)

Date Narrative Sales $

Cash $

Total $

5.1.X7 14.1.X7

Invoice 1033 Invoice 1129

400 300

400 700

Payables ledger 6.5 Example:

Tewson Co. (Supplier)

Date Cash $

Purchases $

Total $

1.1.X7 Invoice A112

400

400

Manley & Co. (Supplier)

Date Cash $

Purchases $

Total $

4.1.X7 6.1.X7 16.1.X7

Invoice 063 Cash book Invoice 097

350

350

200

350 –

200

Page 84: f3 Course Notes

4: SOURCES, RECORDS AND BOOKS OF PRIME ENTRY

84

7 Chapter summary Section Topic Summary 1 Statement of financial

position The statement of financial position shows the assets and liabilities of a business at a particular point in time.

2 The income statement The income statement shows its performance over a period.

3 The relationship between the statement of financial position and the income statement

The income statement largely explains the movement between the business’ assets and liabilities at the beginning of the year and at the end of the year.

4 From business transactions to financial statements

A business will enter many transactions during the year. All of these need to be recorded and summarised to produce the entity’s financial statements.

5 Books of prime entry The business’ transactions must first be categorised into the books of prime entry. The cash book records money paid in to and out of the bank account; the sales day book records credit sales; the purchase day book records credit purchases; the petty cash book records transactions made in petty cash and the journal book is used to correct errors and make other adjustments such as accruals and prepayments. The totals on these books are then summarised in the nominal ledger.

6 Memorandum ledgers There are two memorandum ledgers: the receivables ledger and the payables ledger. The receivables ledger shows how much the business is owed by each individual customer at a point in time and the payables ledger show how much it owes to each individual supplier at any point in time.

Page 85: f3 Course Notes

4: SOURCES, RECORDS AND BOOKS OF PRIME ENTRY

85

Chapter Summary

Income statement

Memorandum ledgers

Sources, records and books of prime entry

Statement of financial position

Books of prime entry

Cash book Sales day book Purchase day book

Petty cash book Journal book

• Shows the assets and liabilities of the business at a point in time.

• Shows the income and expenses for a business over a period of time, usually a year

• Receivables ledger: - amount owed by a particular

customer • Payables ledger: - amount owed to a particular

supplier

'Categorise similar transactions together'

• Cash receipts into the bank

• Cash payments from the bank

• Credit sales • Credit purchases • Small cash transactions made via the petty cash tin

• Correction of errors and period end adjustments

Page 86: f3 Course Notes

4: SOURCES, RECORDS AND BOOKS OF PRIME ENTRY

86

Page 87: f3 Course Notes

87

Chapter 4: Question

Page 88: f3 Course Notes

4: QUESTION

88

4.1 Which of the following is not a book of prime entry?

A Wages day book B Cash book C Sales ledger (1 mark)

Page 89: f3 Course Notes

89

Chapter 4: Answer

Page 90: f3 Course Notes

4: ANSWER

90

4.1 C

END OF CHAPTER

Page 91: f3 Course Notes

91

5

Syllabus Guide Detailed Outcomes Having studied Chapters 4 and 5 you will be able to:

• Identify and explain the function of the main data sources in an accounting system and how the accounting system provides useful information.

• Outline the contents and purpose of different types of business documentation such as an invoice.

• Identify the main types of business transactions, for example, sales, purchases, payments and receipts.

• Understand and apply the concept of double entry accounting, the duality concept and the accounting equation.

• Identify the main types of ledger account and illustrate how to balance and close a ledger account.

• Understand and illustrate the uses of journals and the posting of journal entries into ledger accounts.

• Identify correct journals from given narrative.

• Record credit sale; credit purchase and cash transactions in ledger accounts and day books.

• Understand and record sales and purchase returns.

• Understand the need for a record of petty cash transactions and security over the petty cash system.

• Describe the features and operation of a petty cash imprest system.

• Account for petty cash using imprest and non-imprest methods.

Exam Context Your understanding of double entry will be crucial to passing Financial Accounting. Whilst an individual question may not ask you to produce a double entry it will be instrumental in answering the question. For example, a question may ask you to derive the income statement expense for electricity where amounts need to be accrued at the year end. You will only get this right if you understand the double entry for recording expenses and accruals. A question could also describe a transaction and ask you to identify the correct double entry to record this.

Qualification Context Being confident at double entry will help you account for many of the more complex accounting standards you will meet in the Fundamentals level paper, Financial Reporting (F7) and the Professional level paper, Corporate Reporting (P2).

Ledger accounts and double entry

Page 92: f3 Course Notes

5: LEDGER ACCOUNTS AND DOUBLE ENTRY

92

Overview

Ledger accounts and double entry

Balancing off

Ledger accounts Double entry

Debit Credit

Page 93: f3 Course Notes

5: LEDGER ACCOUNTS AND DOUBLE ENTRY

93

1 Introduction 1.1 This chapter is designed to enable you to explain the principles of double entry and apply

these principles to the preparation of accounting records within the nominal/general ledger.

1.2 In Chapter 4 we saw how transactions were categorised in books of prime entry, the next step is to summarise the information in a format nearer to that of the final financial statements.

The nominal ledger 1.3 (a) Each item in the statement of financial position or income statement will have an

"account" (which might be a page in a book or a record on a computer). (b) All the accounts are collected together in the nominal ledger. (c) The books of prime entry are totalled up and two entries will be made in these

accounts with each of these totals – this is called double entry.

The dual effect 1.4 The method used stems from the fact that every transaction affects two things, for example:

(a) A sole trader pays $6,000 in the business bank account: Cash increases by $6,000 Capital increases by $6,000

(b) A sole trader purchases goods on credit for $400: Purchases increase by $400 Trade payables increase by $400

2 Ledger accounts (T-accounts) 2.1 Debit CAPITAL Credit

$ $ Decrease Capital Increase Capital We make two entries from each total extracted from the books of prime entry, and call one a Debit (Dr), and the other one a Credit (Cr).

TOTAL DEBITS = TOTAL CREDITS

Page 94: f3 Course Notes

5: LEDGER ACCOUNTS AND DOUBLE ENTRY

94

Principles of double entry bookkeeping 2.2 The cash account is a good starting point: Dr CASH Cr

$ $ CASH IN = DEBIT CASH OUT = CREDIT

General rules 2.3 (a) DEBIT entry represents:

(i) an increase in an asset; (ii) a decrease in a liability; (iii) an item of expense.

(b) CREDIT entry represents: (i) an increase in a liability; (ii) a decrease in an asset; (iii) an item of income.

This can be remembered as follows

Debits Credits (increase) (increase) Expenses Liabilities Assets Income Drawings Capital

Quick Quiz

Q1 - 4

Page 95: f3 Course Notes

5: LEDGER ACCOUNTS AND DOUBLE ENTRY

95

Lecture example 1 Preparation question Required

What is the double entry for each of the following? Explain each entry in terms of the general rules above.

Solution

Transaction Debit Credit (a)

Sales for cash.

(b)

Sales on credit.

(c)

Purchase for cash.

(d)

Purchase on credit.

(e)

Pay electricity bill.

(f)

Receive cash from a credit customer.

(g)

Pay cash to a credit supplier.

Page 96: f3 Course Notes

5: LEDGER ACCOUNTS AND DOUBLE ENTRY

96

Transaction Debit Credit (h)

Borrow money from the bank.

Lecture example 2 Technique demonstration Douglas Douglas had the following transactions during January: (1) Introduced $5,000 cash as capital; (2) Purchased goods on credit from Richard, worth $2,000; (3) Paid rent for one month, $500; (4) Paid electricity for one month, $200; (5) Purchased car for cash, $1,000; (6) Sold half of the goods on credit to Tish for $1,750; (7) Drew $300 for his own expenses; (8) Sold goods for cash, $2,100. Required Post transactions (1) to (8) to the relevant ledger accounts.

Solution

Cash $ $

Capital $ $

Page 97: f3 Course Notes

5: LEDGER ACCOUNTS AND DOUBLE ENTRY

97

Trade payables $ $

Purchases $ $

Rent $ $

Electricity $ $

Car $ $

Drawings $ $

Page 98: f3 Course Notes

5: LEDGER ACCOUNTS AND DOUBLE ENTRY

98

Trade receivables $ $

Sales $ $

3 Flow of information 3.1 In Lecture example 2 the original transactions were posted to the ledger accounts. A

business would firstly categorise this information in the books of prime entry. The totals from the books of prime entry are then posted to the nominal ledger using double entry.

3.2

Page 99: f3 Course Notes

5: LEDGER ACCOUNTS AND DOUBLE ENTRY

99

4 Balancing off the ledger accounts 4.1 The totals from the books of prime entry may be posted to the nominal ledger each month. A

business will want to know the balance on each account. This is done by 'balancing off' each account.

Lecture example 3 Technique demonstration The following information has been posted to the cash account below. Required Balance off the cash account to determine the amount of cash held at the end of January.

Solution

Dr Cash Cr $ $ 2/1 Sales 500 1/1 Purchases 300 10/1 Sales 500 25/1 Telephone 50

Steps 4.2 (1) Add the debit and credit sides separately.

(2) Fill in the higher of the two totals on both sides. (3) Literally 'balance' the account (what number do we need and on which side to make

the two sides equal?) – balance c/d (4) Complete the 'double entry' – balance b/d on opposite side.

Lecture example 4 Technique demonstration Douglas Refer to Lecture example 2 on page 96. Required

Balance off the ledger accounts for Douglas

Solution Complete in the solution space for Lecture example 2.

Page 100: f3 Course Notes

5: LEDGER ACCOUNTS AND DOUBLE ENTRY

100

5 Chapter summary Section Topic Summary

1 Introduction In chapter 4 the totals on the books of prime entry were summarised in the nominal ledger. These amounts are posted to the nominal ledger using double entry. The principles of double entry work on the basis that for each debit entry there must be a credit entry. This is also known as the dual effect.

2 Ledger accounts A debit entry increases assets, expenses and drawings and a credit entry increases liabilities, income and capital – this can be remembered as DEAD CLIC.

3 Flow of information A business’ transactions are categorised in the books of prime entry and the totals are then posted to the nominal ledger. A trial balance (chapter 6) can then be extracted from the balances on the nominal ledger accounts and the statement of financial position and income statement produced.

4 Balancing off the ledger accounts

At the end of each period the nominal ledger accounts (T accounts) are 'balanced off' to determine the closing balance on each account.

Page 101: f3 Course Notes

5: LEDGER ACCOUNTS AND DOUBLE ENTRY

101

Chapter Summary

Ledger accounts and double entry

Balancing off

Ledger accounts Double entry

Debit Credit

• The totals from the books of prime entry are totalled and then are posted to the nominal ledger.

• increases: - expenditure - asset - drawings

• increases: - liability - income - capital

• Steps: (1) Add the debit and credit sides

separately (2) Fill in the higher of the two totals

on both sides (3) Balance the account by inserting

the 'balance c/d' on the relevant side

(4) Complete the double entry and put the 'balance b/d' on the opposite side

Page 102: f3 Course Notes

5: LEDGER ACCOUNTS AND DOUBLE ENTRY

102

Page 103: f3 Course Notes

103

Chapter 5: Questions

Page 104: f3 Course Notes

5: QUESTIONS

104

5.1 A credit balance of $3,000 brought down on X Co’s account in Y Co’s books means that

A X Co is owed $3,000 by Y Co

B Y Co is owed $3,000 by X Co

C Y Co has sold $3,000 of goods to X Co (1 mark)

5.2 Which of the statements below best describes the nominal ledger?

A A list of all assets and liabilities at a point in time

B A collection of accounts to record the transactions of the business

C A record of amounts owed to/from individual suppliers and customers

D An initial record of internally generated transactions (2 marks)

Page 105: f3 Course Notes

105

Chapter 5: Answers

Page 106: f3 Course Notes

5: ANSWERS

106

5.1 A The balance represents the outstanding amount i.e. purchases less cash paid.

5.2 B

END OF CHAPTER

Page 107: f3 Course Notes

107

6

Syllabus Guide Detailed Outcomes Having studied this chapter you will be able to:

• Identify the purpose of a trial balance.

• Extract ledger balances into a trial balance.

• Prepare extracts of an opening trial balance.

• Identify and understand the limitations of a trial balance.

Exam Context Questions on this chapter may require you to derive missing figures (for example, profit for the period) using the accounting equation and identify the correct double entry to record transactions such as closing inventory or drawings.

Qualification Context Financial Accounting is the only paper where you are required to produce financial statements for a sole trader. Financial statements for limited liability companies are tested in detail in the Fundamentals level paper, Financial Reporting (F7) and the Professional level paper, Corporate Reporting (P2).

From trial balance to financial statements

Page 108: f3 Course Notes

6: FROM TRIAL BALANCE TO FINANCIAL STATEMENTS

108

Overview

From trial balance to financial statements

Income statement

Trial balance Closing inventory adjustment

Statement of financial position

Accounting equation

Page 109: f3 Course Notes

6: FROM TRIAL BALANCE TO FINANCIAL STATEMENTS

109

1 Introduction 1.1 We saw in Chapters 4 and 5 that:

• transactions are categorised in the books of prime entry; • the totals are then posted to the ledger accounts in the nominal ledger using double

entry; • the ledger accounts are then balanced off and the balances brought down.

2 The trial balance 2.1 The trial balance consists of a list of the balances brought down on each ledger account,

separated in to debits and credits as below.

Example 2.2 Miss Smith – Trial Balance at as 31 December 20X7:

Account

Debit $

Credit $

Cash Capital Sales Purchases Furniture Electricity Telephone Drawings

Total

720

1,100 500 120 60

200 2,700

500

2,200

2,700

2.3 The trial balance should balance, i.e.

Total debits = Total credits

If the trial balance doesn't balance then an error must have occurred. The correction of errors is covered in Chapter 16.

Page 110: f3 Course Notes

6: FROM TRIAL BALANCE TO FINANCIAL STATEMENTS

110

Lecture example 1 Technique demonstration Douglas Refer to Lecture example 2 in Chapter 5 on pages 96 to 98 where the ledger accounts were balanced off. Using the ledger accounts for Douglas, prepare the trial balance as at the end of January.

Solution

Page 111: f3 Course Notes

6: FROM TRIAL BALANCE TO FINANCIAL STATEMENTS

111

3 The closing inventory adjustment

Objective 3.1 Whilst a business will purchase items to sell during the year it is unlikely that all of them will

have been sold by the year end. The items still held at the year end are known as inventories. These are an asset of the business and so should be included in inventories in the statement of financial position. Also when a business determines its profit for the year it should match the sales revenue earned to the cost of goods it sold, i.e. cost of sales.

Lecture example 2 Preparation question

Colin opens a business selling cordless telephones. In the first month he buys 50 phones for $20 each, and sells 20 for $30 each. Complete the trading account below.

Solution $ $ Sales Cost of sales Purchases Less: closing inventories Gross profit

Accounting treatment 3.2 The closing inventory adjustment is accounted for via a journal entry. The double entry is:

Dr Inventories (SOFP) Cr Closing inventories (COS – I/S)

3.3 This adjustment is usually made after the preliminary trial balance has been prepared. 3.4 Last period's closing inventories will become this period's opening inventories. These items

will be sold in the year and so will form part of cost of sales. As the items are sold they will no longer be an asset of the business and should be removed from the statement of financial position. The double entry is:

Dr Opening inventories (COS – I/S) Cr Inventories (SOFP)

This can be done as soon as the new period begins.

Page 112: f3 Course Notes

6: FROM TRIAL BALANCE TO FINANCIAL STATEMENTS

112

4 The income statement 4.1 The income statement is part of the double entry system and can be shown as a T-account.

Completing the income statement 4.2 The balances on all the income and expenditure T-accounts are transferred to the income

statement and the closing inventory adjustment is made.

4.3 The income and expenditure accounts have now been closed out and a new account will be created for each income and expenditure item next year.

Lecture example 3 Technique demonstration Douglas Refer to Lecture example 1 on page 110. The cost of goods remaining unsold at year end was $250. Required

Prepare an income statement in ledger account form.

Solution

Income Statement a/c

Page 113: f3 Course Notes

6: FROM TRIAL BALANCE TO FINANCIAL STATEMENTS

113

5 The statement of financial position

Completing the statement of financial position 5.1 Statement of financial position:

(a) lists all ledger accounts with balances remaining; (b) i.e. all assets and liabilities; (c) is not part of double entry system so these balances are not transferred out.

5.2 At end of period, clear balances on income statement and drawings to capital account.

Lecture example 4 Technique demonstration Douglas Refer to Lecture example 1 on page 110 and lecture example 3 on page 112. Required

Draw up an income statement for the period and a statement of financial position at the end of January.

Solution DOUGLAS INCOME STATEMENT FOR THE MONTH OF JANUARY

$ $

Sales

Less cost of sales:

Purchases

Less: closing inventories

Gross profit

Less expenses:

Rent

Electricity

Net profit

Page 114: f3 Course Notes

6: FROM TRIAL BALANCE TO FINANCIAL STATEMENTS

114

DOUGLAS STATEMENT OF FINANCIAL POSITION AS AT 31 JANUARY

NON-CURRENT ASSET $ $

Motor vehicle

CURRENT ASSETS

Inventories

Trade receivables

Cash

PROPRIETOR’S INTEREST $ $

Capital introduced on 1 January

Profit for the year

Less: drawings

Balance 31 January

CURRENT LIABILITIES

Trade payables

Page 115: f3 Course Notes

6: FROM TRIAL BALANCE TO FINANCIAL STATEMENTS

115

Lecture example 5 Technique demonstration Douglas Refer to Lecture example 4 on page 113. Required

Transfer the profit and drawings to the capital account.

Solution

Drawings 5.3 Drawings are amounts being taken out of a business by its owner. Drawings are generally in

the form of cash, but an owner may also take inventory out of the business. Drawings of inventories are recorded at the cost of the inventories not the sales price.

Page 116: f3 Course Notes

6: FROM TRIAL BALANCE TO FINANCIAL STATEMENTS

116

6 The accounting equation 6.1 The accounting equation expresses the statement of financial position as an equation.

6.2 At its most simple:

Different types of liabilities (credits)

CAPITAL PROFIT(less drawings) PAYABLES

ASSETS(debits) = LIABILITIES

(credits)

Proprietor’s interest

Lecture example 6 Technique demonstration Douglas Refer to Lecture example 4. Required Prepare the accounting equation for Douglas.

Solution

Page 117: f3 Course Notes

6: FROM TRIAL BALANCE TO FINANCIAL STATEMENTS

117

7 Chapter summary Section Topic Summary

2 The trial balance The trial balance consists of a list of the balances brought down on each ledger account.

3 The closing inventory adjustment

At the end of the year an adjustment must be made for closing inventory to match sales revenue to the cost of making those sales and also to reflect the fact that the inventories are an asset of the business. The opening inventory balance should also be transferred to cost of sales.

4 The income statement The balances on all of the income and expenditure ledger accounts are transferred to the income statement along with any adjustments that will affect profit (such as closing inventory).

5 The statement of financial position

The statement of financial position lists out the balances on all of the asset and liability ledger accounts.

6 The accounting equation

The accounting equation expresses the statement of financial position as an equation:

Assets = capital + profit – drawings + payables

8 Double Entry Summary for Chapter 6 8.1 Closing inventory adjustment:

8.2 Opening inventory adjustment:

8.3 The accounting equation:

Dr Inventories (SOFP) Cr Closing inventories (I/S)

Dr Opening inventories (I/S) Cr Inventories (SOFP)

Assets = Liabilities Assets = Capital + Profit - Drawings + Payables

Page 118: f3 Course Notes

6: FROM TRIAL BALANCE TO FINANCIAL STATEMENTS

118

Chapter Summary

From trial balance to financial statements

Income statement

Trial balance Closing inventory adjustment

Statement of financial position

Accounting equation

'A list of the balances brought down on each ledger account'

• Items held in inventories at the year end should be recorded as an asset in the statement of financial position and will reduce cost of sales

• Record via a journal entry: Dr Inventories (SOFP) Cr Closing inventories (COS – I/S)

• Expresses the statement of financial position as an equation • Assets = liabilities

Page 119: f3 Course Notes

119

Chapter 6: Questions

Page 120: f3 Course Notes

6: QUESTIONS

120

6.1 At the end of the accounting period and after the statement of financial position and income statement have been prepared for a sole trader:

A All journals are reversed

B The balances on asset and liability accounts are transferred to the capital account

C The balances on the income statement and drawings account are transferred to the capital account

D Balances are carried forward on all the accounts in the nominal ledger (2 marks)

6.2 A business has cash of $1,100, trade payables of $2,500, a mortgage liability of $8,000 and land of $16,000.

What is the proprietor's interest? $ (2 marks)

6.3 Joe, a sole trader, set up business on 1 October 20X6 with $40,000 of his own money. During the year to 30 September 20X7 he won $50,000 on the lottery and paid $30,000 of this into his business. He took cash drawings of $5,000 during the year and at 30 September 20X7 the net assets of the business totalled $59,000.

What was the profit or loss of the business for the year ended 30 September 20X7?

A $4,000 profit

B $6,000 profit

C $16,000 loss

D $6,000 loss (2 marks)

6.4 Joan

Joan, a second hand bookseller, has been in business for two months. In this time she:

(1) paid in cash $5,000 as capital;

(2) took the lease of a stall and paid two months’ rent. The annual rental was $1,200;

(3) purchased, on credit from J Fox, books at cost of $825;

(4) spent $420 cash on the purchase of other books from W Smith;

(5) paid an odd-job man $75 to paint the exterior of the stall and repair a broken lock;

(6) put an advertisement in the local paper at a cost of $10;

(7) sold three volumes containing "The Complete Works of Shakespeare" to an American for $60 cash;

(8) sold six similar sets on credit to a local school for $300;

(9) paid J Fox $525 on account for the amount due to him;

(10) received $200 from the school;

(11) purchased cleaning materials at a cost of $10 and paid a char lady $30;

(12) took $100 from the business to pay for her own personal expenses;

(13) made other cash sales during the two months of $1,500;

(14) all books had been sold by the end of two months.

Required

(a) Write up the relevant ledger accounts for these transactions. (b) Balance off all of the ledger accounts. (c) Prepare a trial balance, an income statement and a statement of financial position.

Page 121: f3 Course Notes

6: QUESTIONS

121

6.5 Brian Brian set himself up in business on 1 January selling ice creams. During his first two months in business he:

(1) Introduced $20,000 of cash as capital into the business;

(2) Purchased a second hand ice cream van from John. He paid John $10,500 cash;

(3) Paid Terry $200 to repair the ice cream machine in the van;

(4) Purchased on credit, inventories totalling $750;

(5) Spent $400 on petrol;

(6) Sold goods for $750 in cash;

(7) Paid $600 in tax and insurance;

(8) Made additional cash purchases of $80 for strawberry sauce and chocolate flakes;

(9) Withdrew $300 for his own expenses;

(10) The cost of goods remaining unsold was $500.

Required

(a) Post transactions (1) – (9) to the relevant ledger accounts. (b) Balance off the ledger accounts. (c) Prepare a trial balance. (d) Prepare an income statement in ledger account form (remembering to deal with item 10). (e) Draw up an income statement for the period and a statement of financial position at the end of

the period. (f) Transfer the loss and drawings to the capital account.

6.6 Dealers

On 1 January the proprietor’s interest in a business, Dealers, was $18,500. At 31 January the assets and liabilities of the business were as follows. $ Plant and equipment 10,000 Motor vehicles 5,000 Trade payables 3,000 Trade receivables 2,000 Inventories 4,500 Accrued expenses 250 Balance in the bank 3,500 Cash in the till 250 On 7 January the proprietor had paid in additional capital of $2,000. On 14 January he had taken goods at a cost of $350 for his own consumption and on 30 January had drawn cash of $1,250 from the business, for his own personal expenditure.

Required

(a) Calculate the net asset value at 1 January. (b) Calculate the net asset value of the business at 31 January. (c) Calculate the profit of the business for the month of January. (d) Show the accounting equation at 31 January.

Page 122: f3 Course Notes

6: QUESTIONS

122

Page 123: f3 Course Notes

123

Chapter 6: Answers

Page 124: f3 Course Notes

6: ANSWERS

124

6.1 C

6.2 $6,600 Dr Cr $ $ Cash 1,100 Trade payables 2,500 Mortgage liability 8,000 Land 16,000 Proprietor's interest (balancing figure) 6,600 17,100 17,100

6.3 D $ Net assets at 1.10.X6 40,000 Capital introduced 30,000 Drawings (5,000) ∴ loss for year (balancing figure) (6,000) Net assets at 30.9.X7 59,000

6.4 Joan

Bank (SOFP) $ $ (1) Capital 5,000 (2) Rent 200 (7) Sales 60 (4) Purchases 420 (10) Trade receivables 200 (5) Repairs 75 (13) Sales 1,500 (6) Advertising 10 (9) Trade payables 525 (11) Cleaning materials 10 (11) Cleaning 30 (12) Drawings 100 Balance c/d 5,390 6,760 6,760 Balance b/d 5,390

Capital (SOFP) $ $ Balance c/d 5,000 (1) Bank 5,000 5,000 5,000 Balance b/d 5,000

Rent (I/S) $ $ (2) Bank 200 Balance c/d 200 200 200 Balance b/d 200

Page 125: f3 Course Notes

6: ANSWERS

125

Trade payables (SOFP) $ $ (9) Bank 525 (3) Purchases 825 Balance c/d 300 825 825 Balance b/d 300

Purchases (I/S) $ $ (3) Trade payables 825 Balance c/d 1,245 (4) Bank 420 1,245 1,245 Balance b/d 1,245

Repairs (I/S) $ $ (5) Bank 75 Balance c/d 75 75 75 Balance b/d 75

Advertising (I/S) $ $ (6) Bank 10 Balance c/d 10 10 10 Balance b/d 10

Sales (I/S) $ $ Balance c/d 1,860 (7) Bank 60 (8) Trade receivables 300 (13) Bank 1,500 1,860 1,860 Balance b/d 1,860

Trade receivables (SOFP) $ $ (8) Sales 300 (10) Bank 200 Balance c/d 100 300 300 Balance b/d 100

Cleaning materials (I/S) $ $ (11) Bank 10 Balance c/d 10 10 10 Balance b/d 10

Page 126: f3 Course Notes

6: ANSWERS

126

Cleaning (I/S) $ $ (11) Bank 30 Balance c/d 30 30 30 Balance b/d 30

Drawings (SOFP) $ $ (12) Bank 100 Balance c/d 100 100 100 Balance b/d 100

Trial Balance

Debit Credit $ $ Bank 5,390 Capital 5,000 Rent 200 Trade payables 300 Purchases 1,245 Repairs 75 Advertising 10 Sales 1,860 Trade receivables 100 Cleaning materials 10 Cleaning 30 Drawings 100 7,160 7,160 Joan Income statement for the two months ended……

$ $ Sales 1,860 Purchases (1,245) Gross profit 615 Rent 200 Repairs 75 Advertising 10 Cleaning (10 + 30) 40 (325) Profit for the year 290

Page 127: f3 Course Notes

6: ANSWERS

127

Joan Statement of financial position as at…. $ Current Assets Trade receivables 100 Bank 5,390 5,490

Proprietor's Interest $ Capital 5,000 Profit 290 Less: drawings (100) 5,190 Current Liabilities Trade payables 300 5,490

6.5 Brian

(a) Bank (SOFP) $ $ (1) Capital 20,000 (2) Van 10,500 (6) Sales 750 (3) Repairs & Maintenance 200 (5) Petrol 400 (7) Tax & Insurance 600 (8) Purchases 80 (9) Drawings 300

Capital (SOFP)

$ $ (1) Bank 20,000

Van (SOFP)

$ $ (2) Bank 10,500

Repairs & Maintenance (I/S)

$ $ (3) Bank 200

Purchases (I/S)

$ $ (4) Trade payables 750 (8) Bank 80

Trade payables (SOFP)

$ $ (4) Purchases 750

Page 128: f3 Course Notes

6: ANSWERS

128

Petrol (I/S) $ $ (5) Bank 400

Sales (I/S)

$ $ (6) Bank 750

Tax & Insurance (I/S)

$ $ (7) Bank 600

Drawings (SOFP) $ $ (9) Bank 300

(b) Bank (SOFP) $ $ (1) Capital 20,000 (2) Van 10,500 (6) Sales 750 (3) Repairs & Maintenance 200 (5) Petrol 400 (7) Tax & Insurance 600 (8) Purchases 80 (9) Drawings 300 Bal c/d 8,670 20,750 20,750 Bal b/d 8,670

Capital (SOFP)

$ $ Bal c/d 20,000 (1) Bank 20,000 20,000 20,000 Bal b/d 20,000

Van (SOFP)

$ $ (2) Bank 10,500 Bal c/d 10,500 10,500 10,500 Bal b/d 10,500

Repairs & Maintenance (I/S)

$ $ (3) Bank 200 Bal c/d 200 200 200 Bal b/d 200

Page 129: f3 Course Notes

6: ANSWERS

129

Purchases (I/S) $ $ (4) Trade payables 750 (8) Bank 80 Bal c/d 830 830 830 Bal b/d 830

Trade payables (SOFP)

$ $ Bal c/d 750 (4) Purchases 750 750 750 Bal b/d 750

Petrol (I/S) $ $ (5) Bank 400 Bal c/d 400 400 400 Bal b/d 400

Sales (I/S)

$ $ Bal c/d 750 (6) Bank 750 750 750 Bal b/d 750

Tax & Insurance (I/S)

$ $ (7) Bank 600 Bal c/d 600 600 600 Bal b/d 600

Drawings (SOFP)

$ $ (9) Bank 300 Bal c/d 300 300 300 Bal b/d 300

(c) Trial balance Debit Credit $ $ Bank 8,670 Capital 20,000 Van 10,500 Repairs and Maintenance 200 Purchases 830 Trade payables 750 Petrol 400 Sales 750 Tax & Insurance 600 Drawings 300 21,500 21,500

Page 130: f3 Course Notes

6: ANSWERS

130

(d) Income Statement $ $ Purchases 830 Sales 750 Gross profit c/d 420 Closing inventories 500 1,250 1,250 Petrol 400 Gross profit b/d 420 Repairs & Maintenance 200 Tax & Insurance 600 Net loss c/d 780 1,200 1,200 Net loss b/d 780

Purchases (I/S)

$ $ (4) Trade payables 750 (8) Bank 80 Balance c/d 830 830 830 Balance b/d 830 Income statement 830

Petrol (I/S) $ $ (5) Bank 400 Balance c/d 400 400 400 Balance b/d 400 Income statement 400

Repairs & Maintenance (I/S) $ $ (3) Bank 200 Balance c/d 200 200 200 Balance b/d 200 Income statement 200

Tax & Insurance (I/S) $ $ (7) Bank 600 Balance c/d 600 600 600 Balance b/d 600 Income statement 600

Sales (I/S)

$ $ Balance c/d 750 (6) Bank 750 750 750 Income statement 750 Balance b/d 750

Page 131: f3 Course Notes

6: ANSWERS

131

Inventories (SOFP) $ $ Closing inventories (I/S) 500

(e) Brian Income statement for the two months ended 28 February $ $ Sales 750 Less cost of sales: Purchases 830 Less: closing inventories (500) 330 Gross profit 420 Less expenses: Petrol 400 Repairs & Maintenance 200 Tax & Insurance 600 ((1,200) Net loss for the period (780) Brian Statement of financial position as at 28 February Non current assets $ Motor vehicles 10,500 Current assets 500 Inventories 500 Bank 8,670 Total assets 19,670 Proprietor’s interest $ $ Capital introduced on 1 January 20,000 Loss for the period (780) Less: drawings (300) (1,080) Balance at 28 February 18,920 Current liabilities Trade payables 750 Total capital and liabilities 19,670

(f) Drawings (SOFP)

$ $ (9) Bank 300 Bal c/d 300 300 300 Bal b/d 300 Capital a/c 300

Page 132: f3 Course Notes

6: ANSWERS

132

Income statement $ $ Purchases 830 Sales 750 Gross profit c/d 420 Closing inventories 500 1,250 1,250 Petrol 400 Gross profit b/d 420 Repairs & Maintenance 200 Tax & Insurance 600 Net loss c/d 780 1,200 1,200 Net loss b/d 780 Capital a/c 780

Capital account (SOFP) $ $ Bal c/d 20,000 (1) Bank 20,000 20,000 20,000 Drawings 300 Bal b/d 20,000 Net loss 780 Bal c/d 18,920 20,000 20,000 Bal b/d 18,920

6.6 Dealers

(a) Net assets = proprietor’s interest ∴ Net assets at 1 January are $18,500

(b) Net assets = assets – liabilities At 31 January the assets total: $ $ Plant and equipment 10,000 Motor vehicles 5,000 Trade receivables 2,000 Inventories 4,500 Balance in the bank 3,500 Cash in the till 250 25,250 At 31 January the liabilities total: Trade payables 3,000 Accrued expenses 250 3,250 ∴ Net assets at 31 January 22,000

(c) Profit = Increase in net Drawings between Additional capital

assets between + the same two – paid in between two points in points in time the same two points time in time

∴ Profit for the month of January = (22,000 – 18,500) + (350 + 1,250) – 2,000 = $3,100

Page 133: f3 Course Notes

6: ANSWERS

133

(d) Accounting equation at 31 January ASSETS = CAPITAL + PROFIT – DRAWINGS + PAYABLES

25,250 = 20,500 + 3,100 – 1,600 + 3,250 $ $ Plant & equipment 10,000 Capital at 1 January 18,500 Motor vehicles 5,000 Additional capital 2,000 Inventories 4,500 Profit 3,100 Trade receivables 2,000 23,600 Balance in bank 3,500 Less: drawings 1,600 Cash in the till 250 22,000 Trade payables 3,000 Accrued expenses 250 25,250 25,250

Page 134: f3 Course Notes

6: ANSWERS

134

END OF CHAPTER

Page 135: f3 Course Notes

135

7

Syllabus Guide Detailed Outcomes Having studied this chapter you will be able to:

• Understand the general principles of the operation of a sales tax.

• Calculate sales tax on transactions and record the consequent accounting entries.

Exam Context This topic is likely to be tested in two main ways. You may be asked to identify the correct journal entry to post sales and purchases transactions including sales tax. You may also be required to consider how sales tax affects the calculation of amounts to be capitalised for non-current assets and the amount for trade receivables where discounts are offered.

Qualification Context Financial Accounting introduces accounting for sales tax. More detailed rules and calculations relating to this area are covered in the Fundamentals level paper, Taxation (F6).

Sales tax

Page 136: f3 Course Notes

7: SALES TAX

136

Overview

Sales tax

Discounts

Output tax Input tax

Irrecoverable sales tax

Accounting treatment

Page 137: f3 Course Notes

7: SALES TAX

137

1 Introduction 1.1 This chapter is designed to enable you to prepare basic accounting entries for sales tax,

known in many countries as Value Added Tax (VAT).

Sales tax 1.2 A business' sales and purchases are often subject to sales tax. This is an indirect tax, as it

is not levied directly on the individual like personal income tax. Sales tax is collected by traders who charge it on the goods they sell to the customer. A business charges sales tax on its sales (output tax) and suffers sales tax on its purchases (input tax). Typically, a business which is registered for sales tax only needs to make a payment to the tax authorities of the net amount of sales tax (i.e. sales tax owed on outputs less sales tax suffered on inputs).

1.3 A registered business shows: (a) items of income and expenditure net of sales tax; (b) trade receivables and trade payables gross of sales tax.

1.4 Illustration (all figures include sales tax at 15%). Purchase raw materials $115.00 Sell finished product $287.50 Required Calculate the amounts due to or from the sales tax authority. $ Input tax Output tax The rate of sales tax will always be provided in an exam question.

Purchases Goods into factory

(input tax)

Sales Goods out of factory

(output tax)

Page 138: f3 Course Notes

7: SALES TAX

138

2 Accounting treatment

Lecture example 1 A business buys goods for $1,000 plus 15% sales tax. They then sell those goods for $1,500 + 15% sales tax. The purchases will cost ($1,000 × 1.15) = $1,150 The sales will raise ($1,500 × 1.15) = $1,725 The sales tax payable to tax authorities will be: $ Payable on outputs (sales) (15% × $1,500) 225.00 Reclaimable on inputs (purchases) (15% × $1,000) (150.00) Net sales tax to tax authorities 75.00 As the business is purely collecting the sales tax for the tax authorities, and is able to set off its sales tax suffered it does not include sales tax as either an expense or income in the income statement. The sales tax is accounted for when the transaction occurs. Required

(a) Post the double entry to the ledger account below. $ $ Dr Purchases 1,000 Dr Sales tax control account 150 Cr Trade payables 1,150

Solution (a) Purchases (I/S) Trade payables (SOFP)

Sales tax control account (SOFP)

Page 139: f3 Course Notes

7: SALES TAX

139

Points to note Purchases – NET Trade payables – GROSS

(b) Post the double entry to the ledger account below. $ $ Dr Trade receivables 1,725 Cr Sales 1,500 Cr Sales tax control account 225

Solution Sales (I/S) Trade receivables (SOFP)

Sales tax control account (SOFP)

$ $ Balance b/d 150

Points to note Sales – NET Trade receivables – GROSS

3 Irrecoverable sales tax 3.1 In some tax regimes, sales tax on certain inputs is never recoverable. For example, sales

tax on business entertaining or on cars may not be recoverable. In this case the tax is a genuine expense of the business and is charged to the income statement or included in the cost of an asset to be depreciated. For example, the double entry for buying a car where the sales tax is irrecoverable would be:

Dr Motor vehicles account Cost + sales tax Cr Cash account Cost + sales tax

Page 140: f3 Course Notes

7: SALES TAX

140

4 Sales tax and discounts 4.1 Many businesses offer discounts to their customers. There are two types:

• trade discounts • settlement discounts

4.2 Sales tax is calculated on the amount after all discounts.

4.3 The calculation and accounting treatment of discounts is covered in Chapter 14.

Page 141: f3 Course Notes

7: SALES TAX

141

Additional Notes

Page 142: f3 Course Notes

7: SALES TAX

142

5 Rates of sales tax 5.1 Most goods and services are subject to sales tax at the standard rate. This may be 17½%

as it is in the UK or any other rate provided by the question.

5.2 Some goods are known as zero rated or exempt goods.

5.3 If goods are zero rated, such as books and newspapers, then sales tax is still charged on them but it is charged at a rate of 0%. A business making zero rated supplies can still claim back any input sales tax suffered on its purchases (at the relevant rate).

5.4 Exempt goods, such as banking, however are not subject to sales tax and a business making only exempt supplies cannot be registered for sales tax and cannot therefore reclaim input sales tax suffered on its purchases.

6 Chapter summary 6.1

Section Topic Summary

1 Introduction A business acts as a collecting agent for the tax authorities and charges sales tax (output tax) on its sales and reclaims sales tax (input tax) on its purchases.

2 Accounting treatment Sales and purchases are recorded at the net amount. Sales tax may be charged at various rates, however the rate of sales tax will always be provided in an exam question.

3 Irrecoverable sales tax Sales tax may not be recoverable on certain purchases. Where this is the case the question will state that the sales tax is not recoverable and the cost recorded will be the gross amount.

4 Sales tax and discounts

The effect of discounts on sales tax is covered in Chapter 14.

5 Rates of sales tax Zero rated supplies have sales tax charged on them at 0% whereas exempt supplies are not subject to sales tax.

Quick Quiz

Page 143: f3 Course Notes

7: SALES TAX

143

7 Double Entry Summary for Chapter 7 7.1 Recording a credit purchase with sales tax:

7.2 Recording a credit sale with sales tax:

Dr Purchases net Dr Sales tax control account tax Cr Trade payables gross

Dr Trade receivables gross Cr Sales net Cr Sales tax control account tax

Page 144: f3 Course Notes

7: SALES TAX

144

Chapter Summary

Sales tax

Discounts

Output tax Input tax

Irrecoverable sales tax

Accounting treatment

• A business charges sales tax on its sales • This is paid over to the tax authority once any

recoverable input tax is deducted

• A business will recover sales tax on its purchases

• Sales and purchases are recorded net of sales tax • Trade receivables and trade payables are recorded gross of sales tax

• Sales tax cannot be recovered on certain items such as some non-current assets

• The question will always state if this is the case

• Covered in Chapter 14

Page 145: f3 Course Notes

145

Chapter 7: Questions

Page 146: f3 Course Notes

7: QUESTIONS

146

7.1 Elmo is a trader registered for sales tax. All his sales and purchases carry sales tax at a rate of 15%. A customer has just returned goods sold for $230 plus sales tax, the double entry for this transaction is

A Debit payables $264.50, Credit sales tax $34.50, Credit sales $230

B Debit sales $264.50, Credit trade receivables $264.50

C Debit sales $230, Debit sales tax $34.50, Credit trade receivables $264.50

D Debit sales $230, Debit irrecoverable sales tax $34.50, Credit trade receivables $264.50 (2 marks)

7.2 During 20X1 Fergus buys two vans and a car each costing $10,000 plus sales tax at 15%. The car will be used 70% for business use and 30% personal use. He depreciates vehicles on a straight line basis, vans over five years and cars over six years. What is his depreciation expense to the nearest $ for the year?

In the tax regime in which Fergus operates sales tax is only recoverable on items used wholly for business purposes.

A $5,917

B $6,517

C $6,100

D $5,666 (2 marks)

Page 147: f3 Course Notes

147

Chapter 7: Answers

Page 148: f3 Course Notes

7: ANSWERS

148

7.1 C

7.2 A Sales tax on the car is not recoverable as it is not wholly used for business purposes. Sales tax is however recoverable on the vans.

$ Vans (2 × $10,000) ÷ 5 = 4,000

Car ($10,000 × 115%) ÷ 6 = 1,917 5,917

END OF CHAPTER

Page 149: f3 Course Notes

149

8

Syllabus Guide Detailed Outcomes Having studied this chapter you will be able to:

• Recognise the need for adjustments for inventory in preparing financial statements.

• Record opening and closing inventory.

• Identify the alternative methods of valuing inventory.

• Understand and apply the IASB requirements for valuing inventories.

• Recognise which costs should be included in valuing inventories.

• Calculate the value of closing inventory using 'first in, first out' and 'average cost'.

• Understand the use of continuous and period end inventory records.

• Understand the impact of accounting concepts on the valuation of inventory.

• Identify the impact of inventory valuation methods on profit and on assets.

Exam Context Accounting for inventories and inventory valuation is a basic principle that affects any business. Examination questions are likely to test your understanding of the terms cost and net realisable value. You should also expect calculations on this area and be able to make adjustments for both opening and closing inventory.

Qualification Context The Fundamentals level paper Management Accounting (F2) explores inventories in more detail. There you will look at the classification of costs (for example, production versus non production and fixed versus variable) and you will also cover detailed calculations on overhead absorption.

Inventory

Page 150: f3 Course Notes

8: INVENTORY

150

Overview

Cost

Inventory

Accounting adjustments

Valuation

FIFO AVCO

Methods of estimating cost

Net realisable value

Effects on profit

Page 151: f3 Course Notes

8: INVENTORY

151

1 Introduction 1.1 For some businesses, for example manufacturing entities, inventory can be a significant

figure.

1.2 It impacts the financial statement in two ways: (a) Statement of financial position: a potentially large balance within Current Assets (b) Income statement: opening and closing inventory have a direct

impact on cost of sales and therefore profits

1.3 Businesses must therefore ensure that their financial statements account for inventory accurately in terms of: (a) the accounting adjustment (b) its valuation

2 Accounting adjustment 2.1 Inventory is generally accounted for as a year end adjustment via a journal entry.

2.2 Opening inventory The trial balance produced by the entity at the end of the year will show an inventory figure. This amount generally relates to the opening inventory – i.e. the goods held by the business at the beginning of the year. Such goods will have been sold during the year. They are no longer an asset of the entity but will form part of the costs that should be matched against sales revenue when determining profit. The accounting entry is: Dr Cost of sales (I/S) Cr Inventories (SOFP)

2.3 Closing inventory The goods held by the business at the end of the year must be included as an asset in the statement of financial position and within cost of sales in the income statement. The accounting entry is: Dr Inventories (SOFP) Cr Cost of sales (I/S)

Page 152: f3 Course Notes

8: INVENTORY

152

2.4 The inventories figure comprises two elements: QUANTITY × VALUATION

Quantity: normally ascertained by inventory count at end of accounting period or

by continuous inventory records. Valuation: much more subjective, so guidance is provided in IAS 2.

2.5 Inventory overview Inventory = Quantity x Valuation Continuous Inventory Lower of Inventory records count Cost and NRV

$ Selling price X Less: completion costs (X) Less: selling costs (X) X

Actual cost Deemed cost FIFO Average Cost

3 Valuation 3.1 The basic rule per IAS 2: Inventories is:

'Inventories should be measured at the lower of cost and net realisable value.'

3.2 This is another example of prudence in presenting financial information. (a) If inventory is expected to be sold at a profit:

(i) value at cost (ii) do not anticipate profit.

(b) If inventory is expected to be sold at a loss: (i) value at net realisable value (ii) do provide for the future loss.

All costs to get item to current location in current condition

Page 153: f3 Course Notes

8: INVENTORY

153

4 Cost 4.1 The cost of an item of inventory includes:

For example: • purchase price • import duties Cost of purchase But not: • sales tax • trade discounts Relating to productions: Costs of conversion • direct labour • direct/variable overheads • an allocation of fixed

overheads (based on normal level of activity)

Other costs incurred in bringing the inventories to their present For example: location and condition • carriage inwards

Lecture example 1 Exam standard question worth 2 marks According to IAS 2: Inventories, which of the following should not be included in determining the cost of the inventories of an entity? (1) Labour costs (2) Transport costs to deliver goods to customers (3) Administrative overheads (4) Depreciation on factory machine A All four items B 1 only C 2 and 3 only D 2, 3, and 4 only

Solution

Section 5.4

Page 154: f3 Course Notes

8: INVENTORY

154

5 Net realisable value (NRV) 5.1 The net realisable value of an item is essentially its net selling proceeds after all costs have

been deducted.

5.2 It is calculated as: $ Estimated selling price X Less: estimated costs of completion (X) Less: estimated selling and distribution costs (X) X

Lecture example 2 Preparation question Jessie is trying to value her inventory. She has the following information available: $ Selling price 35 Costs incurred to date 20 Cost of work to complete item 12 Selling costs per item 1 Required

What is the net realisable value of Jessie's inventory? $

Workings

No netting off 5.3 The IAS 2 rule 'lower of cost and net realisable value' should be applied as far as

possible on an item by item (or line by line) basis.

Page 155: f3 Course Notes

8: INVENTORY

155

Illustration 5.4 Suppose an entity has four items of inventories on hand at the year end. Their costs and

NRVs are as follows: Inventory item

Cost

NRV Lower of cost

and NRV $ $ $ 1 27 32 27 2 14 8 8 3 43 55 43 4 29 40 29 113 135 107 It would be incorrect to compare total cost of $113 with total NRV of $135 and state inventories as $113. A loss on item 2 of $6 can be foreseen and should therefore be recognised. The comparison should be made for each item of inventory and thus a value of $107 would be attributed to inventories. This would be accounted for by the journal entry: $ $ Dr Inventories (SOFP) 107 Cr Cost of sales (I/S) 107

6 Theoretical methods of estimating cost

Issue 6.1 If various batches of inventories have been purchased at different times during the year and

at different prices, it may be impossible to determine precisely which items are still held at the year end and therefore what the actual purchase cost of the goods was. IAS 2 therefore allows an entity to approximate the cost of its inventories. There are two methods examinable at Paper F3: • First in, first out (FIFO) • Average cost

6.2 (a) FIFO Under FIFO it is assumed that:

(i) first goods purchased/produced will be the first to be sold (ii) remaining inventories are the most recent purchases/production.

(b) Average Cost (AVCO) There are two average costs available: (i) Simple average cost

The cost of all purchases/production during the year is divided by the total number of units purchased

Section 4.3

Page 156: f3 Course Notes

8: INVENTORY

156

(ii) Weighted average cost The weighted average of the cost of similar items is recalculated each time a new item is purchased/produced during the period (IAS 2 requires the weighted average to be used)

Lecture example 3 Preparation question On 1 January 20X7 a company held 200 units of finished goods valued at $10 each. During January the following transactions took place.

Date Units purchased Cost per unit 10 January 300 $10.85 20 January 350 $11.50 25 January 250 $13.00

Sales during January were as follows:

Date Units sold Sales price per unit 14 January 280 $18.00 21 January 400 $18.00 28 January 80 $18.00

Required Determine the valuation of closing inventories and cost of sales using: (a) FIFO (b) Weighted average cost

Solution (a) Closing inventories (FIFO)

Purchases 1.1.X7 10.1.X7 20.1.X7 25.1.X7 Sales Cost of sales (FIFO)

Page 157: f3 Course Notes

8: INVENTORY

157

(b) Closing inventories and cost of sales (AVCO) Average Total Cost of Units Cost Unit Cost Cost Sales $ $ $ $ 1.1.X7 b/f 10.1.X7 Purchase 14.1.X7 Sale 20.1.X7 Purchase 21.1.X7 Sale 25.1.X7 Purchase 28.1.X7 Sale Workings

Advantages and disadvantages 6.3 FIFO: more “realistic” value on statement of financial position.

Average cost: can be complex as weighted average is required by IAS 2.

Page 158: f3 Course Notes

8: INVENTORY

158

7 Valuation effects on profit 7.1 All of the inventory valuation methods affect profits. Using the FIFO, and average cost

examples above, this can be illustrated in an income statement. FIFO Weighted average $ $ $ $ Sales (760 × $18) 13,680 13,680 Cost of sales

Opening inventories 2,000 2,000 Purchases 10,530 10,530 Closing inventories (4,285) (4,160)

8,245 8,370 Gross profit 5,435 5,310

7.2 The only figure that varies is the closing inventories, the result being quite different profit figures. This re-emphasises the significance of inventory valuation in the preparation of financial statements.

Effects in times of changing prices 7.3 In the above example, the purchase price of inventories was rising during the period. Notice

that when prices are rising: FIFO will tend to give higher inventory values and higher profits.

Page 159: f3 Course Notes

8: INVENTORY

159

8 Chapter summary Section Topic Summary

1 Introduction Inventories can be a significant figure in an entity’s accounts and will impact both the profit figure and the net asset position. It is important therefore that it is recorded correctly.

2 Accounting adjustment As seen in chapter 6 the income statement matches the sales revenue earned in a period with the cost of sales incurred to generate that revenue. There are therefore two inventory adjustments: the opening inventory adjustment and the closing inventory adjustment.

3 Valuation Inventories should be valued at the lower of cost and net realisable value.

4 Cost The cost of inventory includes the cost of purchase, costs of conversion and any other costs necessary to bring the inventory to its present location and condition.

5 Net realisable value (NRV)

Net realisable value is the estimated selling price less the costs to completion and any selling and distribution costs.

6 Theoretical methods of estimating cost

Methods available to estimate the cost of inventories are first in, first out (FIFO) and average cost. Under FIFO the inventories held at the year end are the most recent purchases but under average cost the cost of all inventories purchased during the year is weighted to produce an average figure.

7 Valuation effects on profit

In times of rising prices, using FIFO will mean the financial statements show higher inventory values and higher profits.

Page 160: f3 Course Notes

8: INVENTORY

160

Chapter Summary

Cost

Inventory

Accounting adjustments

Valuation

FIFO AVCO

Methods of estimating cost

• Opening inventory: Dr Cost of sales (I/S) Cr Inventories (SOFP) • Closing inventory: Dr Inventories (SOFP)

Cr Cost of sales (I/S)

'Inventories should be measured at the lower of cost and net realisable value' • This is on a line by line basis

Net realisable value

• Calculation: Selling price X Less: completion costs (X) Less: selling costs (X) X

Effects on profit

• Closing inventory valuation will differ depending on cost method used

• This has a direct impact on cost of sales and therefore on gross profit

• When prices are rising FIFO will give higher inventory values and therefore higher profits

'First in, first out' • The first goods

purchased will be the first sold

• Year-end inventories relate to the most recent purchases

'Average cost' • Simple average calculation: Total purchases cost ÷ total number of units purchased • Weighted average (required by IAS 2): A new average is recalculated each time inventories are

purchased

• Cost includes: - costs of purchase - costs of conversion - other costs

Page 161: f3 Course Notes

161

Chapter 8: Questions

Page 162: f3 Course Notes

8: QUESTIONS

162

8.1 An item of inventory could be sold for $100 after it has been modified at a cost of $21. The company incurs selling and distribution costs of 5% of selling price on each article sold. The cost is $45 per unit excluding carriage inwards of $2 and production overheads of $17 per unit.

Following the rules in IAS 2 at what valuation should this item be included in the inventories of the company? $ (2 marks)

8.2 Harrow Co sells one line of inventory. At the year end it has 200 units in inventory which originally cost $10 per unit and had incurred delivery costs of $120 in total. They expect these goods to sell for $13 per unit. Harrow Co incurs selling costs amounting to 10% of the selling price on all its sales.

In the statement of financial position these items should be valued at:

A $2,000

B $2,080

C $2,120

D $2,600 (2 marks)

8.3 Lamp makes the following purchases in the year.

Units $/unit Total ($) (i) 21.01.X9 100 12.00 1,200 (ii) 30.04.X9 300 12.50 3,750 (iii) 31.07.X9 40 12.80 512 (iv) 01.09.X9 60 13.00 780 (v) 11.11.X9 80 13.50 1,080 At the year end 200 units are in inventory but eight are damaged and are only worth $10 per unit. These are identified as having been part of the 11.11.X9 delivery. Lamp operates a FIFO system for valuing inventories.

The figure for inventories at 31 December 20X9 is:

A $2,524

B $2,594

C $2,622

D $2,700 (2 marks)

8.4 Inventories

At the year end, Biggs Co holds the following inventories: (1) 10 units of L in a completed state; each unit cost $160 to make and has a selling price of $200. (2) 45 units of M in a partly completed state. Costs to date have amounted to $240 per unit and

completion costs will amount to $90 per unit. Selling price per unit is $360. (3) 60 units of N purchased for $40 each. These sell at $56 each and would now cost $48 each if

additional units were bought. (4) 50 units of O costing $10 each. These cannot be sold unless they are modified at a cost of $2 per

unit. After that, the selling price will be $8. The company’s selling costs are 25% of the selling price. Required

Calculate the value of inventories that would be shown on the statement of financial position at the end of the year.

Page 163: f3 Course Notes

8: QUESTIONS

163

8.5 T Bag

T Bag commenced business as a tea importer on 1 January 20X5. His purchases and sales during his first six months of trading are set out below:

Purchases Sales Tonnes Price per tonne Total price Tonnes Proceeds $ $ $ 1 January 30 700 21,000 15 February 20 750 15,000 27 February 40 36,000 31 March 40 820 32,800 16 April 25 880 22,000 30 April 35 35,000 30 May 35 900 31,500 8 June 10 1,050 10,500 28 June 70 77,000 132,800 148,000 Required

Calculate the value of closing inventories and produce a trading account for the 6 months ended 30 June 20X5 assuming:

(a) Inventories are valued on a FIFO basis (b) Inventories are valued on a weighted average basis

Page 164: f3 Course Notes

8: QUESTIONS

164

Page 165: f3 Course Notes

165

Chapter 8: Answers

Page 166: f3 Course Notes

8: ANSWERS

166

8.1 $64 NRV = 100 – 21 – (5% × 100) = $74 Cost = 45 + 2 + 17 = $64 Lower of cost and NRV = $64

8.2 C $ 200 @ $10 = 2,000 Delivery costs 120 2,120 Cost/ unit = $10.60

Net realisable value per unit = $13 × 90% = $11.70

⇒ valued at cost

8.3 B $ Inventories = 8 @ $10 = 80 72 @ $13.50 = 972 60 @ $13 = 780 40 @ $12.80 = 512 20 @ $12.50 = 250 200 2,594 8.4 Inventories

The inventories total on the statement of financial positionwould be: $12,200 ($1,500 + $8,100 + $2,400 + $200). Workings

(1) 1 unit of L would be valued at: $ Selling price 200 Less selling costs (25%) 50 ∴ NRV 150 Cost 160 NRV is lower and so 10 units of L are valued at $1,500 (2) 1 unit of M would be valued at: $ $ Selling price 360 Less: Selling costs (25%) 90 Costs to completion 90 180 ∴ NRV 180 Cost 240 NRV is lower and so 45 units of M are valued at $8,100 (3) 1 unit of N would be valued at: $ Selling price 56 Less selling costs (25%) 14 ∴ NRV 42 Cost 40 Cost is lower and 60 units of N are valued at $2,400 Replacement cost is irrelevant.

Page 167: f3 Course Notes

8: ANSWERS

167

(4) 1 unit of O would be valued at: $ $ Selling price 8 Less: Selling costs (25%) 2 Costs of modification 2 4

∴ NRV 4 Cost 10 NRV is lower and so 50 units of O are valued at $200

8.5 T Bag

Trading account for the 6 months ended 30 June 20X5

FIFO Weighted average

$ $ Sales 148,000 148,000 Cost of sales Purchases 132,800 132,800 Closing inventories (W) (15,000) (13,245) (117,800) (119,555) Gross profit 30,200 28,445

Workings

(W1) FIFO method 1 Jan 15 Feb 31 Mar 16 Apr 30 May 8 June Purchases in tonnes 30 20 40 25 35 10 Sales in tonnes: 27 Feb (30) (10) 30 Apr (10) (25) 28 June (15) (25) (30) Inventories at 30 June 20X5 – – – – 5 10 Cost per tonne $900 $1,050 ∴ Valuation $4,500 $10,500 Total valuation $4,500 + $10,500 = $15,000

(W2) Weighted average method Average Total Cost of Sales Tonnes Cost Unit Cost Cost $ $ $ $ 1 Jan 30 700 21,000 15 Feb 20 750 15,000 50 720 36,000 27 Feb (40) (28,800) 28,800 10 7,200 31 March 40 820 32,800 16 April 25 880 22,000 75 827 62,000 30 April (35) (28,945) 28,945 40 33,055 30 May 35 900 31,500 8 June 10 1,050 10,500 85 883 75,055 28 June (70) (61,810) 61,810 15 13,245 119,555

Page 168: f3 Course Notes

8: ANSWERS

168

END OF CHAPTER

Page 169: f3 Course Notes

169

9

Syllabus Guide Detailed Outcomes Having studied this chapter you will be able to:

• Define non-current assets and recognise the difference between current and non-current assets.

• Explain the difference between capital and revenue items and classify expenditure accordingly.

• Prepare ledger entries to record the acquisition, disposal, depreciation and accumulated depreciation of non-current assets.

• Calculate and record profits or losses on disposal of non-current assets in the income statement.

• Record the revaluation of a non-current asset and calculate its subsequent depreciation and profit or loss on disposal.

• Illustrate how non-current asset balances and movements are disclosed in company financial statements.

• Explain the purpose and function of an asset register.

• Understand and explain the purpose of depreciation.

• Calculate the charge for depreciation using the straight line and reducing methods, identifying when each is appropriate.

• Calculate the adjustments to depreciation necessary if changes are made in the estimated useful life and/or residual value of a non-current asset.

• Record depreciation in the income statement and statement of financial position.

Exam Context Tangible non-current assets and depreciation are an important part of the F3 syllabus and you should expect several questions on this area. Questions are likely to focus on areas such as calculating depreciation and asset values (both on assets held at historic cost and revalued amounts), profits or losses on disposal of assets and the components that can be included in the cost of a non-current asset.

Qualification Context The knowledge covered in this chapter is developed in the Fundamentals level paper Financial Reporting (F7) where you will deal with more complex issues such as impairments of non-current assets and leasing.

Tangible non-current assets

Page 170: f3 Course Notes

9: TANGIBLE NON-CURRENT ASSETS

170

Overview

Tangible non-current assets

Disposals Revaluations Depreciation

Straight line method

Reducing balance depreciation

Capital versus revenue expenditure Cost

Page 171: f3 Course Notes

9: TANGIBLE NON-CURRENT ASSETS

171

1 Introduction 1.1 The purchase of a non-current asset is often a significant cost to a business which will have

a large impact on its financial statements.

1.2 It is important therefore that this expenditure is accounted for appropriately.

2 Non-current assets

Definition 2.1 Non-current assets are assets which are intended to be used by the business on a

continuing basis and include both tangible and intangible assets. Intangible non-current assets are covered in Chapter 10.

2.2 The accounting treatment of tangible non-current assets is covered by IAS 16: Property, Plant and equipment. Tangible non-current assets are defined as those which: (a) are held for use in the production or supply of goods or services or for administrative

purposes; and (b) are expected to be used during more than one period.

Lecture example 1 Idea generation Required

What examples of tangible non-current assets can you identify?

Solution (a) (b) (c) (d)

Page 172: f3 Course Notes

9: TANGIBLE NON-CURRENT ASSETS

172

Capital versus revenue expenditure 2.3 (a) Capital expenditure: results in the acquisition, replacement or improvement

of non-current assets. (b) Revenue expenditure: – for the trade of the business, or – to repair, maintain and service non-current assets.

2.4 Capital expenditure results in the appearance of a non-current asset in the statement of financial position of the business. Revenue expenditure results in an expense in the income statement.

Cost 2.5 Tangible non-current assets should initially be recorded at cost.

Cost includes: • Purchase price: excluding sales tax and trade discounts but including import

duties • Directly attributable costs to bring the asset to its intended location and ready to

use. These include: (a) Initial delivery and handling costs (b) Installation and assembly costs (c) Costs of testing whether the asset is working properly (d) Professional fees The following costs may not be included: (a) The cost of maintenance contracts (b) Administration and general overhead costs (c) Staff training costs

2.6 The asset can then be kept at cost and depreciated or the entity may choose to revalue its tangible non-current assets.

Lecture example 2 Exam standard worth 2 marks On 10 December 20X7 an entity bought a machine. The breakdown on the invoice showed:

$ Cost of machine 20,000 Delivery costs 200 One-year maintenance contract 900 21,100 Further installation costs of $500 were also incurred.

Section 1.3

Page 173: f3 Course Notes

9: TANGIBLE NON-CURRENT ASSETS

173

Required At what amount should the machine be capitalised in the entity's records? A $20,000 B $20,700 C $20,200 D $21,600

Solution

3 Depreciation 3.1 Tangible non-current assets are used in the business to generate the income shown in the

income statement. Assets will eventually be worn out (used up) and so there is a cost of generating income. This cost should be shown in the income statement to 'match' against the income. This is called depreciation.

3.2 Depreciation results in the non-current asset being systematically charged to the income statement over several accounting periods in recognition of the fact that the asset will contribute to the income-generating activities of each of these periods. A formal definition is given by the accounting standard, IAS 16:

"…the systematic allocation of the depreciable amount of an asset over its useful life."

'Depreciable amount' = cost/revalued amount – residual value 'Residual value' = the amount the asset is expected to be sold for at the end of its

useful life (scrap value).

3.3 Land normally has an unlimited useful life and is therefore not depreciated. Buildings have a limited life and, therefore, are depreciable assets.

Page 174: f3 Course Notes

9: TANGIBLE NON-CURRENT ASSETS

174

4 Methods of depreciation 4.1 There are two main methods for calculating depreciation:

(a) Straight line method (b) Reducing balance method

5 Straight line method 5.1 The depreciation charge is the same every year.

Formula 5.2 Depreciation

(years) life usefulvalue residual cost −= or (Cost – Residual value) × %

where: Residual value = expected proceeds/scrap value at the end of the asset's useful life. Useful life = the number of years the business expects to make use of the asset.

5.3 This method is suitable for assets which are used up evenly over their useful life.

Lecture example 3 Preparation question A business buys a machine for $2,500. It is expected to have a useful life of three years after which time it will have a scrap value of $250. Required

(a) Calculate the annual depreciation charge. (b) Calculate the cost, accumulated depreciation and net book value (NBV) for each year of the

asset's life. Note: NBV = cost – accumulated depreciation to date.

Solution (a) (b)

Year Cost Accumulated depreciation

NBV

$ $ $ 1 2 3

Page 175: f3 Course Notes

9: TANGIBLE NON-CURRENT ASSETS

175

6 Reducing balance depreciation 6.1 This method is suitable for those assets which generate more revenue in earlier years than

in later years; for example a machine which may become progressively less efficient as it gets older.

Under this method the depreciation charge will be higher in the earlier years and reduce over time.

Formula 6.2 Depreciation = Depreciation rate (%) × Net Book Value (NBV)

where: net book value (NBV) = cost – accumulated depreciation to date Note: This method does not take account of any residual value, since the NBV under this

method will never reach zero. The depreciation rate percentage will be provided in the question.

Lecture example 4 Preparation question A business buys a machine costing $6,000. The depreciation rate is 40% on a reducing balance basis. Required

Calculate depreciation expense, accumulated depreciation and net book value of the asset for the first three years.

Solution Year NBV b/d Depreciation

rate Depreciation

expense Accumulated depreciation

NBV c/d

$ $ $ $ 1 2 3

Page 176: f3 Course Notes

9: TANGIBLE NON-CURRENT ASSETS

176

7 Accounting for depreciation

Dual effect 7.1 Depreciation has a dual effect which needs to be accounted for:

(a) It reduces the value of the asset in the statement of financial position. (b) It is an expense in the income statement.

7.2 The asset remains at its original cost in the asset account. Two accounts are set up to record depreciation: Dr Depreciation expense Cr Accumulated depreciation

Accumulated depreciation account 7.3 (a) Used to provide for the reduction in value of the asset.

(b) Reduces original cost of the asset on the statement of financial position. (The balance on the account is offset against the cost account for the corresponding asset.)

(c) Separate account kept for each class of asset (eg motor vehicles, buildings, plant and machinery).

Lecture example 5 Preparation question Required

Using the information in Lecture example 3, show: (a) The journal entry which would have been written at the end of the first year. (b) The treatment of depreciation for all years in the relevant ledger accounts. (c) The relevant income statement and statement of financial position extracts for each year.

Solution (a) Journal entry

Debit Credit $ $

Page 177: f3 Course Notes

9: TANGIBLE NON-CURRENT ASSETS

177

(b) Machine (SOFP)

Depreciation expense (I/S)

Accumulated depreciation (SOFP)

(c) Income statement (extracts) Year 1 Year 2 Year 3 $ $ $ Expenses

Page 178: f3 Course Notes

9: TANGIBLE NON-CURRENT ASSETS

178

Statement of financial position (extracts) Cost Accumulated Net book depreciation value $ $ $ (Year 1) (Year 2) (Year 3)

8 Disposal of non-current assets

Profit or loss on disposal 8.1 When a non-current asset is disposed, of its net book value needs to be removed from the

statement of financial position. The sales proceeds received are unlikely to be exactly the same as the asset's net book value and so a profit or loss on disposal will arise. If:

Sales proceeds > NBV ⇒ profit on disposal Sales proceeds < NBV ⇒ loss on disposal This is not a 'true' profit or loss, but rather a book adjustment to reflect the fact that the depreciation charged over the asset's life wasn't completely accurate.

Accounting treatment 8.2 Everything to do with the disposal is transferred to a Disposal Account.

Steps: (1) Remove the cost of the asset:

Dr Disposal account Cr Non-current asset

(2) Remove the accumulated depreciation charged to date: Dr Accumulated depreciation Cr Disposal account

Note: Steps (1) and (2) have effectively transferred the NBV of the asset to the disposal account.

Page 179: f3 Course Notes

9: TANGIBLE NON-CURRENT ASSETS

179

(3) Account for the sales proceeds: Dr Cash Cr Disposal account

(4) Balance off disposal account to find the profit or loss on disposal. A gain on disposal is shown in the income statement as sundry income, a loss as an expense.

Lecture example 6 Preparation question The machine costing $6,000 in Lecture example 4 is sold in year 3 for $3,000. No depreciation is charged in the year of disposal. Required

(a) Calculate the profit or loss on disposal of the machine. (b) Complete the ledger accounts to show how the disposal would be accounted for.

Solution (a) (b)

Machine (SOFP) $ $ Bal b/d 6,000

Page 180: f3 Course Notes

9: TANGIBLE NON-CURRENT ASSETS

180

Accumulated depreciation (SOFP) $ $ Bal b/d 3,840

Disposal account (I/S) $ $

Part exchange allowance 8.3 Instead of receiving sales proceeds as cash, a part exchange allowance could be offered

against the cost of a replacement asset: Dr New asset cost Cr Disposal account The part exchange allowance takes the place of proceeds in the disposals account.

Lecture example 7 Preparation question Assume in Lecture example 6 that instead of cash proceeds of $3,000, there is a part exchange allowance of $3,000 on a replacement machine costing $10,000. Required

(a) Calculate the profit or loss on disposal of the machine. (b) Calculate the amount of cash paid for the new machine. (c) Complete the ledger accounts to show both the disposal and the acquisition.

Page 181: f3 Course Notes

9: TANGIBLE NON-CURRENT ASSETS

181

Solution (a) (b) (c)

Old machine (SOFP) $ $ Bal b/d 6,000

Accumulated depreciation (SOFP) $ $ Bal b/d 3,840

New machine (SOFP) $ $

Disposal account (I/S) $ $

Page 182: f3 Course Notes

9: TANGIBLE NON-CURRENT ASSETS

182

9 Revaluations 9.1 If an entity owns a property it may notice that its value increases over time.

9.2 IAS 16 requires tangible non-current assets to initially be recorded at cost. The entity can then either keep the asset at cost (and depreciate it) or choose to revalue it (depreciation is still required). This is a choice of accounting policy.

9.3 If an entity chooses a policy of revaluation then all items in the same class of assets must be revalued. Examples of classes of assets are: • land and buildings • plant and machinery • motor vehicles

9.4 Revaluations must be carried out sufficiently often so that the assets carrying value is not materially different from its market value.

Steps and accounting treatment 9.5 (1) Adjust cost account to revalued amount.

(2) Remove accumulated depreciation charged on the asset to date. (3) Put the balance to the revaluation reserve. Note: The balance posted to the revaluation reserve will equal the new revalued amount

less the previous net book value.

9.6 The required journal is: Dr Non-current asset cost Dr Accumulated depreciation Cr Revaluation reserve

9.7 Depreciation should now be based on the revalued amount.

Page 183: f3 Course Notes

9: TANGIBLE NON-CURRENT ASSETS

183

Lecture example 8 Preparation question A building costing $100,000 on which depreciation of $20,000 has been charged is to be revalued to $150,000. Required

(a) Show the double entry to record the revaluation and make the postings to the ledger accounts.

(b) What would be the depreciation charge for the year if the building has a remaining useful life of 40 years?

Solution (a)

Building (SOFP) $ $

Accumulated depreciation (SOFP) $ $

Page 184: f3 Course Notes

9: TANGIBLE NON-CURRENT ASSETS

184

Revaluation reserve (SOFP) $ $

(b)

Page 185: f3 Course Notes

9: TANGIBLE NON-CURRENT ASSETS

185

Additional Notes

Page 186: f3 Course Notes

9: TANGIBLE NON-CURRENT ASSETS

186

10 Depreciation revisited 10.1 Depreciation is charged to allocate the wearing out of an asset (depreciable amount) to the

income statement over its useful life. There are two main depreciation methods available: • straight line • reducing balance

10.2 The useful life of an item of property, plant and equipment should be reviewed at least every financial year-end and, if expectations are significantly different from previous estimates, the depreciation charge for current and future periods should be revised. This is achieved by writing the net book value off over the asset's revised remaining useful life.

Lecture example 9 Preparation question 1.1.X1 Asset cost $40,000

Estimated useful life five years No residual value

1.1.X3 Total useful life revised to four years. Required

Calculate the depreciation charge, accumulated depreciation and NBV for each year of the asset's life (year end 31 December).

Solution Depreciation

charge Accumulated depreciation

NBV

$ $ $ 20X1 20X2 20X3 20X4

Section 3.12

Page 187: f3 Course Notes

9: TANGIBLE NON-CURRENT ASSETS

187

Review of depreciation method 10.3 The depreciation method should be reviewed at least every financial year-end and, if there

has been a significant change in the expected pattern of the asset's use, the method should be changed. This is achieved by writing the net book amount off over the remaining useful life, using the revised method.

Lecture example 10 Preparation question 1.1.X1 Asset cost $40,000

Residual value $1,500 Useful life five years Depreciation: 25% reducing balance

1.1.X3 Change depreciation method to straight line Required

Calculate the depreciation charge, accumulated depreciation and NBV for each year of the asset’s life (year ended 31 December).

Solution

Depreciation

charge Accumulated depreciation

NBV

$ $ $ 20X1 20X2 20X3 20X4 20X5

Section 3.7-3.9

Page 188: f3 Course Notes

9: TANGIBLE NON-CURRENT ASSETS

188

11 Chapter summary Section Topic Summary

2 Non-current assets Capital expenditure results in a non-current asset being shown on the statement of financial position. Revenue expenditure, such as repairs and maintenance, is shown as an expense in the income statement. Tangible non-current assets should initially be recorded at cost. This includes the purchase price of the item plus any directly attributable costs to bring the item to its intended location and ready to use.

3 Depreciation Depreciation is an expense charged in relation to the asset each year to reflect the using up of the asset. Land usually has an unlimited useful life and so is not depreciated.

4 Methods of depreciation

Depreciation is usually calculated on a straight line or reducing balance basis.

5 Straight line method This method is suitable for assets which are used up evenly during their life time. The depreciation expense is the same each year.

6 Reducing balance depreciation

This method is suitable for assets which generate more revenue in the earlier years of their life. The depreciation expense is higher in the initial years.

7 Accounting for depreciation

Depreciation is recorded by way of a journal entry. The expense is recorded as a debit entry and reduces profit. The credit is made to the accumulated depreciation account and reduces the carrying value of the asset in the statement of financial position.

8 Disposal of non-current assets

On disposal of a non-current asset the sales proceeds are compared to the net book value of the asset in order to calculate the profit or loss on disposal. Where an asset is given in part exchange for another asset, the part exchange allowance takes the place of the sales proceeds.

9 Revaluations An entity may choose to revalue its assets rather than hold them at cost – this is a choice of accounting policy. Where an entity revalues, it must revalue all assets in the same class and the depreciation charge is based on the revalued amount.

Page 189: f3 Course Notes

9: TANGIBLE NON-CURRENT ASSETS

189

Section Topic Summary

10 Depreciation revisited If an entity changes the method of depreciation used from straight line to reducing balance (or vice versa) or revises the useful life of an asset it should write off the asset’s net book value using the revised method or useful life.

12 Double Entry Summary for Chapter 9 12.1 Depreciation adjustment:

12.2 Disposal of a non-current asset (four steps): (1) Remove the cost of the asset:

(2) Remove the accumulated depreciation charged to date:

(3) Account for the sales proceeds:

(4) Balance off the disposal account to determine the profit or loss on disposal.

12.3 Revaluation of a non-current asset:

Dr Depreciation expense (I/S) Cr Accumulated depreciation (SOFP)

Dr Disposal account (I/S) Cr Non-current assets (SOFP)

Dr Accumulated depreciation (SOFP) Cr Disposal account (I/S)

Dr Cash (SOFP) Cr Disposal account (I/S)

Dr Non-current asset cost (SOFP) Dr Accumulated depreciation (SOFP) Cr Revaluation reserve (SOFP)

Page 190: f3 Course Notes

9: TANGIBLE NON-CURRENT ASSETS

190

Chapter Summary

Tangible non-current assets

Disposals Revaluations Depreciation

Straight line method

Reducing balance depreciation

Capital versus revenue expenditure Cost

• Capital expenditure: acquisition, replacement or improvement of non-current assets

• Revenue expenditure: trading expenses or the repair, maintenance and service of non-current assets

• Cost includes the purchase price plus directly attributable costs

• Directly attributable costs include: - delivery - installation/ testing - professional fees • Directly attributable costs exclude: - maintenance contracts - administration and general overheads - staff training

• Steps: (1) Adjust cost to

revalued amount (2) Remove the

accumulated depreciation charged to date

(3) Put the balance to the revaluation reserve

• The balance transferred to the revaluation reserve is:

'revalued amount – net book value' • Revaluation is a choice

of accounting policy. All assets in the same class must be revalued

• Depreciation is now based on the revalued amount

'The wearing out of an asset as it generates revenue' • Accounting adjustment: Dr Depreciation expense Cr Accumulated depreciation

• Depreciation charge is the same each year

• Formula:

cost - residual value

useful life

or (cost – residual

value) × %

• Depreciation charge is higher in the earlier years of the asset's life

• Formula: Depreciation rate

(%) × net book value

• Steps: (1) Remove the cost of the asset (2) Remove the accumulated

depreciation charged to date (3) Account for sales proceeds (4) Balance off disposal account to

find the profit or loss on disposal • Profit/ loss on disposal calculation: Proceeds X Less: NBV (X) Profit/(loss) X/(X)

Page 191: f3 Course Notes

191

Chapter 9: Questions

Page 192: f3 Course Notes

9: QUESTIONS

192

Data for Questions 9.1 and 9.2 Bungo Co purchases a car for its managing director, which would cost $17,000, by paying $1,000 cash, and trading in an old vehicle. The old vehicle had a net book value of $15,500 immediately before the trade in took place.

9.1 What is the effect of the above transaction on the profit for the year in respect of the disposal of the old vehicle?

A Reduce profit by $1,500

B Increase profit by $1,500

C Reduce profit by $500

D Increase profit by $500 (2 marks)

9.2 Bungo Co charges depreciation at 10% per annum, with a full year’s charge in the year of acquisition.

What will the annual depreciation charge on the new vehicle be? $ (2 marks)

9.3 A company held property, plant and equipment at 31 December 20X5 with a net book value of $22,700. During 20X6 items with a net book value of $2,100 were sold, realising a profit of $700. The depreciation charge in the 20X6 income statement was $4,300. Items with a book value of $15,200 were revalued to $21,250. At 31 December 20X6 the company’s statement of financial position showed the net book value of property, plant and equipment as $44,100.

What was the cost of new property, plant and equipment acquired during 20X6?

A $13,150

B $17,550

C $22,050

D $21,750 (2 marks)

9.4 Nick

Nick started trading on 1 January 20X8 and bought equipment for his business as follows: 1 January 20X8 – Purchased a cutting machine for $4,960. The estimated useful life of the

machine is eight years, after which it will have no resale value. 2 January 20X8 – Purchased a car for $6,800. 1 March 20X8 – Purchased a van for $3,800. This has an estimated useful life of four years,

after which Nick believes he could sell it for $200. 1 May 20X8 – Purchased office furniture costing $5,400. This has an estimated useful life of

10 years with no resale value. Depreciation for all assets, except the car, is to be calculated on the straight line basis, time apportioned where the asset is owned for part of a year. The car is to be depreciated at 40% per annum on the reducing balance basis.

Required

For the years ending 31 December 20X8 and 31 December 20X9, prepare relevant extracts from the financial statements, together with the appropriate ledger accounts.

Page 193: f3 Course Notes

9: QUESTIONS

193

9.5 Eggo

On 1 January 20X4 Eggo Co, a manufacturer, acquired two identical grinding machines at a cost of $10,000 each, and a duplicating machine at a cost of $3,000. The grinding machines are depreciated at the rate of 30% per annum on a reducing balance basis, and the duplicating machine, which has an estimated life of 10 years and a residual value of $500, is depreciated on a straight line basis. On 1 January 20X5 one of the grinding machines was sold for $5,000 and replaced by a new one costing $12,000. Required

Prepare the relevant ledger accounts dealing with the non-current assets, depreciation and the disposal for the years to 31 December 20X4 and 31 December 20X5, respectively.

9.6 Hopkins

During 20X4 Hopkins gave his old van in part-exchange for a new van. The old van had cost $4,000 and had accumulated depreciation of $2,400 at the date of exchange. Hopkins received a part-exchange allowance of $1,800 and made a cash payment of $6,200 for the new van. Depreciation is over four years on a straight line basis.

Required

(a) Calculate the profit or loss on disposal of the old van. (b) Calculate the depreciation expense for the year ended 20X4.

Page 194: f3 Course Notes

9: QUESTIONS

194

Page 195: f3 Course Notes

195

Chapter 9: Answers

Page 196: f3 Course Notes

9: ANSWERS

196

9.1 D Profit on disposal = (17,000 – 1,000) – 15,500 = $500

9.2 $1,700

10% × $17,000 = $1,700

9.3 D

Property, plant and equipment (NBV) $ $ B/d 22,700 Disposals 2,100 Revaluation 6,050 Depreciation 4,300 (21,250-15,200) Additions ? C/d 44,100 50,500 50,500 ∴ additions = $21,750

9.4 Nick

Income statement for the year ended 31 December .... (extract) 20X8 20X9 Depreciation Expense $ $ Machine 620 620 Car 2,720 1,632 Van 750 900 Furniture 360 540 4,450 3,692 Statement of financial position as at 31 December 20X8 (extract) Non-current assets Cost Accumulated

depreciation Net Book

Value $ $ $ Machine 4,960 620 4,340 Car 6,800 2,720 4,080 Van 3,800 750 3,050 Furniture 5,400 360 5,040 20,960 4,450 16,510 Statement of financial position as at 31 December 20X9 (extract) Non-current assets Cost Accumulated

depreciation Net Book

Value $ $ $ Machine 4,960 1,240 3,720 Car 6,800 4,352 2,448 Van 3,800 1,650 2,150 Furniture 5,400 900 4,500 20,960 8,142 12,818

Machine (SOFP) $ 1.1.X8 Bank 4,960

Page 197: f3 Course Notes

9: ANSWERS

197

Machine – Accumulated Depreciation (SOFP) $ $ 31.12.X8 bal c/d 620 31.12.X8 Dep’n expense: machine 620 620 620 1.1.X9 bal b/d 620 31.12.X9 bal c/d 1,240 31.12.X9 Dep’n expense : machine 620 1,240 1,240 1.1.Y0 bal b/d 1,240

Car (SOFP) $ 2.1.X8 Bank 6,800

Car – Accumulated Depreciation (SOFP)

$ $ 31.12.X8 bal c/d 2,720 31.12.X8 Dep’n expense: car 2,720 2,720 2,720 1.1.X9 bal b/d 2,720 31.12.X9 bal c/d 4,352 31.12.X9 Dep'n expense: car 1,632 4,352 4,352 1.1.Y0 bal b/d 4,352

Van (SOFP) $ 1.3.X8 Bank 3,800

Van – Accumulated Depreciation (SOFP) $ $ 31.12.X8 bal c/d 750 31.12.X8 Dep’n expense: van 750 750 750 1.1.X9 bal b/d 750 31.12.X9 bal c/d 1,650 31.12.X9 Dep’n expense: van 900 1,650 1,650 1.1.Y0 bal b/d 1,650

Furniture (SOFP) $ 1.5.X8 Bank 5,400

Page 198: f3 Course Notes

9: ANSWERS

198

Furniture – Accumulated Depreciation (SOFP) $ $ 31.12.X8 bal c/d 360 31.12.X8 Dep’n expense: furniture furniture 360 360 360 1.1.X9 bal b/d 360 31.12.X9 bal c/d 900 31.12.X9 Dep’n expense: furniture 540 900 900 1.1.Y0 bal b/d 900

Depreciation Expense : Machine (I/S) $ $ 31.12.X8 Acc’d dep’n: machine 620 31.12.X8 I/S 620 31.12.X9 Acc’d dep’n: machine 620 31.12.X9 I/S 620

Depreciation Expense : Car (I/S) $ $ 31.12.X8 Acc’d dep’n: car 2,720 31.12.X8 I/S 2,720 31.12.X9 Acc’d dep’n: car 1,632 31.12.X9 I/S 1,632

Depreciation Expense : Van (I/S) $ $ 31.12.X8 Acc’d dep’n: van 750 31.12.X8 I/S 750 31.12.X9 Acc’d dep’n: van 900 31.12.X9 I/S 900

Depreciation Expense : Furniture (I/S) $ $ 31.12.X8 Acc’d dep’n: furniture 360 31.12.X8 I/S 360 31.12.X9 Acc’d dep’n: furniture 540 31.12.X9 I/S 540

Workings: Depreciation charge 20X8 20X9 $ $ Machine 4,960 ÷ 8 620 620

Car 6,800 × 40% 2,720 (6,800 – 2,720) × 40% 1,632 (note: reducing balance method)

Van (3,800 – 200) ÷ 4 = 900

900 × 1210 = 750

(10 months) 750 (full year) 900

Furniture (5,400) × 128 (8 months) 360 (full year) 540

Page 199: f3 Course Notes

9: ANSWERS

199

9.5 Eggo Grinding machines (SOFP)

$ $ 1.1.X4 Bank 20,000 31.12.X4 Balance c/d 20,000 1.1.X5 Balance b/d 20,000 1.1.X5 Disposals 10,000 1.1.X5 Bank 12,000 31.12.X5 Balance c/d 22,000 32,000 32,000 1.1.X6 Balance c/d 22,000

Grinding machines – Accumulated Depreciation (SOFP)

$ $ 31.12.X4 Balance c/d 6,000 31.12.X4 Dep'n expense (W) 6,000 1.1.X5 Disposals 3,000 1.1.X5 Balance b/d 6,000 31.12.X5 Balance c/d 8,700 31.12.X5 Dep'n expense (W) 5,700 11,700 11,700 1.1.X6 Balance b/d 8,700

Duplicating machine (SOFP)

$ $ 1.1.X4 Bank 3,000 31.12.X4 Balance c/d 3,000 1.1.X5 Balance b/d 3,000 31.12.X5 Balance c/d 3,000 1.1.X5 Balance b/d 3,000

Duplicating machine – Accumulated Deprecation (SOFP)

$ $ 31.12.X4 Balance c/d 250 31.12.X4 Depreciation expense 250 250 250 1.1.X5 Balance b/d 250 31.12.X5 Balance c/d 500 31.12.X5 Depreciation expense 250 500 500 1.1.X6 Balance b/d 500 Depreciation expense (I/S)

$ $ 31.12.X4 Acc dep'n – grinding machines 6,000 31.12.X4 Acc dep'n – duplicating machine 250 31.12.X4 I/S 6,250 6,250 6,250 31.12.X5 Acc dep'n – grinding machines 5,700 31.12.X5 Acc dep'n – duplicating machine 250 31.12.X5 I/S 5,950 5,950 5,950

Page 200: f3 Course Notes

9: ANSWERS

200

Disposal account (I/S)

$ $ 1.1.X5 Grinding machines 10,000 1.1.X5 Bank 5,000 1.1.X5 Acc Dep'n – grinding machine 3,000 31.12.X5 I/S – loss on disposal 2,000 10,000 10,000 Working

Depreciation charge Year ended 31 December 20X4 20X5 $ $ Grinding machines ($20,000 × 30%) 6,000 Duplicating machine ($3,000 – $500) ÷ 10 250 Grinding machines Machine 1 ($10,000 – $3,000) × 30% 2,100 Machine 2 ($12,000 × 30%) 3,600 Duplicating machine 250 6,250 5,950

9.6 Hopkins

(a) $200 profit

Working Disposal account (I/S)

$ $ Cost 4,000 Accumulated depreciation 2,400 Profit on disposal 200 Part exchange allowance 1,800 4,200 4,200 Or alternatively:

$ "Proceeds" – part-exchange allowance 1,800 Net book value ($4,000 – $2,400) (1,600) Profit on disposal 200

(b) $2,000

$ Cost of new van to be depreciated 8,000 ($6,200 + $1,800) Depreciate over four years 2,000

END OF CHAPTER

Page 201: f3 Course Notes

201

10

Syllabus Guide Detailed Outcomes Having studied this chapter you will be able to:

• Recognise the difference between tangible and intangible non-current assets.

• Identify types of intangible assets.

• Identify the definition and treatment of research and development costs in accordance with IFRS.

• Calculate amounts to be capitalised as development expenditure or to be expensed from given information.

• Calculate and account for the charge for amortisation and explain its purpose.

Exam Context Intangible non-current assets are a smaller part of the syllabus than tangible non-current assets; however you should still expect this area to be tested. Questions are likely to focus on the difference between tangible and intangible assets, the accounting treatment for research and the capitalisation criteria for development expenditure. You should also be confident in calculating amortisation.

Qualification Context The knowledge covered in this chapter forms a platform which will be built on in the Fundamentals level paper Financial Reporting (F7). There you will cover internally generated intangible assets and goodwill.

Intangible non-current assets

Page 202: f3 Course Notes

10: INTANGIBLE NON-CURRENT ASSETS

202

Overview

Accounting treatment Accounting treatment

Amortisation

Intangible non-current assets

Research Development expenditure

Page 203: f3 Course Notes

10: INTANGIBLE NON-CURRENT ASSETS

203

1 Definition 1.1 An intangible non-current asset is an identifiable non-monetary asset without physical

substance. 1.2 The following are examples of intangible assets:

– Development expenditure – Goodwill – Concessions, patents, licences, trade marks. The Paper F3 syllabus only requires knowledge of the accounting treatment of research and development expenditure.

2 Research and development expenditure 2.1 Many companies, such as pharmaceutical companies, spend huge amounts on research

and development every year in order to maintain or enhance their competitive position.

2.2 Companies need to account for these costs and whilst the credit entry will be recorded as a current liability, the question remains as to where the debit entry should be shown. The choices are: (a) to debit the income statement with an expense, or (b) to debit the statement of financial position with an intangible non-current asset. An intangible non-current asset should only be recorded when the entity is confident that the expenditure will generate future profit.

3 IAS 38: Intangible assets

Definitions 3.1 (a) Research is original and planned investigation undertaken with the prospect of

gaining new scientific or technical knowledge and understanding. (b) Development is the application of research findings or other knowledge to a plan or

design for the production of new or substantially improved materials, devices, products, processes, systems or services before the start of commercial production or use.

Page 204: f3 Course Notes

10: INTANGIBLE NON-CURRENT ASSETS

204

4 Accounting treatment 4.1

Research Development • No certainty that the expenditure

will generate future profit • Future profits are expected

• Show as an expense in income

statement • MUST capitalise as an intangible non-

current asset if all of the relevant criteria are satisfied

• Dr Research expense (I/S) Cr Bank/payables

• Dr Intangible non-current assets (SOFP) Cr Bank/payables

P robable future economic benefits I ntention to complete and use/sell asset R esources adequate and available to complete

and use/sell asset A bility to use/sell the asset T echnical feasibility of completing asset for

use/sale E xpenditure can be measured reliably • Amortise asset over its useful life once

asset is ready for use

Lecture example 1 Preparation question Z Co incurred the following costs during the year ended 31 August 20X8. (1) $20,000 on salaries for market research staff sent out to canvass drivers' opinions on a

potential new car. (2) $100,000 to purchase a machine to manufacture components for the new car. It has an

estimated useful life of 10 years. (3) $25,000 on materials to manufacture a prototype and $50,000 on salaries relating to its

design and manufacture. The new car is expected to go on sale in 20X9. Required How should each of the above items be shown in the financial statements for the year ended 31 August 20X8?

Page 205: f3 Course Notes

10: INTANGIBLE NON-CURRENT ASSETS

205

Solution

5 Amortisation of capitalised development expenditure 5.1 A tangible non-current asset, such as a machine, is capitalised and then depreciated over its

useful life. This is to allocate its costs over the accounting periods which benefit from its use.

5.2 In the same way development expenditure which is incurred now will generate revenue and profits in the future. The cost of the development expenditure should be matched against the revenue it produces. This is called amortisation.

5.3 The 'depreciable amount' (cost less residual value) should be amortised over the useful life in the same way that revenues are expected to be generated.

5.4 Amortisation should begin when the asset is ready for use.

Page 206: f3 Course Notes

10: INTANGIBLE NON-CURRENT ASSETS

206

5.5 It is an expense in the income statement and is accounted for using the following entry: Dr Amortisation expense (I/S) Cr Accumulated amortisation (SOFP)

Lecture example 2 Technique demonstration Development Co incurs the following expenditure in years 20X1 – 20X5. Research

$ Development

$ 20X1 35,000 55,000 20X2 – 65,000 20X3 – – 20X4 – – 20X5 38,000 – The development expenditure meets the IAS 38 criteria that require capitalisation ('PIRATE'). The item developed in 20X1 and 20X2 goes on sale on 1.1.X3 and it will be three years from then until any competitor is expected to have a similar product on the market. Required

Show income statement and statement of financial position extracts for the years 20X1 – 20X5 inclusive.

Solution

Income statement extracts X1 X2 X3 X4 X5 $ $ $ $ $ Expenses Research expenditure Amortisation of development expenditure Statement of financial position extracts X1 X2 X3 X4 X5 $ $ $ $ $ Non-current assets Development expenditure Amortisation Net book value

Page 207: f3 Course Notes

10: INTANGIBLE NON-CURRENT ASSETS

207

6 Chapter summary Section Topic Summary 1 Definition An intangible non-current asset is an identifiable non-

monetary asset without physical substance. 2 Research and

development expenditure

Some entities spend significant sums of money on research and development it is therefore essential that these transactions are accounted for appropriately.

3 Intangible assets (IAS 38)

IAS 38 defines research and development. Research expenditure is incurred where the entity is acquiring new scientific or technical knowledge. Development expenditure relates to the application of research findings.

4 Accounting treatment Research relates to costs incurred to obtain knowledge or understanding. There is no certainty of future profit from this expenditure and so it should be shown as an expense in the income statement. Development expenditure MUST be capitalised as an intangible non-current asset provided all of the PIRATE criteria are met. This asset will then be amortised over the period during which it is expected to generate income.

5 Amortisation of capitalised development expenditure

Amortisation is essentially the same as depreciation but relates to intangibles. Where an entity has capitalised development expenditure it should amortise the intangible once the asset is ready for use.

Quick Quiz

Page 208: f3 Course Notes

10: INTANGIBLE NON-CURRENT ASSETS

208

Chapter Summary

Accounting treatment Accounting treatment

Amortisation

Intangible non-current assets

Research Development expenditure

'Investigation to gain new scientific or technical knowledge and understanding'

'Application of research findings or other knowledge to produce new/substantially improved materials, processes etc'

• There is no certainty of future profits • Write-off as an expense in the income

statement

• Future profits are expected • Capitalise as an intangible non-current asset if all PIRATE

criteria are satisfied • PIRATE: Probable future economic benefits Intention to complete and use/sell asset Resources adequate and available to use/sell asset Ability to use/sell asset Technical feasibility of completing asset for use/sale Expenditure can be measured reliably

• Amortise asset over its useful life once asset is ready for use

Page 209: f3 Course Notes

209

Chapter 10: Question

Page 210: f3 Course Notes

10: QUESTION

210

10.1 Which of the following statements about research and development are true?

(1) Development expenditure shown on the statement of financial position should be amortised over the periods expected to benefit from the product or service.

(2) Development expenditure must be capitalised if it meets various criteria.

(3) Research expenditure is always written off.

A All of the above

B (1) and (2)

C (2) and (3)

D (1) and (3) (2 marks)

Page 211: f3 Course Notes

211

Chapter 10: Answer

Page 212: f3 Course Notes

10: ANSWER

212

10.1 A

END OF CHAPTER

Page 213: f3 Course Notes

213

11

Syllabus Guide Detailed Outcomes Having studied this chapter you will be able to:

• Understand how the matching concept applies to accruals and prepayments.

• Identify and calculate the adjustments needed for accruals and prepayments in preparing financial statements.

• Prepare the journal entries and ledger entries for the creation of an accrual or prepayment.

• Understand and identify the impact on profit and net assets of accruals and prepayments.

Exam Context Accruals and prepayments are key accounting adjustments and you should expect to see them tested in Paper F3. You may be asked to calculate the statement of financial position amount for accruals and prepayments and/or the relevant expense that would be shown in the income statement. Alternatively, you may be asked to determine the appropriate journal entries to record accruals and prepayments. The Pilot Paper included questions on the calculation of a year-end prepayment of an expense and the income to be shown in the income statement where rent is received both in advance and in arrears.

Qualification Context This area is a basic skill which is not tested in detail in any other paper. The matching concept however is fundamental to the preparation of financial statements and this is relevant to Paper F7, Financial Reporting.

Accruals and prepayments

Page 214: f3 Course Notes

11: ACCRUALS AND PREPAYMENTS

214

Overview

Accruals and prepayments

Accounting treatment

Accounting treatment

Reversing out accruals and prepayments

Year end adjustments Presentation in the statement of financial position

Accrued income and deferred income

Page 215: f3 Course Notes

11: ACCRUALS AND PREPAYMENTS

215

1 Introduction 1.1 This chapter is designed to enable you to apply accounting concepts and principles in

relation to the calculation of and adjustments for accruals and prepayments.

1.2 IAS 1 requires financial statements to be prepared on an accruals basis. This is so that transactions and events are recognised when they occur (and not as cash or its equivalent is received or paid) and they are recorded in the accounting records and reported in the financial statements of the period to which they relate. The accruals basis is also an underlying assumption in the IASB's Framework for the Preparation and Presentation of Financial Statements.

Accruals 1.3 Accruals are expenses incurred by the business during the accounting period but not yet

paid for, i.e. expenses in arrears.

Example 1.4 Fred prepares accounts to 31 December each year. On 1 January 20X8, he pays a

telephone bill of $60 which relates to the period October-December 20X7. Although the payment does not go through the cash book until 20X8, this expense must be included in the accounts for the year ended 31 December 20X7, as it was incurred during this period.

Prepayments 1.5 Prepayments arise when expenses are paid for before they have been used, i.e. expenses

in advance.

Example 1.6 On 20 December 20X7 Fred pays for insurance on his business premises for the 12 months

commencing 1 January 20X8. Although the payment was made in 20X7, the expense should not appear in the accounts for 20X7. The accounts for 20X7 will show a prepayment for the full amount of the insurance cost and the expense will be recorded in 20X8.

Page 216: f3 Course Notes

11: ACCRUALS AND PREPAYMENTS

216

2 Accounting treatment

Year-end adjustments 2.1 Adjustments for accruals and prepayments tend to occur at the end of the year and are

made by way of a journal entry. The required entries are:

Accruals Dr Expense (I/S) Cr Accruals (SOFP)

Prepayments Dr Prepayments (SOFP) Cr Expense (I/S)

Presentation in the statement of financial position 2.2 Accruals:

Sub-heading under 'current liabilities' Prepayments: Sub-heading under 'current assets'.

Lecture example 1 Preparation question Fiona set up a business on 1 January 20X7. Her cash payments for the year to 31 December 20X7 included:

Date paid Amount$

Period

Electricity 10.3.X7 96 2 months to 28 February 20X7 12.6.X7 120 quarter to 31 May 20X7 14.9.X7 104 quarter to 31 August 20X7 10.12.X7 145 quarter to 30 November 20X7 Rent 1.2.X7 375 3 months to 31 March 20X7 6.4.X7 1,584 12 months to 31 March 20X8

Note: On 6 March 20X8 Fiona received an electricity bill for $168 for the quarter to 28 February 20X8.

Page 217: f3 Course Notes

11: ACCRUALS AND PREPAYMENTS

217

Required

(a) Calculate the expense incurred by Fiona for electricity and rent for the year ended 31 December 20X7. (b) Calculate the amount of any accruals/prepayments at the end of the year. (c) State the journal entry required for the year-end adjustments.

Solution

Page 218: f3 Course Notes

11: ACCRUALS AND PREPAYMENTS

218

Lecture example 2 Preparation question Required

Using the figures from Lecture example 1: Complete the necessary entries in Fiona’s ledger accounts as at 31 December 20X7, then balance off the accounts.

Solution Electricity expense (I/S) $ $ 10.3.X7 Cash 96 12.6.X7 Cash 120 14.9.X7 Cash 104 10.12.X7 Cash 145

Rent expense (I/S) $ $ 1.2.X7 Cash 375 6.4.X7 Cash 1,584

Accruals (SOFP) $ $

Prepayments (SOFP) $ $

Section 1.9

Page 219: f3 Course Notes

11: ACCRUALS AND PREPAYMENTS

219

3 Reversing out accruals and prepayments

Problem 3.1 Using the figures from Lecture example 1, what is Fiona’s rent expense for the year to 31

December 20X8 assuming that on 10 April 20X8 she paid rent of $1,740 for the 12 months commencing 1 April 20X8?

3.2 1.1.X8 1.4.X8 31.12.X8

Expense = $1,7011,740)$($1,584)( 129

123 =×+×

Double entry 3.3 Rent expense

$ $ 10.4.X8 Cash 1,740 31.12.X8 Prepayments ( 1,74012

3 × ) 435

Prepayments $ $ 1.1.X8 Balance b/d 396 31.12.X8 Rent 435

This does not produce a sensible answer! The rent expense in the ledger account would result in a charge to the income statement of $1,305 (not $1,701) and the balance on the prepayment account would be overstated by $396.

Page 220: f3 Course Notes

11: ACCRUALS AND PREPAYMENTS

220

Solution 3.4 The opening prepayment must therefore be reversed, ie:

Debit Rent expense (I/S) $396 Credit Prepayments (SOFP) $396 Post this to the ledger accounts in 3.3 and balance off – the expense should now be correct!

Summary 3.5 Accruals and prepayments brought forward at the start of the year must be reversed.

Reversal of accrual Dr Accruals (SOFP) Cr Expense (I/S)

Prepayments Dr Expense (I/S) Cr Prepayments (SOFP)

Approach to questions 3.6 There are four steps to follow:

(1) Reverse opening accrual/prepayment. (2) Post cash paid during the year. (3) Post closing accrual/prepayment. (4) Balance off the accounts.

Lecture example 3 Preparation question In 20X8 Fiona paid the following electricity bills:

Date paid Amount $

Period

12.3.X8 168 quarter to 28 February 20X8 9.6.X8 134 quarter to 31 May 20X8 12.9.X8 118 quarter to 31 August 20X8 12.12.X8 158 quarter to 30 November 20X8

During March 20X9 Fiona received an electricity bill for $189 for the quarter to 28 February 20X9. Required

Calculate the electricity expense and accrual for the year ended 31 December 20X8 and complete the ledger accounts.

Page 221: f3 Course Notes

11: ACCRUALS AND PREPAYMENTS

221

Solution

Electricity expense (I/S) $ $

Accruals (SOFP) $ $

Lecture example 4 Exam standard question for 2 marks Jimmy Co prepares its financial statements for the year to 30 June each year. The company pays for its insurance quarterly in advance on 1 March, 1 June, 1 September and 1 December each year. The annual insurance premium was $24,000 until 31 August 20X6, after that date it increased to $30,000 per year. Required

What insurance expense and end of year prepayment should be included in the financial statements for the year ended 30 June 20X7? Expense Prepayment A $29,000 $2,500 B $29,000 $5,000 C $28,500 $2,500 D $28,500 $5,000

Page 222: f3 Course Notes

11: ACCRUALS AND PREPAYMENTS

222

Solution

Page 223: f3 Course Notes

11: ACCRUALS AND PREPAYMENTS

223

Additional Notes

Page 224: f3 Course Notes

11: ACCRUALS AND PREPAYMENTS

224

4 Accrued income and deferred income 4.1 Accruals and prepayments relate to when expenses are paid in arrears or advance. Income

may also be received in arrears or advance.

Accrued income 4.2 This relates to when income has been earned during the accounting period but not invoiced

or received.

Illustration 4.3 Jenny owns a property which she rents out for $3,000 per quarter. The property was

occupied all year; however Jenny only received $9,000 in rent because she forgot to send out the final invoice of the year. As the property was let for 12 months, Jenny's income statement should show income of $12,000 (4 × $3,000) as this is what she has earned. She will therefore need to accrue the 'missing' income of $3,000 as a year end journal and also show a receivable for "rent in arrears". The adjustment is: $ $ Dr Rent in arrears (SOFP) 3,000 Cr Rental income (I/S) 3,000 The rent in arrears is shown in the statement of financial position within current assets.

Deferred income 4.4 This relates to when income is received in advance of it being earned.

Illustration 4.5 Ben has a year end of December and rents out his property for $1,000 per month. His

tenant pays on time each month and during December 20X7 paid Ben $2,000 as he would be away when the January 20X8 payment was due. Ben has received income of $13,000 but only $12,000 of this relates to the current year. He must therefore remove $1,000 of income from this year's accounts because it relates to next year. A liability will also be shown for "rent in advance". The adjustment is: $ $ Dr Rental income (I/S) 1,000 Cr Rent in advance (SOFP) 1,000 The rent in advance is shown in the statement of financial position within current liabilities.

Page 225: f3 Course Notes

11: ACCRUALS AND PREPAYMENTS

225

Approach to questions 4.6 The approach for accrued income and deferred income is exactly the same as for accruals

and prepayments. There are four steps to follow: (1) Reverse opening rent in arrears/advance. (2) Post cash received during the year. (3) Post closing rent in arrears/advance. (4) Balance off the accounts.

5 Chapter summary Section Topic Summary

1 Introduction An entity should produce its financial statements using the accruals basis. This is an underlying assumption in the IASB Framework. Accruals are made when expenses are paid in arrears, whereas prepayments arise when expenses are paid for in advance.

2 Accounting treatment Accruals increase expenses and are shown as a liability on the statement of financial position at the year end. Prepayments reduce expenses and are an asset on the statement of financial position.

3 Reversing out accruals and prepayments

Accruals and prepayments from the previous year are reversed at the beginning of the next accounting period so that the current year expense is correct.

4 Accrued income and deferred income

These follow a similar theory to accruals and prepayments but relate to income. An entity will accrue income where it has earned the income during the period but not yet invoiced for it. This will increase income and be shown as a receivable at the year end. Where an entity has received income in advance of it being earned it should be deferred to the following period. This will reduce income and be shown as a payable at the year end.

Quick Quiz Q2-5

Page 226: f3 Course Notes

11: ACCRUALS AND PREPAYMENTS

226

6 Double Entry Summary for Chapter 11 6.1 Accruals adjustment:

6.2 Prepayments adjustment:

6.3 Approach to questions (four steps): (1) Reverse opening accrual/ prepayment:

(2) Post cash paid during the year. (3) Post closing accrual/ prepayment. (4) Balance off the ledger accounts.

Dr Expense (I/S) Cr Accruals (SOFP)

Dr Prepayments (SOFP) Cr Expense (I/S)

Accruals: Dr Accruals (SOFP) Cr Expense (I/S) Prepayments: Dr Expense (I/S) Cr Prepayments (SOFP)

Page 227: f3 Course Notes

11: ACCRUALS AND PREPAYMENTS

227

Chapter Summary

Accruals and prepayments

Accounting treatment

Accounting treatment

Reversing out accruals and prepayments

Year end adjustments Presentation in the statement of financial position

Accrued income and deferred income

'Accruals: expenses incurred by the business during the period but not yet paid for'

'Prepayments: expenses paid for before they have been used'

• Accruals increase expenses and represent a liability:

Dr Expenses (I/S) Cr Accruals (SOFP) • Prepayments decrease

expenses and are an asset at the year end:

Dr Prepayments (SOFP) Cr Expenses (I/S)

• Accruals: current liabilities • Prepayments: current assets

• Accruals and prepayments brought forward at the start of the year must be reversed

• Steps to answering questions: (1) Reverse opening accrual/prepayment (2) Post cash paid during the year (3) Post closing accrual/prepayment (4) Balance off the accounts

'Accrued income: income earned but not yet invoiced' 'Deferred income: income invoiced but not yet earned'

Accrued income: Dr Receivable (SOFP) Cr Income (I/S) Deferred income: Dr Income (I/S) Cr Payable (SOFP) • Opening accrued and deferred income balances

must be reversed at the beginning of each year.

Page 228: f3 Course Notes

11: ACCRUALS AND PREPAYMENTS

228

Page 229: f3 Course Notes

229

Chapter 11: Questions

Page 230: f3 Course Notes

11: QUESTIONS

230

Data for Questions 11.1 – 11.3 A company made the following payments in 20X5 in respect of rent and telephone expenses:

Rent Date paid Amount $ Quarter ended 31 January 20X5 02.01.20X5 300 Quarter ended 30 April 20X5 02.01.20X5 300 Quarter ended 31 July 20X5 30.04.20X5 450 Quarter ended 31 October 20X5 31.07.20X5 450 Quarter ended 31 January 20X6 01.11.20X5 450 Telephone Quarter ended 31 January 20X5 02.03.20X5 270 Quarter ended 30 April 20X5 05.06.20X5 310 Quarter ended 31 July 20X5 02.09.20X5 320 Quarter ended 31 October 20X5 10.12.20X5 330

A telephone bill for $345 in respect of the quarter ended 31 January 20X6 was received by the company in February 20X6. The company's year end is December.

11.1 What balance should have been brought forward on the accruals account in relation to rent payable at 1 January 20X5?

A $100 credit

B $200 credit

C $100 debit

D $200 debit (2 marks)

11.2 What will be the income statement charge for telephone expenses for the year ended 31 December 20X5?

A $1,165

B $1,180

C $1,255

D $1,280 (2 marks)

11.3 At 31 December 20X5 what balance will be included as a prepayment or accrual in respect of rent?

A $300 prepayment

B $200 accrual

C $150 prepayment

D $150 accrual (2 marks)

11.4 At 31 December 20X8 Blue Anchor Co has an insurance prepayment of $250. During the year they pay $800 in respect of various insurance contracts. The closing accrual for insurance is $90.

What is the income statement charge for insurance for year ended 31 December 20X9? $

(2 marks)

Page 231: f3 Course Notes

11: QUESTIONS

231

11.5 Max has paid his rent for the period 1 April 20X0 to 30 June 20X1 of $4,800. His first set of accounts is drawn up for the period from 1 April 20X0 to 28 February 20X1. His accounts should reflect

A Rent expense of $4,800 only

B Rent expense of $3,520, a prepayment of $1,280

C Rent expense of $3,600, a prepayment of $1,200

D Rent expense of $3,840, a prepayment of $960 (2 marks)

11.6 Constains Co has an insurance prepayment of $320 at 31 March 20X2. During the year ended 31 March 20X2 Constains paid two insurance bills, one for $1,300 and one for $520. The charge for the year in the accounts for insurance was $1,760.

What was the prepayment at 31 March 20X1? $ (2 marks)

11.7 An electricity prepayment for $300 was treated as an accrual in a sole trader’s income statement. As a result the profit was

A Overstated by $600

B Understated by $300

C Understated by $600 (1 mark)

11.8 A. Cruel

A. Cruel prepares his financial statements for the year to 31 December each year. He pays rent on his premises quarterly in advance on 1 February, 1 May, 1 August and 1 November. The annual rent was $12,000 until 30 September 20X7 and $15,000 per year thereafter.

(i) What rent expense and prepayment should be included in the financial statements for the year ended 31 December 20X7?

Expense Prepayment

A $12,750 $1,250 B $12,750 $2,500 C $15,000 $2,250 D $15,000 $1,250

(ii) The following year the reversal of the prepayment will result in which of the following in the rent expense account?

A Credit balance of $1,250 B Debit balance of $1,250 C Credit balance of $2,500 D Debit balance of $2,250

(iii) A. Cruel has just looked at the accounts you have prepared and is confused as he knows he has paid more rent than is showing in the income statement.

Which accounting concept means that the income statement may not just show the cash paid?

A Going concern B Accruals C Business entity

Page 232: f3 Course Notes

11: QUESTIONS

232

11.9 Fairlop

The accounts of Fairlop are made up to 31 December every year. When preparing the accounts for 20X7 you extract the following information from the payments side of the cash book: $ 20X6 1 October Rent (to 31.3.X7) 500 20X7 10 January Electricity 300 1 April Rent 550 10 April Electricity 300 10 July Electricity 250 1 October Rent 550 10 October Electricity 250 20X8 10 January Electricity 350 You ascertain that rent is paid half-yearly in advance and that electricity bills relate to the quarter ended in the month before payment. Required

Calculate the following amounts:

(i) The rent expense for the year ended 31 December 20X7 (ii) The electricity expense for the year ended 31 December 20X7 (iii) The balance on the prepayment account at 31 December 20X7 (iv) The balance on the accruals account at 31 December 20X7

Page 233: f3 Course Notes

233

Chapter 11: Answers

Page 234: f3 Course Notes

11: ANSWERS

234

11.1 B 32 × 300 = 200

11.2 D $ Reverse accrual at 1.1.X5 (180) Paid (270 + 310 + 320 + 330) 1,230 Accrual at 31.12.X5 ( 3

2 x 345) 230 ∴ I/S charge 1,280

11.3 C 31 × 450 = 150

11.4 $1,140

$250 + $800 + $90 = $1,140

11.5 B Rent for the 15-month period $4,800 Prepayment 15

4 × $4,800 $1,280

11.6 $260 Insurance Expense

$ $ ∴ Prepayment reversal (β) 260 Cash 1,300 I/S 1,760 Cash 520 Prepayment 320 2,080 2,080

11.7 C The prepayment would have decreased the electricity expense by $300 and increased profits. Treating the prepayment as an accrual would have increased the electricity expense and decreased profit. Profit is therefore understated by 2 × $300 = $600.

11.8 A. Cruel

(i) A

Rent expense: $ January – September 20X7 ($12,000 × 9/12) 9,000 October – December 20X7 ($15,000 × 3/12) 3,750 12,750 Prepayment:

1 November payment of $3,750 ($15,000 × ¼) relates to November, December and January.

∴ prepay January 20X8 expense: $3,750 × 1/3 = $1,250.

(ii) B

(iii) B

Page 235: f3 Course Notes

11: ANSWERS

235

11.9 Fairlop

(i) Rent expense: $1,075 (ii) Electricity expense: $1,150 (iii) Prepayments: $275 (iv) Accruals: $350

Workings Prepayments (SOFP)

$ $

1.1.X7 Balance b/d (500 × 3/6) 250 1.1.X7 Rent 250 31.12.X7 Rent (550 × 3/6) 275 31.12.X7 Balance c/d 275 525 525 1.1.X8 Balance b/d 275

Accruals (SOFP) $ $ 1.1.X7 Electricity 300 1.1.X7 Balance b/d 300 31.12.X7 Balance c/d 350 31.12.X7 Electricity 350 650 650 1.1.X8 Balance b/d 350

Rent (I/S) $ $ 1.1.X7 Prepayments 250 1.4.X7 Bank 550 1.10.X7 Bank 550 31.12.X7 Income statement 1,075 31.12.X7 Prepayments 275 1,350 1,350

Electricity (I/S) $ $ 1.1.X7 Accruals 300 10.1.X7 Bank 300 10.4.X7 Bank 300 10.7.X7 Bank 250 10.10.X7 Bank 250 31.12.X7 Accruals 350 31.12.X7 Income statement 1,150 1,450 1,450

Page 236: f3 Course Notes

11: ANSWERS

236

END OF CHAPTER

Page 237: f3 Course Notes

237

12

Syllabus Guide Detailed Outcomes Having studied this chapter you will be able to:

• Explain and identify examples of receivables and payables.

• Identify the benefits and costs of offering credit facilities to customers.

• Understand the purpose of credit limits and an aged receivables analysis.

• Prepare the bookkeeping entries to write off a bad debt, record a bad debt recovered and create and adjust an allowance for receivables.

• Identify the impact of bad debts on the income statement and on the statement of financial position.

• Illustrate how to include movements in the allowance for receivables in the income statement and how the closing balance of the allowance should appear in the statement of financial position.

• Account for contras between trade receivables and payables.

• Prepare, reconcile and understand the purpose of supplier statements.

• Classify items as current or non-current liabilities in the statement of financial position.

Exam Context Questions on this topic are likely to require you to perform basic calculations dealing with writing off debts, adjusting for cash subsequently received and adjusting the allowance for receivables. You will also need to be able to determine the balances to be shown in the income statement and statement of financial position.

Qualification Context This area is a basic skill and detailed calculations are not tested in any other paper.

Irrecoverable debts and allowances

Page 238: f3 Course Notes

12: IRRECOVERABLE DEBTS AND ALLOWANCES

238

Overview

Amounts recovered

Bad debts

Irrecoverable debts and allowances

Allowances

General Specific

Doubtful debts

Page 239: f3 Course Notes

12: IRRECOVERABLE DEBTS AND ALLOWANCES

239

1 Introduction 1.1 This chapter is designed to enable you to calculate and make adjustment for bad debts, and

allowances for receivables.

1.2 A trade receivable should only be classed as an asset if it is probable that it is recoverable (ie that the customer will pay the amounts due).

2 Bad debts 2.1 If a debt is definitely irrecoverable it should be written off to the income statement as a

bad debt. This is an example of prudence.

Accounting treatment 2.2 Dr Bad debt expense (I/S)

Cr Trade receivables (B/S) You may see the debit entry being made to an 'irrecoverable debts expense' account. This is effectively the same thing.

Lecture example 1 Preparation question Fight & Co has trade receivables at 31 December 20X7 of $65,000. A review of customer files indicates that two customers, Ali and Tyson, which owe $7,000 and $8,000 respectively, have gone bankrupt and their debts are considered irrecoverable. Required (a) Calculate the balance c/d on the trade receivables account at the end of the year. (b) Calculate the bad debt expense shown in the income statement.

Solution

Trade receivables (SOFP) $ $ 31.12.X7 Bal b/d 65,000

Bad debt expense (I/S) $ $

Page 240: f3 Course Notes

12: IRRECOVERABLE DEBTS AND ALLOWANCES

240

3 Doubtful debts 3.1 If a debt is possibly irrecoverable an allowance for the potential irrecoverability of that debt

should be made. A new account is created, Allowance for receivables, this account is offset against the trade receivables’ balance on the statement of financial position and the expense taken to the income statement.

Accounting treatment 3.2 Dr Doubtful debts expense (I/S)

Cr Allowance for receivables (SOFP) Again, an 'irrecoverable debts expense' account can also be used.

Lecture example 2 Preparation question A further review of Fight & Co's customer files indicates there is some uncertainty as to whether a debt of $3,500 owed by Bugner is recoverable. (a) Calculate the allowance for receivables shown on the statement of financial position. (b) Calculate the doubtful debts expense shown in the income statement. (c) Show how the information from Lecture examples 1 and 2 would be shown in extracts from

the income statement and statement of financial position.

Solution Allowance for receivables (SOFP)

Doubtful debts expense (I/S)

Page 241: f3 Course Notes

12: IRRECOVERABLE DEBTS AND ALLOWANCES

241

Types of allowance 3.3 (a) Specific: provided against a particular/named individual customer.

(b) General: percentage applied to total trade receivables after: (i) writing off bad debts; (ii) deducting full balance of any customers for which specific allowance has been

created.

Order of calculation 3.4 (a) Write up trade receivables and account for credit sales and cash received in period.

(b) Write off bad debts Dr Bad debt expense (I/S) Cr Trade receivables (SOFP)

(c) Make any entries for specific allowances: Dr Doubtful debts expense (I/S) Cr Allowance for receivables (SOFP)

(d) In workings, calculate the general allowance on trade receivables (after bad debts written off and excluding full amounts for which specific allowance has been made).

(e) $ Total trade receivables 100 Less: specific allowances (20) 80 General allowance @ 5% = 4 ∴total allowance: Specific 20 General 4 24

Page 242: f3 Course Notes

12: IRRECOVERABLE DEBTS AND ALLOWANCES

242

Lecture example 3 Preparation question A business’s trade receivables account showed a year end balance of $47,440. It was decided that amounts totalling $340 should be written off as irrecoverable, a specific allowance was to be made against an amount of $400 due from Dodgy Co, a customer, and a general allowance of 2% was to be made against remaining debts. Required (a) Calculate the allowance for receivables shown in the statement of financial position. (b) Calculate the bad and doubtful debts expense shown in the income statement.

Solution Trade receivables (SOFP) $ $ Balance b/d 47,440

Allowances for receivables (SOFP) $ $

Bad and doubtful debts expense (I/S) $ $

General allowance

$ Trade receivables (net of bad debts written off) Less: specific allowance General allowance @ 2%

Page 243: f3 Course Notes

12: IRRECOVERABLE DEBTS AND ALLOWANCES

243

4 Effect in subsequent periods

Bad debts written off last year, customer pays this year 4.1 If a bad debt is recovered having previously been written off, it is credited to the bad debt

expense account, i.e. the accounting treatment from the original write-off is reversed. Accounting treatment (1) Cash received

Dr Cash Cr Trade receivables

Reverse original write off

Dr Trade receivables Cr Bad debt expense

OR (2) Short method

Dr Cash Cr Bad debt expense

Lecture example 4 Preparation question Fight & Co (see Lecture example 1) subsequently receive a cheque of $7,000 from Ali. Required

Show the treatment of this recovery in the relevant ‘T’ accounts.

Solution

Trade receivables (SOFP) $ $ 1.1.X8 Bal b/d 50,000

Page 244: f3 Course Notes

12: IRRECOVERABLE DEBTS AND ALLOWANCES

244

Bad debt expense (I/S) $ $

Cash (SOFP) $ $

Doubtful debts – specific allowance last year, customer pays outstanding amounts this year 4.2 A credit entry for the cash is made to the trade receivables account because the debt is still

included in the total trade receivables figure. The allowance is then reversed as it is no longer needed. Accounting treatment (a) Record the cash received

Dr Cash (SOFP) Cr Trade receivables (SOFP)

then: (b) Remove allowance

Dr Allowance for receivables (SOFP) Cr Doubtful debts expense (I/S)

Page 245: f3 Course Notes

12: IRRECOVERABLE DEBTS AND ALLOWANCES

245

Lecture example 5 Preparation question Required

Show the accounting treatment for Fight & Co if, having made a specific allowance (see Lecture example 2), during the next year Bugner repays his debt of $3,500 to Fight & Co in cash?

Solution

Trade receivables (SOFP) $ $ 1.1.X8 Bal b/d 50,000

Allowance for receivables (SOFP) $ $ 1.1.X8 Bal b/d 3,500

Bad and doubtful debt expense (I/S) $ $

Doubtful debts – specific allowance last year, goes bad this year 4.3 The debt is no longer doubtful, but definitely bad. It should therefore be removed from the

trade receivables and the allowance for receivables accounts.

Dr Allowance for receivables (SOFP) Cr Trade receivables (SOFP)

Page 246: f3 Course Notes

12: IRRECOVERABLE DEBTS AND ALLOWANCES

246

Lecture example 6 Preparation question Required

Following on from the information used in Lecture example 2, suppose that in the next accounting period, the debt from Bugner is considered to have gone bad. What double entry would be required to record this?

Solution

Doubtful debts - general allowance 4.4 Allowance is usually changed at the end of each period to reflect the change in value of total

trade receivables.

Accounting treatment 4.5

(1) Remove opening allowance Dr Allowance for receivables Cr Doubtful debts expense Replace with closing allowance Dr Doubtful debts expense Cr Allowance for receivables

Page 247: f3 Course Notes

12: IRRECOVERABLE DEBTS AND ALLOWANCES

247

OR (2) Short method: Increase/decrease opening allowance to arrive at required closing allowance Increase: Dr Doubtful debts expense Cr Allowance for receivables Decrease: Dr Allowance for receivables Cr Doubtful debts expense

Lecture example 7 Preparation question The following information is available for A Co. Year ended 31 December 20X7: Trade receivables $20,000 Year ended 31 December 20X8: Trade receivables $30,000 A Co requires a general allowance of 5% of trade receivables in each year. Required Show the required adjustment to the allowance for receivables account in the year ended 31 December 20X8 using both methods described in section 4.5

Solution Long method: 4.5 (1)

Allowance for receivables $ $

Doubtful debts expense $ $

Page 248: f3 Course Notes

12: IRRECOVERABLE DEBTS AND ALLOWANCES

248

Short method: 4.5 (2) Allowance for receivables

$ $

Doubtful debts expense $ $

Lecture example 8 Exam standard for 2 marks At 30 September 20X7 G Co had an allowance for receivables of $24,000. During the year ended 30 September 20X8 G Co recovered $2,000 from a customer whose balance was written off in 20X7 and wrote off further debts totalling $18,000. The closing allowance for receivables is required to be $21,000. No adjustments have been made for this information. Required What amount should appear in the income statement for the year ended 30 September 20X8 for the above items? A $13,000 B $15,000 C $17,000 D $23,000

Page 249: f3 Course Notes

12: IRRECOVERABLE DEBTS AND ALLOWANCES

249

Solution

5 Chapter summary Section Topic Summary

1 Introduction A trade receivable is an asset of the business which should only be shown in the financial statements if it is believed to be recoverable.

2 Bad debts Bad or irrecoverable debts must therefore be written off as an expense in the income statement.

3 Doubtful debts An allowance should be made against trade receivables where there is concern as to whether or not a balance will be recoverable. There are two types of allowance: specific and general. Specific allowances relate to particular customer balances whereas a general allowance is usually a percentage of remaining debts.

4 Effect in subsequent periods

The key to being able to account for the effect in subsequent periods is to know what accounting entries have previously been made and then make any relevant adjustments. For example, if cash is received from a receivable that was previously written off then the receivable has already been removed from the accounts. Consequently the only adjustments needed are to record the cash received and remove the bad debt expense recorded last year which has proved to be unnecessary.

Quick Quiz

Page 250: f3 Course Notes

12: IRRECOVERABLE DEBTS AND ALLOWANCES

250

6 Double Entry Summary for Chapter 12 6.1 Bad (irrecoverable) debt adjustment:

6.2 Doubtful debt adjustment:

6.3 Recording of cash received from a customer whose balance was previously written off:

6.4 Recording of cash received from a customer against which a specific allowance was previously made:

6.5 Writing a balance off as irrecoverable where a specific allowance was previously made:

Dr Bad debt expense (I/S) Cr Trade receivables (SOFP)

Dr Doubtful debts expense (I/S) Cr Allowance for receivables (SOFP)

Dr Cash (SOFP) Cr Bad debt expense (I/S)

Record cash received: Dr Cash (SOFP) Cr Trade receivables (SOFP) Remove the allowance: Dr Allowance for receivables (SOFP) Cr Doubtful debts expense (I/S)

Dr Allowance for receivables (SOFP) Cr Trade receivables (SOFP)

Page 251: f3 Course Notes

12: IRRECOVERABLE DEBTS AND ALLOWANCES

251

Chapter Summary

Amounts recovered

Bad debts

Irrecoverable debts and allowances

Allowances

General Specific

Doubtful debts

• Record cash received and remove bad debt expense which was previously recognised:

Dr Cash Cr Bad debt expense

'A debt which is definitely irrecoverable' • Write off to the income statement: Dr Bad debt expense (I/S) Cr Trade receivables (SOFP)

'A debt which is possibly irrecoverable' • Make an allowance against the debt: Dr Doubtful debts expense (I/S) Cr Allowance for receivables (SOFP)

• Reduce the value of trade receivables shown in the statement of financial position

• Provided where there is doubt over the recoverability of a particular customer's balance

• When cash is subsequently received (1) Record cash: Dr Cash Cr Trade receivables (2) Remove allowance Dr Allowance for receivables Cr Doubtful debts expense • If debt subsequently goes bad remove from trade receivables and the

allowance for receivables: Dr Allowance for receivables Cr Trade receivables • Note the 'loss' was originally recorded when the allowance was made.

• A percentage applied to total trade receivables after

(1) writing off bad debts (2) deducting the total balance owed by

customers where a specific allowance has been made

• The general allowance is increased or decreased as necessary at each year end

Page 252: f3 Course Notes

12: IRRECOVERABLE DEBTS AND ALLOWANCES

252

Page 253: f3 Course Notes

253

Chapter 12: Questions

Page 254: f3 Course Notes

12: QUESTIONS

254

12.1 A company receives news that a major customer has been declared bankrupt. The entries now required are:

A Debit bad debt expense, Credit trade receivables

B Debit sales, Credit trade receivables (1 mark)

12.2 At 1 January 20X9 Farriers has an allowance for receivables of $2,000 consisting of a specific allowance for $700 in respect of Black Lion Co and a $1,300 general allowance. During the year Black Lion goes into liquidation and the debt is written off. No other debts go bad and at 31 January 20X9 the balance on the trade receivables is $50,950. Farriers wishes to provide for a debt of $950 from Verulam and to have a general allowance of 2½% of good trade receivables. The bad and doubtful debts charged to the income statement for 20X9 is:

A $900

B $924

C $1,600

D $2,200 (2 marks)

12.3 The preliminary trial balance of Jessie and Co as at 30 September 20X7 included:

Debit Credit $ $ Trade receivables 90,350 Allowance for receivables (brought forward as at 1 October 20X6) 2,490 Bad and doubtful debt expense 1,985 Further adjustments are to be made as follows:

(i) No entries have been made in respect of cash of $1,320 received from Dome Co whose balance had been written off last year, and

(ii) At 30 September 20X7 an allowance is required against a balance of $1,950 due from Jed Co as well as a general allowance of 1.5% of remaining debts.

What is the bad and doubtful debt expense in the income statement? $ (2 marks)

12.4 Gillian

On 31 December 20X4, Gillian’s nominal ledger included a trade receivables balance of $47,900 along with an allowance for receivables (brought forward as at 1 January 20X4) of $2,551. Of this $537 relates to a specific customer, the remainder being a general allowance. After a review of trade receivables at the year end, the following adjustments are to be made:

(1) Debts totalling $1,615 are to be written off as irrecoverable.

(2) No entry has yet been made in the books for $418 cash received on 31 December 20X4 from David, a customer whose debt was written off during 20X3.

(3) Cash posted to the trade receivables account during the year include $537 from Jim. The amount due from Jim had been specifically provided against at 31 December 20X3.

(4) Specific allowance is to be made against debts totalling $835 together with a general allowance of 2%.

Required

(a) Write up the relevant ledger accounts for the year ended 31 December 20X4. (b) Show the relevant extracts from the financial statements.

Page 255: f3 Course Notes

12: QUESTIONS

255

12.5 Johnson & Co

(1) Johnson & Co had total receivables owing to them at 31 December 20X7 of $9,650. They included $700 owed by T Black, who had fled the country six months earlier, and various debts due from K White, totalling $335 and dating back to the years 20X1-20X5. It was decided that the above debts should be written off.

(2) During 20X8 Johnson & Co made sales on credit of $40,385 and received cash from trade receivables of $32,050. There were no irrecoverable debts. However, there was some doubt as to whether a debt of $450 owed by J Green would be met and it was decided to make an allowance against this specific debt and against 2% of the remaining debts.

(3) During 20X9 credit sales totalled $50,235 and cash of $37,140 was received from trade receivables. A review of trade receivables at the year end revealed the following:

(i) The amount owed by J Green was now considered irrecoverable and should be written off; (ii) Other irrecoverable debts totalling $545 were to be written off; (iii) Allowance was to be made against an amount of $250 owed by P Brown; (iv) The general allowance was to be maintained at 2% of good debts.

Required

Produce ledger accounts to record the above transactions for the years ended 31 December 20X7, 20X8 and 20X9.

Page 256: f3 Course Notes

12: QUESTIONS

256

Page 257: f3 Course Notes

257

Chapter 12: Answers

Page 258: f3 Course Notes

12: ANSWERS

258

12.1 A

12.2 A $ Trade receivables balance 50,950 Less specific allowance (950) 50,000 General allowance = 2½% × $50,000 = $1,250

Movement in general allowance is a reduction of $50 ($1,300 – $1,250)

Charge to I/S $ Specific allowance Verulam 950 Less: decrease in general allowance (50) 900

12.3 $1,451

$ Bad and doubtful debts expense per trial balance 1,985 Less: bad debt recovered (1,320) Add: increase in allowance (W) 786 1,451 $ Allowance c/d – specific 1,950 – general 1.5% × (90,350 – 1,950) 1,326 3,276 Less allowance b/d 2,490 ∴ increase 786

12.4 Gillian

(a) Trade receivables (SOFP)

$ $ 31.12.X4 Balance b/d 47,900 31.12.X4 Bad debts 1,615 31.12.X4 Balance c/d 46,285 47,900 47,900

Allowance for receivables (SOFP)

$ $ Irrecoverable & doubtful debts (β) 807 1.1.X4 Balance b/d 2,551 31.12.X4 Balance c/d (W1) 1,744 2,551 2,551

Irrecoverable and doubtful debts expense (I/S)

$ $ Trade receivables 1,615 Bank (bad debt recovered) 418 Allowance for receivables 807 ∴ Income statement 390 1,615 1,615

Page 259: f3 Course Notes

12: ANSWERS

259

Working Receivables Allowance $ $ Trade receivables 46,285 Less: specific allowance (835) 835 45,450 General allowance ($45,450 × 2%) 909 1,744

(b) Gillian Statement of financial position as at 31 December 20X4 (extract)

CURRENT ASSETS $ $ Trade receivables 46,285 Less: allowance for receivables (1,744) 44,541

Income statement for the year ended 31 December 20X4 (extract) $ Less expenses: Irrecoverable and doubtful debts expense 390

12.5 Johnson & Co Trade receivables (SOFP)

$ $ 31.12.X7 Balance b/d 9,650 31.12.X7 Irrecoverable

debts expense (700 + 335)

1,035

31.12.X7 Balance c/d 8,615 9,650 9,650 1.1.X8 Balance b/d 8,615 Bank 32,050 Sales 40,385 31.12.X8 Balance c/d 16,950 49,000 49,000 1.1.X9 Balance b/d 16,950 Bank 37,140 Sales 50,235 31.12.X9 Irrecoverable

debts expense 995

(450 + 545) 31.12.X9 Balance c/d 29,050 67,185 67,185 1.1.YO Balance b/d 29,050

Irrecoverable (bad) & doubtful debts expense (I/S)

$ $ 31.12.X7 Trade receivables 1,035 31.12.X7 Income statement 1,035 31.12.X8 Allowance for receivables (W1)

780 31.12.X8 Income statement 780

31.12.X9 Trade receivables 995 31.12.X9 Allowances for receivables

46

31.12.X9 Income statement 1,041

1,041 1,041

Page 260: f3 Course Notes

12: ANSWERS

260

Allowance for receivables (SOFP) $ $ 31.12.X8 Balance b/d 780 31.12.X8 Irrecoverable and

doubtful debts expense

780

1.1.X9 Balance b/d 780 31.12.X9 Irrecoverable and

doubtful debts expense (β)

46

31.12.X9 Balance c/d (W2) 826 826 826

Workings (W1) Allowance for receivables as at 31 December 20X8.

Receivables Allowance $ $ Trade receivables 16,950 Less: specific allowance (J Green) (450) 450 16,500 General allowance ($16,500 × 2%) 330 780

(W2) Allowance for receivables as at 31 December 20X9.

Receivables Allowance $ $ Trade receivables 29,050 Less: specific allowance (P Brown) (250) 250 28,800 General allowance required ($28,800 × 2%) 576 826

END OF CHAPTER

Page 261: f3 Course Notes

261

13

Syllabus Guide Detailed Outcomes Having studied this chapter you will be able to:

• Understand the definition of 'provision', 'contingent liability' and 'contingent asset', distinguish between them and classify items accordingly.

• Identify and illustrate the different methods of accounting for provisions, contingent liabilities and contingent assets.

• Calculate provisions and changes in provisions and account for the movement in provisions.

• Report provisions in the final accounts.

Exam Context Questions on this area are likely to focus on identifying when a provision or contingent liability should be made or disclosed in the financial statements. You may also be required to calculate a provision. The Pilot Paper included a question on how a remote contingent liability should be accounted for.

Qualification Context Your understanding of IAS 37 will be developed at the Fundamentals level paper Financial Reporting (F7) where you are likely to have to consider whether the provision criteria are satisfied based on more subjective scenarios.

Provisions and contingencies

Page 262: f3 Course Notes

13: PROVISIONS AND CONTINGENCIES

262

Overview

Contingent assets

Accounting treatment Recognition criteria

Contingent liabilities

Provisions

Provisions and contingencies

Page 263: f3 Course Notes

13: PROVISIONS AND CONTINGENCIES

263

1 IAS 37: Provisions, contingent liabilities and contingent assets

1.1 Introduction Before the introduction of IAS 37, there was little guidance on when a provision must and must not be made. This caused problems as entities tended to choose to make and then release provisions in order to smooth out profits, rather than making a provision where they had an obligation to incur expenditure. IAS 37 aims to prevent this happening in the future.

2 Provisions 2.1 Definition

A provision is a liability of uncertain timing or amount.

2.2 Recognition A provision should only be recognised (ie. included in the financial statements) when: (a) An entity has a present obligation (legal or constructive) as a result of a past event; (b) It is probable that an outflow of economic resources will be required to settle the

obligation; and (c) A reliable estimate can be made of the amount of the obligation. Unless all three conditions are met, no provision can be recognised.

2.3 Legal obligation A legal obligation usually arises out of a contract. Illustration Grass Co sells lawnmowers and offers a one-year warranty on all models. Once Grass Co sells a lawnmower (the past event) it has a legal obligation to repair any defects according to the warranty agreement. It should therefore make an estimate of the probable costs of repair and make a provision for this amount in its financial statements.

2.4 Constructive obligation A constructive obligation arises through past behaviour and actions where the entity has raised a valid expectation that it will carry out a particular action. Illustration Seed Co also sells lawnmowers. It does not offer a warranty on its products; however it has a reputation for making free reasonable repairs to lawnmowers bought from the business. Customers buying from Seed Co all expect to receive this benefit.

Page 264: f3 Course Notes

13: PROVISIONS AND CONTINGENCIES

264

Here no warranty is offered and so Seed Co does not have a legal obligation. Its past actions however have created a constructive obligation. It should also therefore make a provision for the probable costs of repairs.

2.5 Accounting treatment The provision represents both a cost to the business and a potential liability:

Dr Expense (I/S) Cr Provision (SOFP)

The required provision will be reviewed at each year end and increased or decreased as necessary. To increase a provision:

Dr Expense (I/S) Cr Provision (SOFP)

To decrease a provision: Dr Provision (SOFP) Cr Expense (I/S)

Lecture example 1 Preparation question Grass Co is reviewing its warranty obligations. Based on sales during 20X7 it has established that if all lawnmowers sold required minor repairs this would cost $1m whereas if major repairs were required this would cost $6m. Grass Co expects that 75% of lawnmowers will have no faults, 20% will need minor repairs and 5% major repairs. Required (a) What provision should be made in 20X7 and what accounting entry is needed to record it? (b) What entry should be made in 20X8 assuming the provision required then is $0.75m? (c) What entry should be made in 20X9 assuming the provision required then is $0.3m?

Solution

Page 265: f3 Course Notes

13: PROVISIONS AND CONTINGENCIES

265

3 Contingent liabilities 3.1 A contingent liability is an uncertain liability that does not meet the three criteria for

recognising a provision. IAS 37 defines a contingent liability as the following: (a) A possible obligation that arises from past events and whose existence will be

confirmed only the occurrence or non-occurrence of one or more uncertain future event not wholly within the control of the entity; or

(b) A present obligation that arises from past events but is not recognised because: (i) it is not probable that an outflow of economic resources will be required to

settle the obligation; or (ii) the amount of the obligation cannot be measured with sufficient reliability.

Contingent liabilities should be disclosed in the notes unless probability of an outflow of resources embodying economic benefits is remote.

Illustrative example 3.2 Company A has entered into an agreement to act as guarantor on a bank loan taken out by

Mr Smith. Mr Smith is a financially secure individual, and the directors are of the opinion that the chances of him defaulting on the loan are slim. How should company A account for this guarantee?

Solution 3.3 Company A has a present obligation (it is legally obliged to honour the guarantee).

However, as the likelihood of Company A having to pay out under the guarantee is not probable then no provision for the liability should be made. Instead, the guarantee should be disclosed in the notes as a contingent liability (unless considered remote, in which case it should be ignored altogether).

Page 266: f3 Course Notes

13: PROVISIONS AND CONTINGENCIES

266

3.4 Decision Tree

4 Contingent assets 4.1 A possible asset that arises from past events and whose existence will be confirmed only by

the occurrence or non-occurrence of one or more uncertain future events not wholly within the control of the entity. Contingent assets should be disclosed in the notes where an inflow of economic benefits is probable, otherwise they should be ignored. If the probability of an inflow of economic benefits is virtually certain then the asset is not a contingent asset and should be recognised in the financial statements.

Page 267: f3 Course Notes

13: PROVISIONS AND CONTINGENCIES

267

5 Chapter summary Section Topic Summary

2 Provisions A provision should only be made in the financial statements when an entity has a present obligation to incur expenditure. It must also be more likely than not that the expenditure will be incurred and a reliable estimate of the amount is known.

3 Contingent liabilities A contingent liability should be disclosed where the criteria for making a provision are not met, but where there is either a possible obligation or a present obligation but it is only possible that the expenditure will be incurred.

4 Contingent assets Contingent assets should only be included in the financial statements if it is certain to be received and should be disclosed if probable.

6 Double Entry Summary for Chapter 13 6.1 Adjustment to create or increase a provision:

6.2 Adjustment to decrease a provision:

Dr Expense (I/S) Cr Provision (SOFP)

Dr Provision (SOFP) Cr Expense (I/S)

Quick Quiz

Page 268: f3 Course Notes

13: PROVISIONS AND CONTINGENCIES

268

Chapter Summary

Contingent assets

Accounting treatment Recognition criteria

Contingent liabilities

Provisions

Provisions and contingencies

• Recognise in financial statements: Dr Expense (I/S) Cr Provision (SOFP)

• Entity has a present obligation as a result of a past event

• It is probable that an outflow of economic resources will be required to settle the obligation

• A reliable estimate can be made of the amount

'A liability of uncertain timing or amount'

'An uncertain liability that does not meet the three criteria for recognising a provision' • Possible obligation • Present obligation - which is not probable - where the amount cannot be

measured reliably • Disclose in a note to the financial

statements

'A possible asset that arises from past events and whose existence will be confirmed by one or more uncertain future events not wholly within the control of the entity'. • Disclose where probable • Recognise if virtually certain

Page 269: f3 Course Notes

269

Chapter 13: Questions

Page 270: f3 Course Notes

13: QUESTIONS

270

13.1 H Co is currently in the middle of a protracted lawsuit which it is vigorously defending. The directors are reasonably confident that the action will not be successful but are aware that the opposite outcome is a possibility. It is difficult to quantify any potential damages, but the directors feel they are unlikely to exceed $50,000.

How should the above item be treated in the financial statements?

A Provision

B Contingent liability

C Contingent asset (1 mark)

13.2 How should a contingent liability and a probable contingent asset be accounted for?

A Probable contingent assets and contingent liabilities should be disclosed in the financial statements.

B Probable contingent assets must always be accrued and contingent liabilities must always be disclosed in the financial statements.

C Contingent liabilities must always be either accrued or disclosed and probable contingent assets must always be disclosed in the financial statements.

D Contingent liabilities must always be provided for and probable contingent assets must be disclosed in the financial statements.

(2 marks)

Page 271: f3 Course Notes

271

Chapter 13: Answers

Page 272: f3 Course Notes

13: ANSWERS

272

13.1 B

13.2 A

END OF CHAPTER

Page 273: f3 Course Notes

273

14

Syllabus Guide Detailed Outcomes Having studied this chapter you will be able to:

• Understand the purpose of control accounts for accounts receivable and accounts payable.

• Understand how control accounts relate to the double entry system.

• Prepare ledger control accounts from given information.

• Perform basic control account reconciliations for accounts receivable and accounts payable and identify errors which would be highlighted by performing them.

• Identify and correct errors in control accounts and ledger accounts.

• Account for discounts allowed and discounts received.

• Account for contras between trade receivables and trade payables.

Exam Context Questions on this topic are likely to require you to correct the closing balance on a receivables or payables control account including items such as contras and discounts. You may also be required to calculate receivables/payables balances where goods are sold/bought with trade and/or settlement discounts.

Qualification Context This chapter covers topics which are only examined in Financial Accounting.

Control accounts

Page 274: f3 Course Notes

14: CONTROL ACCOUNTS

274

Overview

Control accounts

Trade discounts

Reconciliations

Contra entries Discounts allowed and received

Returns, credit notes, refunds and over payments

Settlement discounts

Receivables ledger control account Payables ledger control account

Receivables ledger Payables ledger

Sales tax considerations

Page 275: f3 Course Notes

14: CONTROL ACCOUNTS

275

1 Recap 1.1 In Chapters 4 and 5 we saw how a business' transactions were categorised in the books of

prime entry. The totals of these were then posted using double entry to the nominal ledger to give a summary of the information.

1.2 For example, credit sales:

1.3 The nominal ledger contains three ledger accounts which are affected when a business sells on credit: (a) Sales (b) Bank (c) Trade receivables – this shows the total amount owed by all customers at a

particular point in time. – it is also called the receivables ledger control account

(RLCA)

1.4 In order to chase overdue debts however a business must know how much each customer owes at a particular time. This balance could be determined by going back into the detail of the books of prime entry and extracting the information for each customer. This is a very time consuming process and so instead a memorandum ledger is maintained for each individual customer showing invoices raised, cash received and therefore the amount owed to the business. This memorandum ledger is called a receivables ledger.

1.5 The reverse is true when a business buys on credit.

Page 276: f3 Course Notes

14: CONTROL ACCOUNTS

276

Terminology 1.6 In the nominal ledger:

• Receivables ledger control account (trade receivables/RLCA): total owed by all credit customers.

• Payables ledger control account (trade payables/PLCA): total owed to all credit suppliers.

Memorandum ledgers: • Receivables ledger:

balance owed by each individual credit customer • Payables ledger:

balance owed to each individual credit supplier

2 The flow of information 2.1

Trial Balance

Financial Statements

Memo Receivables

Ledger

Sales Invoice

Receiptfrom

customers

Paymentto

suppliersPurchase Invoice

SDB PDB Cash book MemoPayablesLedger

Nominal Ledger PLCA Trade payables

Bank

Sales Purchases

Customer A

Customer B

Customer C

Supplier X

Supplier Y

Supplier Z

Page 277: f3 Course Notes

14: CONTROL ACCOUNTS

277

2.2 The information in the receivables ledger control account (RLCA) and receivables ledger (RL) is posted from the same source documents. Therefore

the balance on the RLCA should equal the sum of all balances from the RL

Similarly

the balance on the PLCA should equal the sum of all balances from the PL

2.3 If the balances do not agree then an error has been made. This will be identified through a control account reconciliation (Section 5).

Lecture example 1 Preparation question A Co has the following information: 10 January 20X6 Sells $150 of goods to customer A Sells $200 of goods to customer B 15 January 20X6 A Co purchases $100 of goods from supplier Y A Co purchases $1,300 of goods from supplier Z 21 January 20X6 A Co receives full payment from customer B and this money is used to pay supplier Y. Required

(1) Record the above transactions in the books of prime entry and the memorandum ledgers. (2) Post the totals from the BOPE to the nominal ledger. (3) Balance off nominal ledger accounts. (4) Reconcile the memorandum ledgers to the control accounts.

Solution (1) Books of prime entry

Sales day book

Date Customer Amount

Page 278: f3 Course Notes

14: CONTROL ACCOUNTS

278

Purchase day book

Date Supplier Amount

Cash receipts book

Date Narrative Total Sales Receivables

Cash payments book

Date Narrative Total Purchases Payables

Memorandum ledgers Receivables ledger

Customer A Customer B

Payables ledger Supplier Y Supplier Z

Page 279: f3 Course Notes

14: CONTROL ACCOUNTS

279

(2) & (3) Nominal ledger RLCA (SOFP) PLCA (SOFP)

Bank (SOFP) Sales (I/S)

Purchases (I/S)

(4) Reconciliation Balance per list of balances $ Receivables ledger Customer A Customer B Balance per RLCA Balance per list of balances $ Payables ledger Supplier Y Supplier Z Balance per PLCA

Page 280: f3 Course Notes

14: CONTROL ACCOUNTS

280

3 Other entries A business must ensure that any transaction recorded in the receivables ledger control account or the payables ledger control account is also reflected in the memorandum ledgers.

Contra entries 3.1 Sometimes a business may have a customer which also supplies the business with goods.

Illustration: P Co is a printing business which sells stationery to F Co, a florist. F Co supplies P Co with flowers and plants for its offices. During October, P Co sells stationery worth $200 to F Co and F Co delivers flowers and plants to P Co worth $70. P Co has the following amounts in its books: Receivables: $200 Payables: $70 The two businesses agree to offset the balances receivable and payable via a contra. The contra will be for the lower of the two amounts: $70. This will decrease both receivables and payables by $70 and the remaining $130 can then be paid in cash.

3.2 A contra entry is always recorded as: Dr PLCA Cr RLCA This will reduce both receivables and payables.

3.3 Note that the memorandum ledgers will also need to be updated for the contra entry.

Returns, credit notes and refunds 3.4 Sometimes when a business has made a sale, the customer will return the goods.

3.5 Steps: (1) Goods are sold to the customer for $250:

Dr RLCA $250 Cr Sales $250

(2) Customer pays for goods: Dr Bank $250 Cr RLCA $250 At this point the balance on the receivables ledger control account is nil.

Page 281: f3 Course Notes

14: CONTROL ACCOUNTS

281

(3) Customer returns the goods and is issued with a credit note: Dr Sales (returns) $250 Cr RLCA $250 This entry reverses the original sale. The receivables ledger control account will show a credit balance reflecting that the business owes money to the customer. This could be offset against future purchases or the customer may request a refund.

(4) The business refunds the customer: Dr RLCA $250 Cr Bank $250 Once again the balance on the receivables ledger control account is nil.

3.6 Again, the memorandum ledgers must also be updated.

Over payment 3.7 If a customer pays too much to settle an invoice or pays an invoice twice the business will

owe the excess to the customer. This may be held and treated like a credit note or the monies refunded to the customer.

Interest on overdue accounts 3.8 If a customer is late in settling their account then an entity may decide to charge them

interest. This will increase the amount they owe and will be shown as interest receivable in the income statement. Interest on overdue accounts is recorded using the following journal:

Dr RLCA Cr Interest receivable (I/S)

4 Discounts 4.1 There are two types of discounts:

(a) Trade discounts

(i) given at the time of the sale/purchase, they reduce the selling price as an inducement to purchase;

(ii) usually for regular customers or bulk buyers. (b) Settlement discounts

(i) offered, but not necessarily taken, as an inducement to settle a debt early; (ii) eg. 5% discount if settled within 14 days.

Page 282: f3 Course Notes

14: CONTROL ACCOUNTS

282

Terminology 4.2 Discounts allowed: offered by the business to their customer.

Discounts received: received by a business from their supplier.

Discounts allowed 4.3 Accounting treatment

Sales are recorded net of (i.e. after) trade discounts but inclusive of (i.e. before) settlement discounts. Therefore trade discounts never appear in the financial statements. Settlement discounts allowed are recorded as discounts allowed and are shown as an expense in the income statement: Dr Discounts allowed (I/S) Cr RLCA (SOFP)

Lecture example 2 Preparation question (a) On 1 January 20X7 a business made a sale on credit for $12,000. A trade discount of

$2,000 was available with a further 10% settlement discount if payment were made within 10 days. Required Record the initial sale.

Solution

The initial sale would be recorded as:

Sales (I/S) RLCA (SOFP)

(b) On 4.1.X7, the customer pays for the goods taking advantage of the settlement discount. The discount will be 10% of sales value. Required

Record the full settlement of the amount owed.

Page 283: f3 Course Notes

14: CONTROL ACCOUNTS

283

Solution

Bank (SOFP) Discounts allowed (I/S)

(c) Required What would your answer be to part (b) if the settlement discount were not taken?

Solution

Bank (SOFP) RLCA (SOFP)

Discounts received 4.4 Accounting treatment

Purchases are recorded net of trade discounts but inclusive of settlement discounts. Again trade discounts never appear in the financial statements. Settlement discounts received are recorded as discounts received and are shown as sundry income in the income statement. Dr PLCA (SOFP) Cr Discounts received (I/S)

Page 284: f3 Course Notes

14: CONTROL ACCOUNTS

284

Lecture example 3 Preparation question Ryan Co purchases goods worth $5,000 from Austin Co. Ryan Co will receive a 5% settlement discount if the goods are paid for within seven days. Ryan Co has every intention of taking advantage of the settlement discount. Required (a) Show the initial recording of the purchase. (b) Record the payment for the goods assuming Ryan pays within seven days. (c) Record the payment for the goods if payment is made after seven days.

Solution

Page 285: f3 Course Notes

14: CONTROL ACCOUNTS

285

Sales tax and discounts 4.5 Sales tax is calculated on the amount after all discounts, regardless of whether the

discount is taken or not.

Lecture example 4 Exam standard for 2 marks Brick buys goods with a list price of $50,000 from Cement. Brick receives a trade discount of 12% from Cement and a further discount of 4% if payment is made within 10 days. Sales tax is at 15%. Required

What amount should Brick show in Cement's payables ledger to record this purchase? A $48,576 B $50,336 C $50,600 D $57,500

Solution

5 Control account reconciliations 5.1 As mentioned in Section 2 if we add up the balances in the receivables and payables

ledgers, they should agree to the balances per the RLCA and PLCA. If not, an error must have occurred at some point in the system. The easiest way to identify the error is to perform a reconciliation between the two amounts.

Page 286: f3 Course Notes

14: CONTROL ACCOUNTS

286

5.2 Proforma control account reconciliation RLCA

$ $ Balance b/d X Transposition error in posting X Sales day book undercast X Sales omitted from SDB X Balance c/d X X X Balance b/d X

Reconciliation Statement

$ $ $ + – Total per listing of receivables ledger balances

X

Adjustments

Balance omitted X Credit balance listed as debit (2X)

X X X Balance as per adjusted control account X

Lecture example 5 Technique demosntration (a) Required

Post the following transactions to and balance off the receivables ledger control account. (1) Opening balance $614,000 (2) Credit sales made during the month $302,600 (3) Receipts from customers $311,000 (4) Bad debts were written off $32,000 (5) Discounts allowed for prompt payment $3,400 (6) Contras against amounts due to suppliers in payables ledger $8,650

(b) The receivables ledger list of balances totals to $563,900. You have found the following errors: (i) The total of the sales day book was undercast by $3,600. (ii) A credit balance of $450 was included in the list of balances as a debit. (iii) A customer balance of $2,150 was left out when the receivables ledger list of

balances was totalled. Required

Reconcile the receivables ledger control account to the receivables ledger list of balances.

Page 287: f3 Course Notes

14: CONTROL ACCOUNTS

287

Solution

Page 288: f3 Course Notes

14: CONTROL ACCOUNTS

288

6 Chapter summary Section Topic Summary

1 Recap The balance of the receivables ledger control account and the payables ledger control account in the nominal ledger show the total owed by all credit customers and due to all credit suppliers. The purpose of the memorandum ledgers is to show the balance on each individual customer or supplier account.

2 The flow of information Given that the nominal ledger and the memorandum ledgers are updated from the same source documentation, at any point in time the balance on the control accounts should equal the total of all the balances in the memorandum ledgers. Where the two balances are not the same an error must have arisen and a reconciliation should be performed to identify the errors (section 5).

3 Other entries If an entity has a customer is also a supplier the two parties may choose to settle their accounts by making a contra entry. The contra is always for the lower of the two balances. If a customer returns goods having paid for them or overpays for goods then the entity will owe money back to that customer and the customer will have a credit balance on their account. If a customer is late in settling their account the entity may decide to charge them interest on the overdue account. This will increase the balance owed.

4 Discounts Sometimes a business may offer discounts to attract custom. There are two types of discounts: trade discounts and settlement discounts. Sales and purchases are recorded after trade discounts but before settlement discounts. Sales tax is calculated on the amount after all discounts, regardless of whether the discount is taken or not.

5 Control account reconciliations

As detailed in section 2 if the balance on the control account does not agree to the total of all the balances on the memorandum ledger then an error must have occurred and a reconciliation will need to be carried out to identify the differences.

Page 289: f3 Course Notes

14: CONTROL ACCOUNTS

289

7 Double Entry Summary for Chapter 14 7.1 Contra entry adjustment:

7.2 Adjustment to record settlement discounts allowed to customers:

7.3 Adjustment to record settlement discounts received from suppliers:

Dr Payables ledger control account (SOFP) Cr Receivables ledger control account (SOFP)

Dr Discounts allowed (I/S) Cr Receivables ledger control account (SOFP)

Dr Payables ledger control account (SOFP) Cr Discounts received (I/S)

Page 290: f3 Course Notes

14: CONTROL ACCOUNTS

290

Chapter Summary

Control accounts

Trade discounts

Reconciliations

Contra entries Discounts allowed and received

Returns, credit notes, refunds and over payments

Settlement discounts

Receivables ledger control account Payables ledger control account

Receivables ledger Payables ledger

• The RLCA and the RL and the PLCA and the PL are showing the same information and so the balances should reconcile

• RLCA: The total owed by all credit customers at a particular point in time.

• PLCA: Total owed to all credit suppliers at a particular point in time

• RL: a list of the amounts owed by each individual credit customer at a particular point in time

• PL: a list of the amounts owed to each individual credit supplier at a particular point in time

'Where a business has a customer which is also a supplier' • A contra will always be for the lower of

the two amounts and will always reduce both receivable and payables:

Dr PLCA Cr RLCA • The memorandum ledgers must also

be updated for the contra entry

• If a customer returns goods having already paid for them or over pays an invoice they will show a credit balance on their account

• The business may issue the customer with a credit note which they can use to pay for future purchases or the customer may request a refund

• Both the control accounts and the memorandum ledgers must be updated for these entries

• Discounts allowed are offered by a business to their customer (an expense)

• Discounts received are received by a business from their supplier (sundry income)

• Given at the time of sale/purchase • For example: bulk buying discounts • Never appear in the financial

statements Sales tax considerations

• Offered as an incentive to settle a debt early

• For example: 3% discount if settled within 10 days

• May or may not be taken • Sales and purchases are

recorded after trade discounts but before settlement discounts • Sales tax is calculated after all discounts, regardless of

whether they are taken or not • The rate of sales tax will be provided in the exam question

Page 291: f3 Course Notes

291

Chapter 14: Questions

Page 292: f3 Course Notes

14: QUESTIONS

292

Data for Questions 14.1 and 14.2 Womble & Sons have an accounting year ended 31 July 20X8. At that date the balance on the receivables ledger control account was $130,000, but the total of the individual accounts in the receivables ledger came to $127,240.

Upon investigation the following facts were discovered:

(i) The sales day book total for week 22 had been overcast by $600.

(ii) A credit balance of $420 on Orinoco’s account had been incorrectly treated as a debit entry when listing the receivables ledger.

(iii) A contra of $3,000 has been entered in Bungo’s account in the receivables ledger but no other entry had been made.

14.1 The adjusted balance on the receivables ledger control account is:

A $125,560

B $126,400

C $127,240

D $129,400 (2 marks)

14.2 The adjusted balance on the receivables ledger is:

A $125,560

B $126,400

C $127,240

D $129,400 (2 marks)

14.3 A page of the sales day book is undercast by $250. The journal necessary to correct the error is:

A Debit trade receivables $500, Credit sales $500

B Debit sales $500, Credit trade receivables $500

C Debit trade receivables $250, Credit sales $250

D Debit sales $250, Credit trade receivables $250 (2 marks)

14.4 Winn Co has opening trade payables of $24,183 and closing trade payables of $34,665. Purchases for the period totalled $254,192 ($31,590 relating to cash purchases).

What were total payments recorded in the payables ledger for the period? $ (2 marks)

14.5 The double entry to record a discount granted by a supplier is:

A Debit trade payables, Credit discounts allowed

B Debit trade payables, Credit discounts received

C Debit discounts received, Credit trade payables

D Debit trade payables, Credit purchases (2 marks)

Page 293: f3 Course Notes

14: QUESTIONS

293

14.6 The following receivables ledger reconciliation has been prepared by the bookkeeper of Julian Co as at 31 October 20X7:

$ Total per listing of receivables ledger balances 26,170 Debit balance omitted 1,740 Credit listed as debit (1,220) Unexplained difference 300 Balance per control account 26,990 Which of the following errors could have produced the ‘unexplained difference’?

A A refund of $300 was omitted from the receivables ledger.

B The sales day book for October was undercast by $300.

C The trade receivables column of the cash receipts book was overcast by $300.

D A payables ledger contra of $300 was not entered in the memorandum records. (2 marks)

14.7 Justin has attempted to write up his own nominal ledger but is very confused about debits and credits. He realises he has made some mistakes and has asked you to correct the following receivables ledger control account:

Receivables ledger control account

$ $ Balance b/d 12,460 Cash sales 4,430 Sales on credit 15,520 Cheques from credit customers 11,650 Purchase ledger contra 1,600 Discounts allowed 890

Balance c/d 12,610 29,580 29,580

The opening balance is correct. What should the closing balance be?

A $9,410

B $13,840

C $15,620

D $17,040 (2 marks)

14.8 Which of the following is not a valid reason for a credit balance on a customer's account in the receivables ledger?

A Over payment

B Cheque dishonoured by bank

C Returned goods credited to account (1 mark)

Page 294: f3 Course Notes

14: QUESTIONS

294

14.9 During April a company receives an invoice for $12,000 relating to goods bought on credit. These purchases qualify for a 5% trade discount which has not yet been taken into account. The company also sells goods with a list price of $20,000. A 6% trade discount is to be offered on these goods. Sales tax is applicable to all items and is at 15%. Sales tax is not included in the above amounts. If there is no opening balance on the sales tax account at the beginning of April, what is the closing balance at the end of April?

A $1,110 Cr

B $1,110 Dr

C $1,200 Cr

D $1,200 Dr (2 marks)

14.10 The following transactions were recorded in a company’s books during one week of its trading year: $ Trade purchases (at list price) 4,500 Sales on credit (at list price) 6,000 Purchase of a van 10,460 A trade discount of $300 was given on the sales. All figures are given exclusive of sales tax at 15%.

If the balance on the sales tax account was $2,165 credit at the beginning of the week, what is the balance at the end of the week? $ (2 marks)

14.11 Thomas

Thomas is a sole trader. He has been reading a book on basic bookkeeping but his grasp of the subject is weak.

He has produced the following receivables ledger control account but is not sure whether his closing balance is correct.

Receivables ledger control account $ $ Balance b/d 1.1.X6 12,240 Cheques received from customers 74,730 Discounts allowed 2,165 Cheques dishonoured by customers 425 Cash paid to customers with Irrecoverable debts 470 credit balances 180 Allowance for receivables 1,470 Sales 71,250 Cash received from customers 870 Returns inwards 2,250 Purchase ledger contras 230 Balance c/d 31.12.X6 10,350 88,315 88,315 Balance b/d 1.1.X7 10,350 Required

Produce a corrected receivables ledger control account.

Page 295: f3 Course Notes

14: QUESTIONS

295

14.12 Duff

On 31 December 20X7 the balance on Duff’s receivables ledger control account was $1,070, but the receivables ledger balances totalled only $890. You ascertain the following:

(1) The sales day book was overcast by $100 on 1 December 20X7. (2) Receivables ledger balances totalling $70 had been omitted from the list. (3) A contra entry of $20 had been made between the payables ledger and receivables ledger

accounts of Jones & Co, but no other entry had been made. (4) The only posting made in respect of sales on 15 December 20X7, $50 in total, had been to

individual ledger accounts. (5) $60 worth of goods had been returned by Smith Co in November; this had been recorded only in

the control account. (6) The ledger account balance of Davis & Co had been listed as $90, but was in fact $190.

Required

Prepare a reconciliation between the receivables ledger control account and the receivables ledger.

Page 296: f3 Course Notes

14: QUESTIONS

296

Page 297: f3 Course Notes

297

Chapter 14: Answers

Page 298: f3 Course Notes

14: ANSWERS

298

14.1 B Receivables ledger control account $ $ Balance b/d 130,000 SDB overcast 600 Contra 3,000 ∴balance c/d 126,400 130,000 130,000

14.2 B Receivables Ledger $ Balance per list of balances 127,240 Credit balance treated as a debit (840) (2 × $420) 126,400

14.3 C

14.4 $212,120 Trade payables $ $ Bal b/d 24,183 ∴ Payments 212,120 Bal c/d 34,665 Purchases ($254,192 – $31,590) 222,602 246,785 246,785

Bal b/d 34,665

14.5 B

14.6 A B – Day book total has no effect on the receivables ledger, where individual invoice amounts will be entered.

C – Cash book total has no effect on receivables ledger.

D – If a contra had been omitted, the receivables ledger total would have to be reduced by $300.

14.7 B Receivables ledger control account $ $ Balance b/d 12,460 Purchase ledger contra 1,600 Sales on credit 15,520 Cheques from credit customers 11,650 Discounts allowed 890 Balance c/d 13,840 27,980 27,980 Note: Cash sales are not recorded in the control account

14.8 B This would leave a debit balance as the original debt would be reinstated.

Page 299: f3 Course Notes

14: ANSWERS

299

14.9 A $ Input sales tax $12,000 × 95% × 15% 1,710 Output sales tax $20,000 × 94% × 15% (2,820) Closing balance on sales tax account at end of April (1,110) Cr

14.10 $776 Cr

$2,165 + [15% × ($6,000 – $300)] – [15% × $4,500] – [15% × $10,460] = $776

14.11 Thomas Receivables ledger control account $ $

Balance b/d 12,240 Cash – cheque dishonoured 425 Cheques 74,730 Cash – credit balances 180 Irrecoverable debts 470 Sales 71,250 Cash received 870 Discounts allowed 2,165 Returns inwards 2,250 Purchase ledger contras 230

Balance c/d 3,380 84,095 84,095 14.12 Duff

Receivables ledger control account

$ $ Balance b/d 1,070 SDB Overcast (1) 100 Sales 15.12.X7 (4) 50 Purchase ledger contra (3) 20 Amended balance c/d 1,000 1,120 1,120 Balance b/d 1,000

Receivables ledger reconciliation statement

$ Total balance per receivables ledger 890 $ $ Adjustments: + – Balances omitted (2) 70 Goods returned (5) 60 Balance understated (6) 100 170 60 110 Amended total 1,000

Page 300: f3 Course Notes

14: ANSWERS

300

END OF CHAPTER

Page 301: f3 Course Notes

301

15

Syllabus Guide Detailed Outcomes Having studied this chapter you will be able to:

• Understand the purpose of bank reconciliations.

• Identify the main reasons for differences between the cash book and the bank statement.

• Correct cash book errors and/or omissions.

• Prepare bank reconciliation statements and identify the bank balance to be reported in the final accounts.

• Derive bank statement and cash book balances from given information.

Exam Context Exam questions are likely to ask you to perform calculations to correct a bank reconciliation. Alternatively they may ask you to state whether differences between the cash book and the bank statement should be adjusted in the cash book or in the reconciliation statement.

Qualification Context This chapter covers a topic which is only examined in Paper F3.

Bank reconciliations

Page 302: f3 Course Notes

15: BANK RECONCILIATIONS

302

Overview

Cash book balance Bank statement balance

Errors by the bank Timing differences

Bank reconciliations

Differences

Errors by the business

Page 303: f3 Course Notes

15: BANK RECONCILIATIONS

303

1 Introduction 1.1 This chapter is designed to enable you to explain and apply the approach to identifying and

correcting errors through the use of bank reconciliations.

1.2 The cash book is used to record the detailed transactions of receipts and payments into and out of the bank account. These are then posted to the nominal ledger periodically using double entry. At the end of each accounting period, the balance on the cash book should equal the balance in the nominal ledger cash account.

1.3 Bank statements provide an independent record of the balance on the bank account but this balance is unlikely to agree exactly to the cash book balance – therefore a reconciliation is required.

Differences between the cash book balance and the bank statement 1.4 Differences essentially occur for three reasons:

(a) Timing differences: (i) unrecorded lodgements (money paid into the bank by the business but not

yet appearing as a receipt on bank statement) (ii) outstanding/unpresented cheques (cheques paid out by business which

have not yet appeared on bank statement). (b) Errors by the business (i.e. in the cash book):

(i) omissions, such as: standing orders direct debits bank charges interest

(ii) transposition errors (iii) casting errors

(c) Errors by the bank.

A word of warning 1.5

In the books of the business: POSITIVE BANK BALANCE = ASSET = DEBIT NEGATIVE BANK BALANCE (OVERDRAFT) = LIABILITY = CREDIT But from the bank’s point of view: POSITIVE BALANCE = LIABILITY = CREDIT (the bank owes you your money) NEGATIVE BALANCE (OVERDRAFT) = ASSET = DEBIT (you owe the bank ∴ this is an asset for the bank)

Page 304: f3 Course Notes

15: BANK RECONCILIATIONS

304

2 Preparing a bank reconciliation

Procedures 2.1 (a) Compare the bank statement to the cash account and tick off all items which agree.

(b) Remaining items must represent timing differences or errors – decide which!

Example of how to set out a bank reconciliation 2.2 Cash account

$ $ Balance b/d X Dishonoured cheque X Bank charges X Standing orders X Under cast error in balance b/d X Direct debits X Balance c/d X X X

$ Balance per bank statement X plus unrecorded lodgements X less outstanding cheques (X) plus/less bank errors X/(X) Balance per adjusted cash account X

Practical tips 2.3 (a) On reconciliation, put overdrafts and payments in brackets.

(b) It is the corrected cash account balance which is shown on the statement of financial position. This figure will be the recalculated 'Balance c/d' on the cash account (or the total at the end of the reconciliation statement – which should be identical!).

Page 305: f3 Course Notes

15: BANK RECONCILIATIONS

305

Lecture example 1 Preparation question The cash account of Graham showed a debit balance of $204 on 31 March 20X8. A comparison with the bank statements revealed the following. $ (1) Cheques drawn but not presented 3,168 (2) Amounts paid into the bank but not credited 723 (3) Entries in the bank statements not recorded in the cash account (i) Standing order payments 35 (ii) Interest on bank deposit account 18 (iii) Bank charges 14 (4) Balance on the bank statement at 31 March 20X8 2,618 Required Make any necessary adjustments to the cash book balance and complete the bank reconciliation statement as at 31 March 20X8.

Solution Adjustment of cash book balance Cash account

$ $

Bank reconciliation statement

$ Balance per bank statement 31 March 20X8 Unrecorded lodgements Outstanding cheques Balance per cash account at 31 March 20X8

Page 306: f3 Course Notes

15: BANK RECONCILIATIONS

306

Lecture example 2 Exam standard for 2 marks Whilst preparing a bank reconciliation statement at 31 December. The following items caused a difference between the bank statement balance and the cash book balance. (1) Bank interest charged to the account in error (2) Direct debit for $500 for insurance (3) Bank charges of $70 (4) Cheque paid to a supplier on 29 December (5) Receipt from a trade receivable by electronic transfer Required Which of these items will result in an adjustment to the balance per the bank statement? A 2, 3, and 5 B 1 and 4 C 1, 4, and 5 D 1, 3 and 5

Solution

Page 307: f3 Course Notes

15: BANK RECONCILIATIONS

307

3 Chapter summary Section Topic Summary 1 Introduction A business maintains a cash book to tell it how much

cash it has at a particular point in time. It should reconcile this balance to the bank statement in order to ensure the cash book information is accurate. Differences between the cash book balance and the bank statement balance will arise for three reasons: timing differences, errors by the business and errors by the bank.

2 Preparing a bank reconciliation

The bank reconciliation is produced by checking all of the items on the bank statement to the cash book to ensure that they have all been recorded. Any items not in the cash book will then need to be recorded and the cash book updated. The balance per the bank statement must then be adjusted for any timing differences (unrecorded lodgements and outstanding cheques) or errors by the bank.

Quick Quiz

Page 308: f3 Course Notes

15: BANK RECONCILIATIONS

308

Chapter Summary

Cash book balance Bank statement balance

Errors by the bank Timing differences

Bank reconciliations

Differences

Errors by the business

• Business's record of the amount of cash held by the business at any point in time

• Bank's record of the amount of cash held by the business at any point in time

• Items shown in the cash book but not currently on the bank statement

• Examples: - unrecorded lodgements - outstanding cheques • Adjust bank statement balance

• Items on the bank statement which have been omitted from the cash book

• Examples: - bank charges - direct debits • Adjust in the cash book

• Examples: - cheque incorrectly debited to the

business's account - lodgement incorrectly credited to

the business's account • Adjust bank statement balance

Page 309: f3 Course Notes

309

Chapter 15: Questions

Page 310: f3 Course Notes

15: QUESTIONS

310

Data for Questions 15.1 and 15.2 In the books of Ted Co the bank account shows a balance overdrawn of $6,530 as at 31 December 20X8. On comparing the bank statements with the cash book the following items are discovered: (i) Bank charges of $100 and overdraft interest of $50 have been omitted. (ii) Cheques received from customers totalling $1,900 have not yet been cleared by the bank. (iii) Cheques drawn in favour of suppliers amounting to $2,300 are outstanding at the year end. (iv) A credit transfer from a customer of $2,000 was not recorded. (v) A direct debit to a supplier of $1,000 was omitted.

15.1 What figure will be shown in the statement of financial position as at 31 December 20X8 for ‘bank overdraft’?

A $5,480 B $5,680 C $6,130 D $7,380 (2 marks)

15.2 Assuming that the above items are all that is required to reconcile the cash book balance to the balance per the bank statement, what balance did the bank statement show as at 31 December 20X8?

A $5,280 overdrawn B $6,080 overdrawn C $7,780 overdrawn D $8,580 overdrawn (2 marks)

15.3 Rectify

A summary of the cash book of Rectify Co for the year to 31 May 20X5 is as follows: Cash Book

$ $ Opening balance b/d 805 Payments 146,203 Receipts 145,720 Closing balance c/d 322 146,525 146,525 After some investigation of the cash book and vouchers you discover that: (1) bank charges of $143 shown on the bank statement have not yet been entered in the cash book; (2) a cheque drawn for $98 has been entered in the cash book as $89, and another drawn at $230

has been entered as a receipt; (3) a cheque received from a customer for $180 has been returned by the bank marked ‘refer to

drawer’, but it has not yet been written back in the cash book; (4) an error of transposition has occurred in that the opening balance of the cash book should have

been brought down at $850; (5) cheques paid to suppliers totalling $630 have not yet been presented at the bank, whilst payments

in to the bank of $580 on 31 May 20X5 have not yet been credited to the company’s account; (6) a cheque for $82 has been debited to the company’s account in error by the bank; (7) the company owes $430 to the electricity board; (8) standing orders appearing on the bank statement have not yet been entered in the cash book:

(i) interest for the half year to 31 March on a loan of $20,000 at 11% pa; (ii) hire purchase repayments on the managing director’s car – 12 months at $55 per month; (iii) dividend received on a trade investment – $1,147;

Page 311: f3 Course Notes

15: QUESTIONS

311

(9) a page of the receipts side of the cash book has been undercast by $200; (10) the bank statement shows a balance overdrawn of $870.

Required

Prepare a bank reconciliation as at 31 May 20X5.

Page 312: f3 Course Notes

15: QUESTIONS

312

Page 313: f3 Course Notes

313

Chapter 15: Answers

Page 314: f3 Course Notes

15: ANSWERS

314

15.1 B $ Unadjusted cash book balance (6,530) Corrections to cash book (i) interest (100 + 50) (150) (iv) unrecorded cash received 2,000 (v) payment to supplier omitted (1,000) Adjusted cash book balance (5,680)

15.2 A $ Adjusted cash book balance (5,680) Less: unrecorded lodgements (1,900) Add: outstanding cheques 2,300 Balance per bank statement (5,280)

15.3 Rectify Bank

$

Balance b/d 322

Error in opening balance (4) (850 – 805) 45

Dividend received (8iii) 1,147

Undercast (9) 200

Balance c/d 838

2,552

$

Bank charges (1) 143

Cheque drawn entered as $89 (2) 9 (98 – 89)

Cheque drawn entered as receipt (2) 460 (2 × $230)

Cheque returned written back (3) 180

Loan interest (8i) 1,100

HP repayments (8ii) 660 2,552

Bank reconciliation statement as at 31 May 20X5

$ Balance per bank statement (870) O/D

Add: lodgements not yet credited 580

Less: outstanding cheques (630)

Add: cheque wrongly debited by bank 82

Balance per cash book (838) O/D

END OF CHAPTER

Page 315: f3 Course Notes

315

16

Syllabus Guide Detailed Outcomes Having studied this chapter you will be able to:

• Identify the types of error which may occur in bookkeeping systems.

• Identify errors which would be highlighted by the extraction of a trial balance.

• Prepare journal entries to correct errors.

• Calculate and understand the impact of errors on the income statement and statement of financial position.

• Understand the purpose of a suspense account.

• Identify errors leading to the creation of a suspense account.

• Record entries in a suspense account and make journal entries to clear it.

Exam Context Questions on this area are likely to focus on three main areas. You may be asked to identify which explanations could have led to a particular difference or be asked to identify the journal entry to correct an error. You may also need to determine the effect errors may have on the profit figure.

Qualification Context This topic is only tested in Financial Accounting.

Correction of errors

Page 316: f3 Course Notes

16: CORRECTION OF ERRORS

316

Overview

Correction of errors

Adjustments to profit

Types of error

Suspense account

Page 317: f3 Course Notes

16: CORRECTION OF ERRORS

317

1 Introduction 1.1 Chapter 6 showed us how the trial balance was extracted from the ledger accounts and that

it should balance, i.e. total debits should equal total credits.

1.2 If the trial balance doesn't balance then an error has definitely been made and must be corrected.

2 Types of error 2.1 The following errors will still allow the trial balance to balance.

Type of error Example Error of omission

Error of commission

Error of principle

Compensating error

2.2 The trial balance will not balance if total debits do not equal total credits. This could be due to the following: (1) Transposition error (2) An entry has been posted where

(a) debits ≠ credits (b) a debit entry has been posted and no corresponding credit made (or vice

versa)

Section 1

Page 318: f3 Course Notes

16: CORRECTION OF ERRORS

318

(c) two debit entries or two credit entries have been posted.

These errors will be corrected by creating a suspense account and making a journal entry to correct the error.

3 Suspense accounts 3.1 A suspense account is a temporary account. They never appear in the final accounts.

3.2 It is used for two main reasons: (1) To account for a debit or credit entry when the accountant is unsure as to where it

should go (2) To make a preliminary trial balance balance when an error has been detected.

3.3 Steps to clear a suspense account. (1) Determine the original accounting entry which was made. (2) Decide what entry should have been made. (3) Make the required adjustment.

3.4 Illustration W Co sold goods with a value of $2,500 to James, a credit customer. When recording the sale W Co posted the transaction to the correct accounts but made two debit entries. Steps (1) Entry made was:

Dr Trade receivables $2,500 Dr Sales $2,500

(2) Entry should have been: Dr Trade receivables $2,500 Cr Sales $2,500

(3) Correction: The trade receivables entry is correct but sales have been debited by $2,500 when they should have been credited by that amount. The correction is therefore twice the original error: Dr Suspense account $5,000 Cr Sales (2 × $2,500) $5,000 Being: correction of sales posting.

Page 319: f3 Course Notes

16: CORRECTION OF ERRORS

319

Lecture example 1 Technique demonstration Dan, the bookkeeper of Tiffany's, has made his usual mess of things and produced the following attempt at a trial balance for the year ended 30 April 20X7.

$ $ Property, plant and equipment At cost 60,000 Provision for depreciation 31,000 Capital at 1 May 20X6 53,000 Profit for the year 12,300 Inventory, at cost 14,000 Receivables ledger control account 9,600 Payables ledger control account 6,500 Balance at bank 1,640 85,240 102,800 As chief accountant you discover the following: (1) A rent payment of $350 in March 20X7 had been debited in the receivables ledger control

account. (2) Discounts allowed of $500 during the year ended 30 April 20X7 had not been recorded in

the books. (3) No entry had been made for the refund of $2,620 made by cheque to V Woolf in March

20X7, in respect of defective goods returned to Tiffany. V Woolf, who had already paid for the goods, returned them on 28 February 20X7.

(4) The total column of the cash receipts book had been overcast by $1,900 in March 20X7. (5) The purchase of stationery for $1,460 cash in June 20X6 has been correctly entered in the

cash account, but no entry has been made to the appropriate expense account. (6) Capital of $35,000 was recorded incorrectly as $53,000. Required Prepare (a) Journal entries to correct the above errors; (b) A suspense account showing how it is cleared.

Page 320: f3 Course Notes

16: CORRECTION OF ERRORS

320

Solution

Page 321: f3 Course Notes

16: CORRECTION OF ERRORS

321

4 Adjustments to profit 4.1 When errors are corrected they may affect the business' profit for the year figure.

4.2 For example in Lecture example 1, item 5 tells us that a stationery expense of $1,460 has not been recorded in the expense account. The profit for the year figure in the trial balance of $12,300 is therefore too high and needs to be corrected.

4.3 This is done by using a statement of adjustments to profit.

Proforma 4.4

$ $ $ + – Original profit X Adjustment: (a) over depreciation X (b) unrecorded expense X (c) unrecorded sale X X (X) X Adjusted profit X

Lecture example 2 Technique demonstration Required Prepare a statement of adjustments to profit for Lecture example 1.

Solution Statement of adjustments to profit for the year ended 30 April 20X7. Increases Decreases $ $ $ Draft profit Adjustments Revised profit

Page 322: f3 Course Notes

16: CORRECTION OF ERRORS

322

Lecture example 3 Exam standard for 2 marks Z Co's income statement showed a profit of $112,400 for the year ended 30 September 20X7. The following errors were later discovered: (1) Sales returns of $2,700 had been recorded as a new sale. (2) A machine which had been held for two years and had originally cost $15,000 was

depreciated this year using a 33 31 % reducing balance basis. Z Co's policy is to depreciate

machines over four years. Required

What would be the net profit after adjusting for these errors? A $103,250 B $105,750 C $105,950 D $108,450

Solution

Page 323: f3 Course Notes

16: CORRECTION OF ERRORS

323

5 Chapter summary Section Topic Summary

2 Types of error There are four types of errors: errors of omission, commission, principle and compensating errors which will still allow the trial balance to balance. If an error is made however where debits ≠ credits then the trial balance will not balance.

3 Suspense accounts Where the trial balance does not balance a suspense account will be inserted and the errors, once identified, will be corrected via a journal entry. A suspense account should never appear in the final financial statements.

4 Adjustments to profit Where the process of correcting errors requires changes to income and expense accounts the business’ profit will be affected. In this case a statement of adjustments to profit can be prepared to determine the revised profit figure.

Quick Quiz

Page 324: f3 Course Notes

16: CORRECTION OF ERRORS

324

Chapter Summary

Correction of errors

Adjustments to profit

Types of error

Suspense account

• Error of omission: Transaction not recorded • Error of commission: Debits and credits balance but the entry is made to the wrong account For example, an expense is debited to the rent account rather than the electricity account • Error of principle: Debits and credit balance but the entry is made to the wrong 'type' of account For example, machine repairs debited to the machine asset account • Compensating error: Two separate errors are made which correct each other • Transposition error: Here debits ≠ credits and so the trial balance will not balance For example, the posting of a credit sale as Dr Trade receivables $210 Cr Sales $120

'A temporary account which never appears in the financial statements' • Used when: – an accountant is unsure of a double entry – a preliminary trial balance does not

balance • Must be cleared out • Steps: (1) What entry was made? (2) What entry should have been made? (3) What entry is required to correct the

entries?

'When errors are corrected they may affect the business' profit' • Only errors relating to items of income or

expenses will affect profit

Page 325: f3 Course Notes

325

Chapter 16: Questions

Page 326: f3 Course Notes

16: QUESTIONS

326

16.1 Which of the following errors could result in a suspense account being required to balance the trial balance?

A Cash received from receivables treated as a cash sale

B Payments to suppliers of $513 recorded as $531 in the payables ledger

C A supplier’s invoice for $19 recorded as $91 in the purchases account (1 mark)

16.2 Duncan corrected the following errors before producing his final balance sheet. What was the balance on the suspense account before he did this?

(i) Sales day book for March overcast by $63.

(ii) Cash receipts from receivables of $713 posted to the receivables ledger control account as $731.

(iii) Cash received from the issue of $1,000 debentures at par had been posted to a suspense account.

A $982 Dr

B $982 Cr

C $1,018 Dr

D $1,018 Cr (2 marks)

16.3 Russell’s bookkeeper transposed some figures when the week’s cash payments were being posted to the nominal ledger. Payments for staff wages of $125 were posted to the wages account as $152 and payments of $31 for stationery were posted to the stationery expense account as $13. The entry required to correct this is

A Dr stationery $18 Dr suspense $9 Cr wages $27

B Dr wages $27 Dr stationery $18 Cr suspense $45

C Dr wages $27 Cr stationery $18 Cr suspense $9

D Dr suspense $45 Cr wages $27 Cr stationery $18 (2 marks)

16.4 Which of the following errors would cause a trial balance imbalance?

(i) The discounts received column of the cash payments book was overcast. (ii) Cash paid for the purchase of office furniture was debited to the general expenses account. (iii) Returns inwards were included on the credit side of the trial balance.

A (i) only

B (i) and (ii)

C (iii) only

D all of the errors (2 marks)

16.5 If sales of $150 has been wrongly entered on the debit side of the purchases account, but correctly entered in the trade receivables account, the totals on the trial balance would show:

A The debit side to be $150 more than the credit side

B The debit side to be $300 more than the credit side

C The debit side to be $150 less than the credit side

D The debit and credit sides to be equal in value (2 marks)

Page 327: f3 Course Notes

16: QUESTIONS

327

16.6 Platinum Co

Platinum Co, a manufacturer of electrical goods, has just produced its draft accounts for the year ended 30 September 20X7. These show a draft profit of $28,960.

Unfortunately, the accountant has since discovered the following matters which require consideration before the final accounts can be prepared:

(1) Returns outwards to Metals Co in June 20X7 of $490 have been treated as returns inwards in error in the nominal ledger.

(2) R. Silverman, a customer owing $1,850 has gone bankrupt. Full allowance had been made against this amount in Platinum's accounts for the year ended 30 September 20X6.

(3) An item of equipment with a net book value of $6,000 (cost $10,000) was sold for $5,000 in September 20X7. The proceeds were included in cash and credited to the motor expenses account. No other entries were made.

(4) An amount owing from Aluminium Co of $780 was written off in January 20X7. The amount was removed from trade receivables and debited to the sales account.

Required

Calculate the corrected profit for the year ended 30 September 20X7.

Page 328: f3 Course Notes

16: QUESTIONS

328

Page 329: f3 Course Notes

329

Chapter 16: Answers

Page 330: f3 Course Notes

16: ANSWERS

330

16.1 C

16.2 B (i) No suspense account effect (ii) Entry made was Dr Bank $713, Cr RLCA $731 and therefore Dr Suspense $18 (iii) Entry made was Dr Bank $1,000, Cr Suspense $1,000

Original suspense account balance is therefore $18 Dr + $1,000 Cr = $982 Cr

16.3 A

16.4 C

16.5 B

16.6 Platinum Co

(a) Statement of corrected profit for the year ended 30 September 20X7.

$ $ $ Draft profit + – 28,960 (1) Returns outwards ($490 x 2) 980 (2) No effect (3) Disposal of machine ($5,000 + $1,000)

6,000

(4) No effect 980 (6,000) (5,020) Revised profit 23,940

END OF CHAPTER

Page 331: f3 Course Notes

331

17

Syllabus Guide Detailed Outcomes Having studied this chapter you will be able to:

• Prepare extracts of an opening trial balance.

• Prepare journal entries to correct errors.

• Record entries in a suspense account

• Make journal entries to clear a suspense account

• Prepare extracts of a statement of financial position and income statement from given information.

Exam Context This chapter recaps some of the key skills you have learnt in the chapters covered to date. Whilst you will not be asked to produce a statement of financial position or an income statement in the real exam any of the adjustments in this chapter could be tested as an individual question. This chapter will also help you to see how financial accounting fits together.

Qualification Context The skills to produce a statement of financial position and an income statement are tested in detail in the Fundamentals level paper, Financial Reporting (F7).

Home study chapter – Preparation of financial statements for sole traders

Page 332: f3 Course Notes

17: HOME STUDY CHAPTER – PREPARATION OF FINANCIAL STATEMENTS FOR SOLE TRADERS

332

Overview

Suspense account Adjustments

Preparation of financial statements for sole traders

Trial balance

Income Statement and Statement of financial

position

Page 333: f3 Course Notes

17: HOME STUDY CHAPTER – PREPARATION OF FINANCIAL STATEMENTS FOR SOLE TRADERS

333

1 Introduction 1.1 The purpose of this chapter is to recap some of the skills covered in Chapters 1–16.

1.2 You will not be required to answer a question in the format of Lecture example 1 in the exam. However completing this exercise will revise your understanding of topics covered so far and enable you to see the end product – a business' transactions ordered into a set of financial statements.

Lecture example 1 Technique demonstation You have been given the information below and asked to prepare the accounts of Mugg for the year ended 31 December 20X7. Trial balance as at 31 December 20X7.

Dr Cr $ $ Capital account at 1 January 20X7 2,377 Rent 500 Inventories 1 January 20X7 510 Electricity 240 Insurance 120 Wages 1,634 Trade receivables 672 Sales 15,542 Repairs 635 Purchases 9,876 Discounts received 129 Drawings 1,200 Petty cash 5 Bank 762 Motor vehicles at cost 1,740 Furniture and fixtures at cost 830 Accumulated depreciation at 1 January 20X7

– Motor vehicles 435 – Furniture and fixtures 166

Travel and entertaining 192 Trade payables 700 Suspense account 433 19,349 19,349 The following information is also available: (1) Closing inventories, valued at cost, amounts to $647; (2) Mugg has drawn $10 a month and these drawings have been charged to wages; (3) Depreciation is to be provided at 25% on cost on motor vehicles, and 20% on cost on

furniture and fixtures; (4) Bad debts totalling $37 are to be written off;

Page 334: f3 Course Notes

17: HOME STUDY CHAPTER – PREPARATION OF FINANCIAL STATEMENTS FOR SOLE TRADERS

334

(5) $180 received from a credit customer was correctly entered in the trade receivables account and credited to the bank account;

(6) Mugg has taken goods from inventories for his own use. When purchased by his business these goods cost $63 and they would have been sold for $91;

(7) The annual rental of the business premises is $600, and $180 paid for electricity in August 20X7 covers the 12 months to 30 June 20X8;

(8) Discounts allowed of $73 have only been recorded in the trade receivables account. Required

(a) Prepare journal entries to record items (1) – (8). (b) Clear the suspense account. (c) Produce an income statement for the year ended 31 December 20X7 and a statement of

financial position as at that date.

Solution (a) Journals

(1) (2) (3) (4)

Page 335: f3 Course Notes

17: HOME STUDY CHAPTER – PREPARATION OF FINANCIAL STATEMENTS FOR SOLE TRADERS

335

(5) (6) (7) (8)

(b) Suspense account

$ $

Page 336: f3 Course Notes

17: HOME STUDY CHAPTER – PREPARATION OF FINANCIAL STATEMENTS FOR SOLE TRADERS

336

(c) Mugg Income statement for the year ended 31 December 20X7 $ $ Sales Less: cost of sales Opening inventories Purchases Less: closing inventories Gross profit Discounts received Less: expenses: Rent Electricity Insurance Wages Repairs Depreciation Travel and entertaining Bad debts Discounts allowed Profit for the period

Mugg Statement of financial position as at 31 December 20X7

Cost Accumulated depreciation

NBV

$ $ $ Non-current assets Motor vehicles Furniture and fixtures Current assets Inventories Trade receivables Prepayments Cash and bank balances Capital Capital as at 1 January 20X7 Profit for the period Less: drawings Current liabilities Trade payables Accruals

Page 337: f3 Course Notes

17: HOME STUDY CHAPTER – PREPARATION OF FINANCIAL STATEMENTS FOR SOLE TRADERS

337

2 Chapter summary Section Topic Summary 1 Introduction The statement of financial position and the income

statement are the end product produced by a business. All the business’ transactions need to be categorised into the books of prime entry and posted to the nominal ledger. The trial balance is then extracted and some adjustments may need to be made before the financial statements are drawn up. You will not have to produce a statement of financial position or income statement; however this chapter should reinforce your understanding of Chapters 1 – 16.

Quick Quiz

Page 338: f3 Course Notes

17: HOME STUDY CHAPTER – PREPARATION OF FINANCIAL STATEMENTS FOR SOLE TRADERS

338

Chapter Summary

Suspense account Adjustments

Preparation of financial statements for sole traders

Trial balance

Income Statement and Statement of financial

position

'A list of the balances brought down on each ledger account'

• For example: - closing inventories - depreciation - bad and doubtful

debts - accruals and

prepayments - correction of errors

'A temporary account which never appears in the financial statements'

Page 339: f3 Course Notes

339

Chapter 17: Question

Page 340: f3 Course Notes

17: QUESTION

340

17.1 Drawings are an expense of the business. Is this statement true or false?

A True

B False (1 mark)

Page 341: f3 Course Notes

341

Chapter 17: Answer

Page 342: f3 Course Notes

17: ANSWER

342

17.1 B

END OF CHAPTER

Page 343: f3 Course Notes

343

18

Syllabus Guide Detailed Outcomes Having studied this chapter you will be able to:

• Understand and apply techniques used in incomplete record situations:

(i) use of accounting equation (ii) use of ledger accounts to calculate missing figures (iii) use of cash and/or bank summaries (iv) use of profit percentages to calculate missing figures

Exam Context Questions on this chapter will require you to identify missing figures, for example sales, closing inventories and drawings. The Pilot Paper included two questions asking you to derive the value of closing inventories using information about the gross profit margin earned by the business.

Qualification Context This topic is only tested in Financial Accounting.

Business Context Some sole traders do not keep very detailed accounting records. They still however need to produce accounts so they know how their business is performing and also how much tax to pay to the tax authorities. The preparation of accounts from incomplete records can generate a lot of income for smaller accountancy practices.

Incomplete records

Page 344: f3 Course Notes

18: INCOMPLETE RECORDS

344

Overview

Incomplete records

Cost structures

Techniques for solving incomplete records

Margin Mark-up

Derive missing figures from given information

Purchases Drawings Sales Inventory

Page 345: f3 Course Notes

18: INCOMPLETE RECORDS

345

1 Issue 1.1 Individuals running small businesses such as a newsagent or greengrocer may not keep all

of the accounting records we have studied or have a detailed understanding of double entry bookkeeping.

1.2 They still need to know how the business is performing and so will produce financial statements. If some necessary information isn't maintained by the business, it will need to be derived from other available information.

2 Cost structures 2.1 Cost structure information is usually expressed in one of two ways, either as a margin or a

mark-up. (a) Margin: here gross profit is expressed as a percentage of sales, for example a

margin of 25% gives: Sales 100% Cost of sales 75% Gross profit 25%

(b) Mark-up: here gross profit is expressed as a percentage of cost of sales, for example a mark-up of 35% gives: Sales 135% Cost of sales 100% Gross profit 35%

2.2 Remember that:

Cost of sales = opening inventories + purchases – closing inventories

Lecture example 1 Preparation question W Co has on average a profit margin of 40%. In 20X7 sales total $476,000. Required

What is cost of sales? $ Workings

Page 346: f3 Course Notes

18: INCOMPLETE RECORDS

346

Lecture example 2 Preparation question Y Co operates with a standard mark-up of 30% and has the following information available for 20X7. $ Sales 221,000 Opening inventories 43,000 Closing inventories 47,500 Required What is the value for purchases in 20X7? $

Workings

Lecture example 3 Exam standard for 2 marks On 1 January 20X7 J Co had inventory of $620,000. Sales for the month amounted to $985,000 and purchases were $700,000. At the end of January a fire in the warehouse destroyed some inventory items. The owners salvaged inventory valued at $180,000. J Co operates with a mark up of 25%. What is the cost of inventory destroyed in the fire? A $335,000 B $352,000 C $401,250 D $532,000

Solution

Page 347: f3 Course Notes

18: INCOMPLETE RECORDS

347

3 Other techniques for solving incomplete records

Lecture example 4 Preparation question A Co has recorded the following details relating to trade payables: $ Balance at 1.1.X7 38,450 31.12.X7 43,825 Cash paid from till 430 Payments from bank 167,224 Required

Based on the information above what was the value of purchases made during the year?

$ Workings

Trade payables

$

$

Page 348: f3 Course Notes

18: INCOMPLETE RECORDS

348

Lecture example 5 Preparation question B Co maintains a cash float of $50. In 20X7, all receipts from credit customers were banked, after the following payments from the till had been made: $ General expenses 4,500 Drawings 6,250 Total bankings in the year amounted to $28,454, and opening and closing trade receivables were $1,447 and $1,928 respectively. Required

Based on the information above what was the value of sales made during the year? $ Workings

Cash $ $

Trade receivables $ $

Page 349: f3 Course Notes

18: INCOMPLETE RECORDS

349

Lecture example 6 Exam standard for 2 marks Bob owns and manages B Co although he does not keep detailed accounting records. All of Bob's sales are for cash. He pays certain expenses from his till and then banks the remaining funds. Bob maintains a $1,000 float and operates with a margin of 20%. He has provided you with the following information. $ Purchases of goods (on credit) 20,000 Wages for clerical assistant (per week; there are 52 weeks in the year) 100 Stationery 500 Electricity 1,200 Bankings 12,800 Opening inventories 2,000 Closing inventories 3,000 Bob is unsure of the level of drawings taken during the year but estimates they were between $60 and $90 per week. Required What were Bob's drawings during the year? $

Workings

Page 350: f3 Course Notes

18: INCOMPLETE RECORDS

350

4 Goods drawn by proprietor 4.1 The owners of the business may at times take goods or cash from the business for their own

use. We have seen these before as drawings. In incomplete records questions these drawings need to be included. Cash drawings Dr Drawings Cr Cash Goods taken for own use Dr Drawings Cr Purchases

These are recorded at the cost to the business not at sale price. They are taken out of purchases and not recorded against inventories. Note: If you are using a trade payables T account to calculate purchases remember to adjust purchases for any goods taken by proprietor.

Example 4.2 During the year ended 31 December 20X7, Peter Albert, a sole trader, carried out the

following transactions: $ Sales (40 units @ $100) 4,000 Purchases (45 units @ $60) 2,700 His inventories (at cost) were: $ 1 January 20X7 (5 units @ $60) 300 31 December 20X7 (8 units @ $60) 480 During the year he had withdrawn two units for his own use. Firstly, ignoring the drawings, an outline trading account would appear as follows:

$ $ Sales 4,000 Cost of sales

Opening inventories 300 Purchases 2,700 3,000 Less: closing inventories (480)

2,520 Gross profit 1,480 How should the drawings of goods be treated?

Page 351: f3 Course Notes

18: INCOMPLETE RECORDS

351

It should be fairly obvious that the debit entry will be to drawings on the balance sheet, but what about the credit entry? It will not, as you might initially think, go to inventories (because these goods were not in hand at the year end so they are not included in the value of $480) but rather to purchases (as this is where they will have been previously recorded). In the trading account, this credit entry is often shown as a separate deduction from cost of sales, i.e.:

$ $ Sales 4,000 Cost of sales

Opening inventories 300 Purchases 2,700 Less: goods drawn by proprietor 2 units @ $60 (120) 2,880 Less: closing inventories (480)

2,400 Gross profit 1,600

Points to note 4.3 (a) Drawings of goods are recorded at cost.

(b) Gross profit figure now makes sense, i.e. profit of $40 per unit × 40 units sold.

Page 352: f3 Course Notes

18: INCOMPLETE RECORDS

352

5 Chapter summary Section Topic Summary

1 Issue Not all businesses keep proper accounting records, however all businesses need to know how much profit they have made in a particular year so that they can pay the relevant amount of tax over to the tax authorities. Where a business does not have sufficient records to produce financial statements they need to piece together the missing information.

2 Cost structures A margin is where a business expresses gross profit as a percentage of sales. A mark-up is where gross profit is expressed as a percentage of cost of sales.

3 Other techniques for solving incomplete records

Other techniques that may be used in solving incomplete records questions involve putting all known information in to one or two ledger accounts and balancing off to derive the required information. These questions are essentially a test of double entry skills.

4 Goods drawn by proprietor

A business is a separate entity from its owner which means that any monies or goods taken out of the business for personal use must be classified as drawings. Drawings of goods are always recorded at cost.

6 Double Entry Summary for Chapter 18 6.1 Adjustment to record cash drawings:

6.2 Adjustment to record drawings of goods:

Dr Drawings (SOFP) Cr Cash (SOFP)

Dr Drawings (SOFP) Cr Purchases (I/S)

Quick Quiz

Page 353: f3 Course Notes

18: INCOMPLETE RECORDS

353

Chapter Summary

Incomplete records

Cost structures

Techniques for solving incomplete records

Margin Mark-up

Derive missing figures from given information

Purchases Drawings Sales Inventory

'Gross profit expressed as a percentage of sales' • For example, a 20% margin: Sales $1.00 COS $0.80 Gross profit $0.20

'Gross profit expressed as a percentage of cost of sales' • For example, a 25% mark-up: Sales $1.25 COS $1.00 Gross profit $0.25

• Derive sales figure by: - putting all known

information into a trade receivables T account

- using cost structure information to work from cost of sales back to sales

• Derive purchases figure by: - putting all known information

into a trade payables T account

- using cost structure information to derive purchases as part of the cost of sales figure

• Put all known information into a cash T account

• If the question states that drawings were between $50 and $80 per week this indicates that drawings are the missing figure

• Derive using cost structure information

• Drawings of inventory are always valued at their cost and not their selling price

Page 354: f3 Course Notes

18: INCOMPLETE RECORDS

354

Page 355: f3 Course Notes

355

Chapter 18: Questions

Page 356: f3 Course Notes

18: QUESTIONS

356

18.1 If a business has sales of $6,000 and a margin of 20%, what is the gross profit? $

(1 mark)

18.2 A trader has budgeted sales for the coming year of $300,000. He achieves a constant mark-up of 25% on cost. He plans to reduce his inventory level by $14,000 over the year. How much will his purchases for the year be?

A $211,000

B $239,000

C $226,000

D $254,000 (2 marks)

18.3 A business has opening inventories of $273 and makes purchases during the year of $2,781. The proprietor removes goods costing $87 for his own use. The business achieves a constant mark-up of 20% on cost and records sales for the year of $3,360.

What is the cost of closing inventories? $ (2 marks)

18.4 Jethro sold goods for $157,470 during the year ended 31 October 20X7. Inventories at that date were valued at $8,920 more than at the previous year end. Jethro prices his goods to give a mark-up of 45%.

What was the total value of purchases in the year ended 31 October 20X7?

A $77,689

B $95,529

C $99,680

D $117,520 (2 marks)

Page 357: f3 Course Notes

357

Chapter 18: Answers

Page 358: f3 Course Notes

18: ANSWERS

358

18.1 $1,200

$6,000 × 0.2 = $1,200

18.2 C $

Cost of sales = $300,000 125100× 240,000

Less: decrease in inventories (14,000) 226,000

18.3 $167 Cost of sales $

$3,360 120100× 2,800

Less: opening inventories (273) purchases (2,781 – 87) (2,694) ∴ closing inventories 167

18.4 D Cost of sales = $157,470 x 100/145 = $108,600 ∴ purchases = $108,600 + $8,920 = $117,520

END OF CHAPTER

Page 359: f3 Course Notes

359

19

Syllabus Guide Detailed Outcomes Having studied this chapter you will be able to:

• Understand and identify the typical content of a partnership agreement, including profit sharing terms.

• Understand the nature of capital accounts, current accounts and division of profits.

• Calculate and record the partners' shares of profits/losses.

• Account for guaranteed minimum profit shares.

• Calculate and record

(i) partners' drawings (ii) interest on drawings (iii) interest on capital (iv) partner salaries

• Prepare an extract of a capital account and a current account.

• Prepare extracts of the income statement, including division of profit, and statement of financial position of a partnership.

• Define goodwill in relation to partnership accounts and identify the factors leading to the creation of goodwill.

• Calculate the value of goodwill from given information.

Exam Context Questions on this topic are likely to require you to calculate a partner's profit share. This may include dealing with partners' salaries, interest on capital and drawings and loan interest. You may also need to allocate goodwill to partners when a new partner is admitted.

Qualification Context Partnerships are only examined in this paper.

Business Context Many individuals set up business as a sole trader – as they expand they need new finance. One way of obtaining this is to go into partnership with someone else. That other person could provide some of the finance needed. They may also bring new ideas to the table. Becoming a partnership will mean that the sole trader will share some of their risk but they will also need to share their profits too!

It is always recommended that a partnership agreement is drawn up to retain a legal record of how the partnership will operate.

Partnerships

Page 360: f3 Course Notes

19: PARTNERSHIPS

360

Overview

Partnerships

Partnership agreements Appropriation account

Capital accounts Current accounts

Loans Guaranteed minimum profit share

Goodwill

Other issues

Page 361: f3 Course Notes

19: PARTNERSHIPS

361

1 Definition 1.1 Partnership: The relationship which exists between two or more persons carrying on

a business with a view to profit.

1.2 Partnerships are similar to sole traders. With a sole trader the owner will run the business and any profits belong to him. The sole trader also bears the risk that the business may not be successful. In a partnership, the owners (partners) run the business together and share profits and risk.

1.3 Most partnerships have unlimited liability which means the partners are personally liable for the debts of the business. Liability is also joint and several so if one partner cannot meet the partnership's obligations the other partners must make up any shortfall.

1.4 Limited liability partnerships (LLPs) exist nowadays to limit partner liability. These are outside the scope of the F3 syllabus.

2 Partnership agreements The partners will need to agree the terms under which the partnership will operate, and decide, for example how much capital each partner will contribute and what share of profits they will be entitled to. This is done by way of a partnership agreement which usually covers the following areas:

2.1 Area Consideration Capital • how much each partner pays in

• whether a "Fixed Capital" level is specified Profit sharing ratio (PSR)

• allocation of profit • more to senior partners? • equal shares? • guaranteed minimum profit share?

Salaries • whether or not partners are entitled to salaries • it is an appropriation of profit • it is not an income statement expense

Interest on capital • whether or not allowed • paid on capital injected • interest rate

Drawings • may set a limit • may set an interest charge

Page 362: f3 Course Notes

19: PARTNERSHIPS

362

3 Accounting for partnerships 3.1 There are two key differences between accounting for a sole trader and a partnership.

These are illustrated below.

3.2 Income statement Sole trader Partnership $ $ Sales X Sales X Cost of sales (X) Cost of sales (X) Gross profit X Gross profit X Less: expenses (X) Less: expenses (X) Profit for period X Profit for period X

All belongs to sole trader

Shared between partners according to the partnership agreement

Appropriating the profit for the period 3.3 The profit for the period is appropriated (shared out) between the partners according to

their partnership agreement. Steps (1) Allocate the partner salaries (2) Allocate any interest on capital (3) Charge any interest on drawings (4) Allocate remaining profit balance in profit sharing ratio This is done using an appropriation account.

Appropriation account Salaries Partner A X Partner B X Interest on capital Partner A X Partner B X * PSR Partner A X Partner B X X

Profit before appropriation X Interest on drawings Partner A X Partner B X

X

* PSR is always the last entry, splitting the residual profit after all other allocations

Page 363: f3 Course Notes

19: PARTNERSHIPS

363

3.4 Statement of financial position Sole trader Partnership $ $ Proprietor’s interest Capital accounts Capital X Partner A X Profit X Partner B X Less: drawings (X) X X Current accounts Partner A X Partner B X X

Amount owed back to the owner by the business

Amount owed back to the partners by the business

3.5 Capital accounts These represent the capital invested in the business by each individual partner. The balances in these accounts will remain relatively static. The capital account can be shown as one T account subdivided into columns. For example, if Partner A contributed $5,000 and Partner B $8,000, the capital account would show.

Capital account

Ptnr A

Ptnr B

Ptnr A

Ptnr B

$ $ $ $ Bal b/d 5,000 8,000

3.6 Current accounts These record each partner's day to day transactions with the business. The main entries in the current account will be the partners’ appropriation of profits (salary, interest on capital and profit share) less drawings they have taken from the business and any interest charged on those drawings.

Current account

Ptnr A

Ptnr B

Ptnr A

Ptnr B

$ $ $ $ Drawings 2,900 970 Bal b/d 1,000 1,500 Interest on drawings 100 30 Salaries 1,500 – Interest on capital 500 800 Bal c/d 4,000 5,000 Profit share 4,000 3,700 7,000 6,000 7,000 6,000

Page 364: f3 Course Notes

19: PARTNERSHIPS

364

Lecture example 1 Preparation question (a) On 1 January 20X4 Tick, Cast and Balance entered into partnership together as chartered

certified accountants. They agreed that Balance would receive a salary of $15,000 p.a., they would all be allowed interest on capital of 12% p.a. and they would share profits in the ratio: Tick five tenths, Cast three tenths, Balance two tenths. They paid in the following capital amounts: Tick $50,000 Cast $30,000 Balance $20,000 In the year to 31 December 20X4 their profit for the period was $50,000. During the year they had made drawings in cash as follows: 30.6.20X4 Tick $6,000 30.9.20X4 Cast $4,000 31.12.20X4 Balance $8,800 Required

(i) Write up their capital accounts in columnar form. (ii) Write up the appropriation account. (iii) Write up their current accounts in columnar form. (iv) Show the partners' balances on the statement of financial position.

Solution

(i) Capital Accounts

Tick Cast Balance $ $ $

Tick Cast Balance $ $ $

Page 365: f3 Course Notes

19: PARTNERSHIPS

365

(ii) Appropriation account for the year ended 31 December 20X4

$

$

(iii) Current accounts

Tick Cast Balance $ $ $

Tick Cast Balance $ $ $

Page 366: f3 Course Notes

19: PARTNERSHIPS

366

(iv) Tick, Cast and Balance Statement of financial position as at 31 December 20X4 (extract) $ $

Capital accounts

Tick

Cast

Balance

Current accounts

Tick

Cast

Balance

(b) What would your answer be to (ii) and (iii) if the agreement had also provided for interest to be charged on drawings at the rate of 10% p.a.? (ii)

Appropriation account for the year ended 31 December 20X4 $

$

Page 367: f3 Course Notes

19: PARTNERSHIPS

367

(iii) Current accounts

Tick Cast Balance $ $ $

Tick Cast Balance $ $ $

Page 368: f3 Course Notes

19: PARTNERSHIPS

368

4 Guaranteed minimum profit share 4.1 It may be that the partnership agreement specifies that one or more partners must receive a

minimum share of profits.

4.2 If when the appropriation of profits is made this level is exceeded, there is nothing further to do.

4.3 If, however, there is a shortfall then this will be made up by the remaining partners in their profit sharing ratio.

4.4 Illustration A, B and C are in partnership and share profits in the ratio 2:2:1. The partnership made a profit for the year of $50,000. A and B each receive a salary of $12,000. Interest due on the partners' capital is $2,000, $1,700 and $1,500 respectively. No interest is charged on drawings. C has a guaranteed minimum profit share of $7,000.

A B C Total $ $ $ $ Salaries 12,000 12,000 – 24,000 Interest on capital 2,000 1,700 1,500 5,200 Profit share (2:2:1) 8,320 8,320 4,160 20,800 Subtotal 22,320 22,020 5,660 50,000 Guaranteed minimum profit share shortfall (2:2) (670) (670) 1,340 – 21,650 21,350 7,000 50,000

5 Loans 5.1 Unlike sole traders, a partner can make a loan to the partnership.

Reasons for making a loan 5.2 (a) Partnership may be short of funds.

(b) Partner is unwilling to tie cash up for long period. (c) Partner wants to earn interest. (d) Partner retires but partnership does not have enough cash to buy out his share.

Accounting treatment 5.3 The loan is shown as a non-current liability on the statement of financial position and not

in the partner's capital account.

5.4 The interest incurred on the loan is shown as an expense in the income statement (just like bank interest). It will need to be deducted from the profit figure before any appropriation is made if it has not already been accounted for.

Page 369: f3 Course Notes

19: PARTNERSHIPS

369

5.5 If the loan interest has not been paid by the end of the year, the liability will be shown in the relevant partner’s current account. The double entry would be:

Dr Loan interest expense (I/S) Cr Current account (SOFP)

Lecture example 2 Exam standard for 2 marks X, Y and Z are in partnership sharing profits in the ratio 6:3:1. Y made a loan of $10,000 to the partnership on 1 July 20X7. The loan carries interest at 12% but this has not yet been accounted for. X and Z receive salaries of $15,000 and $8,000 respectively and interest due on capital to each partner is $400. The profit for the year to 31 December 20X7 was $67,000. Required What is the amount of profit appropriated to each partner for the year ended 31 December 20X7? $

Workings

Page 370: f3 Course Notes

19: PARTNERSHIPS

370

6 Changes to the partnership agreement 6.1 Profits are always appropriated according to the partnership agreement; therefore if the

terms of the agreement change during the period this will affect the profit appropriation.

6.2 Always use the old partnership agreement to appropriate the profits for the first part of the year and the new partnership agreement for the latter part of the year.

6.3 Assume profits accrue evenly unless the question specifies otherwise.

Lecture example 3 Exam standard for 2 marks Melanie, Sarah and Angela are in partnership, compiling their accounts for the year to 31 December each year. The partnership agreement states the following: Until 30 June 20X3

Annual salaries Sarah $40,000 Angela $20,000 Profit sharing ratio Melanie: Sarah: Angela is 60:20:20: From 1 July 20X3 Salaries to be discontinued, profit sharing ratio to be: 50:30:20 The profit for the year ended 31 December 20X3 was $400,000 before charging partners' salaries, accruing evenly through the year and after charging an expense of $40,000, which it was agreed related wholly to the first six months of the year. Required How should the profit for the year be divided among the partners? Use a separate page for your workings.

Melanie Sarah Angela

$ $ $ A 182,000 130,000 88,000 B 200,000 116,000 84,000 C 198,000 118,000 88,000 D 180,000 132,000 88,000

Page 371: f3 Course Notes

19: PARTNERSHIPS

371

7 Changes to the partnership – goodwill 7.1 When a partner retires from the partnership or a new partner is admitted to the partnership it

is usual for the partners to value the business.

7.2 It is likely that over time the value of items such as property, plant and equipment will increase over their net book value. However, hopefully the business will also have built up a good reputation and a loyal customer base and the business itself will be worth more than its individual assets.

7.3 The worth of a business over and above its individual assets is called goodwill.

7.4 When a partner retires it is important that he is paid a sum that represents not just the money he invested but also his share of the extra value created in the business, i.e. his share of goodwill. Illustration A partnership's statement of financial position may have the following assets and liabilities:

$'000 Property 200 Other assets 120 320 Liabilities (100) 220

The business therefore has a 'book value' of $220,000. However when the partnership was valued as a whole it was judged to be worth $350,000: an increase of $130,000. $80,000 of this increase was believed to be attributable to the increase in the value of the property, but the other $50,000 was due to the business's superb regional reputation and wealthy customer base. This $50,000 is known as goodwill. Goodwill is therefore added to the partners' accounts according to the existing or old profit sharing ratio.

7.5 Similarly, when a new partner joins, he will pay in a sum of money (capital). It is important that the original partners value the partnership so they know its worth and can determine how much the partner should contribute.

7.6 Goodwill is an extremely subjective figure and so it is not left in the partnership's statement of financial position, but is removed. This is done using the new profit sharing ratio.

Section 2.8-2.10

Page 372: f3 Course Notes

19: PARTNERSHIPS

372

Lecture example 4 Exam standard

A and B have been in partnership for many years and have built up a business with a balance sheet value of $150,000. The partnership now needs new finance to continue to grow and a new partner, C, is to be admitted on 31 March at which time the value of A and B's partnership is $210,000. A and B contributed capital of $80,000 and $97,000 respectively when the partnership was created. They share profits in the ratio 5:7. C will pay capital of $100,000 into the partnership after which profits will be shared equally between the three partners. The partnership does not include goodwill in its balance sheet. Required Calculate the balance of each of the partner’s capital accounts after C has been admitted to the partnership.

Solution

Page 373: f3 Course Notes

19: PARTNERSHIPS

373

8 Chapter summary Section Topic Summary 1 Definition A partnership exists where two or more persons are

carrying on a business with a view to a profit. 2 Partnership

agreements The purpose of a partnership agreement is to specify how the partnership operates in terms of the how much capital the partners pay in and whether they are paid interest on capital; whether they are entitled to a salary; whether interest is charged on drawings and the profit sharing ratio.

3 Accounting for partnerships

The final profit for the year is appropriated according to the partnership agreement. The profit for the year will be allocated to cover the partners’ salaries and any interest on capital/ drawings. The balance will then be shared between the partners according to the profit sharing ratio. Note that partners’ salaries are not an expense of the business but an appropriation of profit. Capital accounts represent the capital paid in by each partner and are generally static. Current accounts record the partners’ day to day transactions with the business.

4 Guaranteed minimum profit share

The partnership agreement may stipulate that a particular partner is guaranteed a minimum profit share. Where this is the case any shortfall will generally be made up by the other partners according to their profit sharing ratio.

5 Loans Should a partner make a loan to the business then this is shown as a liability on the statement of financial position and the interest is recorded as an expense in the income statement. Any interest owed to the partner at the end of the year will be credited to their current account.

6 Changes to the partnership agreement

The partners may change the partnership agreement during the year. If this is the case then profits are assumed to accrue evenly during the year. The first period of profits should be appropriated using the old partnership agreement and the second period using the new partnership agreement.

Quick Quiz

Page 374: f3 Course Notes

19: PARTNERSHIPS

374

Section Topic Summary 7 Changes to the

partnership - goodwill Whenever a new partner is admitted or an existing partner retires the partnership will be valued. The worth of the partnership over and above the statement of financial position valued is called goodwill. This is allocated to the partners according to their profit sharing ratio.

Page 375: f3 Course Notes

19: PARTNERSHIPS

375

Chapter Summary

Partnerships

Partnership agreements Appropriation account

Capital accounts Current accounts

Loans Guaranteed minimum profit share

Goodwill

• Capital and interest on capital • Drawings and interest on drawings • Salaries • Profit sharing ratio (PSR)

'The sharing out of the profit for the period according to the partnership agreement' • Steps: (1) Allocate partner salaries (2) Allocate interest on capital (3) Charge interest on drawings (4) Allocate residual profit using PSR • Check to see whether any loan interest due to

partners has been deducted from the profit for the period figure

• Record the capital invested in the business by each partner

• Tend to remain static

• Record the partners' day to day transactions with the business

• Entries include: - salaries - interest on capital - profit share - drawings - interest on drawings

• The partnership agreement may specify that one or more partners receive a minimum share of profits

• Any shortfall is made up by the remaining partners according to their PSR

• Show as a non-current liability in the balance sheet

• Interest is an expense in the income statement

• Any unpaid loan interest is shown as a credit to the partner's current account

'The value of a business as a whole over and above the value of its individual assets' • Goodwill is allocated to the

partners using the old PSR • It is not retained in the

balance sheet but is removed using the new PSR

Other issues

Page 376: f3 Course Notes

19: PARTNERSHIPS

376

Page 377: f3 Course Notes

377

Chapter 19: Questions

Page 378: f3 Course Notes

19: QUESTIONS

378

Data for Questions 19.1 and 19.2 John, Paul and David have been in a partnership for one year providing advice on landscape gardening. The partnership agreement provides for the following:

• a salary to John of $5,000 per annum. • interest on capital balances @ 10% per annum. • interest on drawings @ 5% per annum. • profit sharing ratio of 2:2:1 respectively.

The capital accounts as at 31 March 20X3 showed the following balances:

John $30,000 Paul $25,000 David $20,000

The partners made the following drawings during the year:

John $6,000 on 30 June 20X2 Paul $2,000 on 31 December 20X2 David $1,500 on 31 March 20X3

On 30 September 20X2 Paul lent the partnership $100,000. Interest (which has not been included in the accounts) is to be charged at 4% per annum. Loan interest is to be included in the current account. The profit for the year ended 31 March 20X3 was $40,000.

19.1 What is John’s share of the residual profits? $ (2 marks)

19.2 What will be the balance on Paul’s current account at 31 March 20X3? $ (2 marks)

19.3 Which of the following is not true?

A Partners are jointly and severally liable

B Partner salaries are an appropriation of profits

C Interest on drawings is an appropriation of profits (1 mark)

Page 379: f3 Course Notes

19: QUESTIONS

379

19.4 A, B and C

A, B and C are in partnership, agreeing to share profits in the ratio of 4:2:1. They have also agreed to allow interest on capital at 8% per annum, a salary to C of $5,000 per annum, and to charge interest on drawings made in advance of the year end at a rate of 10% per annum.

The statement of financial position as at 30 June 20X8 disclosed the following:

$ $ Capital accounts A 50,000 B 30,000 C 10,000 90,000

Current accounts A 2,630 B 521 C (418) 2,733 Loan account (5% interest) A 15,000 107,733

Drawings were: A $6,400, B $3,100, C $2,000, with all sums being withdrawn on 1 July 20X8.

Profit for the year to 30 June 20X9 was $24,750, before charging interest on A's loan. The partnership made a payment to A for loan interest on 29 June 20X9 but has not recorded this in its books.

Required

Prepare the current accounts and the appropriation account for the partners as at 30 June 20X9.

Page 380: f3 Course Notes

19: QUESTIONS

380

Page 381: f3 Course Notes

381

Chapter 19: Answers

Page 382: f3 Course Notes

19: ANSWERS

382

19.1 $10,300

$ Profit per accounts 40,000 Less loan interest 4% × $100,000 × 12

6 (2,000) 38,000

John Paul David Total $ $ $ $ Salary 5,000 – – 5,000 Interest on capital 3,000 2,500 2,000 7,500 Interest on drawings (225) (25) – (250) PSR (2:2:1) 10,300 10,300 5,150 25,750 (β) Total profit 18,075 12,775 7,150 38,000

19.2 $12,775

Current account – Paul Drawings 2,000 Interest on capital 2,500 Interest on drawings 25 Loan interest 2,000 c/d 12,775 PSR 10,300 14,800 14,800 b/d 12,775

19.3 C Interest on drawings increases available profits to share and is therefore not an appropriation of profit.

Partner salaries are an appropriation of profit, not an expense.

19.4 A, B and C

Current accounts A B C

$ $ $ A B C $ $ $

Balance b/d 418 Drawings 6,400 3,100 2,000 Interest on drawings 640 310 200 Balance c/d 6,990 3,211 5,032 14,030 6,621 7,650

Balance b/d 2,630 521 Salary 5,000 Interest on capital 4,000 2,400 800 Share of profit 7,400 3,700 1,850 14,030 6,621 7,650 Balance b/d 6,990 3,211 5,032

Page 383: f3 Course Notes

19: ANSWERS

383

Appropriation account $ $ Salary C 5,000 Profit (W1) 24,000 Interest A 4,000 Interest A 640 on capital B 2,400 on drawings B 310 C 800 C 200 7,200 1,150 Share of profits in PSR A(4/7) 7,400 B(2/7) 3,700 C(1/7) 1,850 β 12,950 25,150 25,150 (W1) Profit 24,750 Interest on loan (750) 24,000

Page 384: f3 Course Notes

19: ANSWERS

384

END OF CHAPTER

Page 385: f3 Course Notes

385

20

Syllabus Guide Detailed Outcomes Having studied this chapter you will be able to:

• Understand the capital structure of a limited liability company including ordinary shares, preference shares and loan notes.

• Record movements in the share capital and share premium accounts.

• Define a bonus issue and a rights issue, their advantages and disadvantages and show how they are recorded in the balance sheet.

• Identify and record the other reserves which may appear in the company statement of financial position.

• Record dividends in ledger accounts and the financial statements.

• Calculate and record finance costs in ledger accounts and the financial statements.

Exam Context Questions on this chapter are likely to focus on the calculation of share capital movements (new issues, bonus issues and rights issues), dividends and finance costs and their associated journal entries. You may also see a question comparing a sole trader and a limited company as was included in the Pilot Paper.

Qualification Context The knowledge covered in this chapter is developed further in the Fundamentals level paper Financial Reporting (F7). This paper looks in more detail at whether shares and borrowings should be classified as debt or equity and also at how they should be valued. The area of income taxes is also extended to include adjustments for deferred tax as well as current tax.

Business Context When a company is seeking to raise finance it will evaluate its current financing structure and gearing levels before deciding how to secure additional funds. It will also consider the degree of risk attached to each method of financing and will weigh up the cost in terms of interest payments versus future dividends. A company will also receive tax relief on its interest payments (but not on dividends) and so the tax implications will form part of the final decision.

Introduction to company accounting

Page 386: f3 Course Notes

20: INTRODUCTION TO COMPANY ACCOUNTING

386

Overview

Dividends

Reserves Income taxes

Introduction to company accounting

Long term borrowings

Finance costs

Shares

Accounting treatment

Rights issue Issue at a premium Bonus issue

Page 387: f3 Course Notes

20: INTRODUCTION TO COMPANY ACCOUNTING

387

1 Introduction 1.1 We have seen how financial statements are produced for sole traders and partnerships.

These accounts are not subject to any specific regulation and so there is some flexibility as to how they are presented.

1.2 Companies use exactly the same bookkeeping process as sole traders and partnerships; however, the financial statements they produce are subject to regulation and must follow a prescribed format. Many of the differences are due to the terminology used by company financial statements.

2 Proforma financial statements 2.1 Income statement for the year ended 31 March 20X7

$'000 Revenue X Cost of sales (X) Gross profit X Other income X Distribution costs (X) Administrative expenses (X) Finance costs (X) Profit before tax X Income tax expense (X) Profit for the year X

Page 388: f3 Course Notes

20: INTRODUCTION TO COMPANY ACCOUNTING

388

2.2 Statement of financial position as at 31 March 20X7 $'000 ASSETS Non-current assets Property, plant and equipment X Other intangible assets X X Current assets Inventories X Trade receivables X Other current assets X Cash and cash equivalents X X Total assets X EQUITY AND LIABILITIES Equity Share capital X Share premium account X Revaluation reserve X Retained earnings X X Non-current liabilities Long term borrowings X Long term provisions X X Current liabilities Trade payables X Short term borrowings X Current tax payable X Short term provisions X Total equity and liabilities X

2.3 These proformas will be covered in more detail in Chapter 21.

3 Share capital

Share capital 3.1 It is necessary to be able to distinguish between the following types of share capital:

(a) Authorised share capital – maximum number of shares the company may issue. (b) Issued share capital – number of shares actually issued to shareholders. (c) Called up share capital – the amount of issued share capital the company has

asked shareholders to pay for to date. (d) Paid up share capital – amount of called up share capital which has been paid

for.

Section 2.3

Page 389: f3 Course Notes

20: INTRODUCTION TO COMPANY ACCOUNTING

389

Types of shares 3.2

Ordinary share Preference share • Equity share • Fixed rate of dividends (eg 7%

preference share) • Ordinary shareholders – own business • Receive dividend in priority to ordinary

shareholders • Usually have voting rights • On winding up, receive capital in priority • No right to a dividend, receive what

directors decide to pay

4 Share capital: accounting treatment

Issue of new shares 4.1 Rab Co started business on 1 January 20X6 issuing 100,000 ordinary shares of 50c each

for 50c per share. The initial statement of financial position would be: $ Cash 50,000 Share capital – 50c ordinary shares 50,000

Issue of new shares at a premium 4.2 Where shares are issued for more than their nominal value, the excess must be credited to

a share premium account.

Lecture example 1 Preparation question On 1 June 20X6 Rab Co issued a further 200,000 ordinary shares of 50c each for 80c per share. Required

Show how this issue of shares would be accounted for and what the statement of financial position would look like immediately after the issue.

Page 390: f3 Course Notes

20: INTRODUCTION TO COMPANY ACCOUNTING

390

Solution Dr Cr $ $ Dr Cash Cr Share capital Cr Share premium account Rab Co statement of financial position (extract) as at 1 June 20X6 Equity

$ Share capital – 50c ordinary shares Share premium account

Bonus issue (capitalisation issue) 4.3 This is used when a company wishes to increase its share capital without needing to raise

additional finance by issuing new shares. Any reserve may be used including the share premium account.

4.4 Advantages Disadvantage • Bonus issue can be made from the share

premium account which has few other uses

• The rationale for a bonus issue is not always understood by shareholders

• Will allow the share price to fall (without disadvantaging shareholder wealth) to make the company's shares more affordable to new investors

• Shareholders will now own more shares and could sell part of their holding

4.5 A bonus issue is always done at nominal value.

Page 391: f3 Course Notes

20: INTRODUCTION TO COMPANY ACCOUNTING

391

Lecture example 2 Preparation question Rab Co Statement of financial position (extract)

$ Share capital – 50c ordinary shares 150,000 Share premium account 60,000 Retained earnings 200,000 410,000 Several years later Rab Co is to make a bonus issue on a 1 for 4 basis. Required

Show how this issue of shares would be accounted for and prepare the statement of financial position of Rab Co immediately after the issue.

Solution

Dr Cr $ $ Dr Share premium account Cr Share capital

Rab Co Statement of financial position (extract)

$ Share capital – 50c ordinary shares Share premium account Retained earnings

Rights issue 4.6 (a) A rights issue is an issue of shares for cash (unlike a bonus issue) to existing

shareholders. (b) ‘Rights’ are offered to the existing shareholders who can sell them if they wish.

Page 392: f3 Course Notes

20: INTRODUCTION TO COMPANY ACCOUNTING

392

4.7 Advantages Disadvantages • More cost effective way for the

company to raise finance than a fresh issue to the public

• Lack of shareholder interest may reflect badly on the company

• A more time efficient way to issue shares

• Unwelcome predators may try to acquire shares where not all rights are taken up

• If all rights are taken up shareholders will maintain their existing percentage shareholding

• Effect on future dividend policy as company will have issued more shares under the rights issue than it would have under a fresh issue to the public

Lecture example 3 Preparation question One year later, Rab Co is to make a rights issue on a 1 for 5 basis. The rights price is $1.50. All shareholders take up their rights. The following statement of financial position extract shows the position before the issue Rab Co Statement of financial position (extract) $ Share capital – 50c ordinary shares 187,500 Share premium account 22,500 Retained earnings 230,000 440,000 Required

Show how this issue of shares would be accounted for and prepare the statement of financial position of Rab Co immediately following the issue.

Solution Dr Cr $ $ Dr Cash Cr Share capital Cr Share premium account Rab Co Statement of financial position (extract) $ Share capital – 50c ordinary shares Share premium account Retained earnings

Page 393: f3 Course Notes

20: INTRODUCTION TO COMPANY ACCOUNTING

393

5 Reserves 5.1 The following reserves are commonly found in limited liability company accounts.

(a) The share premium account: (i) Typical permitted uses:

(1) to issue bonus shares; (2) to write off share issue expenses.

(b) The revaluation reserve (see Chapter 9): (c) Other reserves:

as designated by the individual company, for example a 'general reserve'. (d) Retained earnings:

cumulative undistributed profits less any losses.

6 Dividends

Definition

6.1 Dividends – a sharing out/appropriation of retained earnings to owners/shareholders.

Illustration 6.2 Suppose a company with 1,000 ordinary $1 shares in issue made a profit of $500 in its first

year. The company has two choices as to what can be done with this profit: (a) distribute it as a dividend to the shareholders; (b) retain it in the business. If this company decides to pay a dividend of 10c per share and retain the remaining profits, the financial statements would appear as follows: Income statement for the year ended 31 December 20X7

$ Profit for the period 500 Statement of financial position as at 31 December 20X7 (extract) $ Share capital – $1 shares 1,000 Retained earnings (500 – 100) 400 1,400

6.3 Dividends are charged directly to retained earnings as they are an appropriation of profits earned to date. They are not an expense of the income statement.

6.4 The double entry is: Dr Retained earnings (SOFP) Cr Dividends payable (SOFP)

Page 394: f3 Course Notes

20: INTRODUCTION TO COMPANY ACCOUNTING

394

6.5 A company may pay dividends in two stages: (a) Interim (mid year) (b) Final (end year) In reality the directors will wait until they know the company's full year profit before declaring the final dividend. The final dividend will only be accounted for in the current year if it is declared before the year end. Otherwise it will be disclosed in a note to the financial statements (see Chapter 22).

Lecture example 4 Preparation question ABC Co has the following share capital: 100,000 6% $1 preference shares 200,000 50c ordinary shares Retained earnings at the beginning of the year were $125,000. During the year ended 31 December 20X7 it made the following profit:

$ Profit before tax 60,000 Income tax expense 10,000 Profit for the period 50,000 Dividends paid and declared during the year were as follows: Interim dividend paid 5c per share Final dividend declared on 20 January 20X8 10c per share Required

Show the movement in retained earnings for ABC Co for the year ended 31 December 20X7.

Solution $ $ Retained earnings at beginning of year Profit for the period Dividends – Preference – Ordinary Retained earnings at end of year

Page 395: f3 Course Notes

20: INTRODUCTION TO COMPANY ACCOUNTING

395

7 Long term borrowings 7.1 A company may choose to raise finance by issuing shares (equity).

Alternatively it can raise funds by issuing debt.

7.2 One way of raising long term finance is for a company to issue loan notes (also called loan stock or debentures). These loans usually carry a fixed rate of interest and have a pre-determined redemption date, for example, $50,000 10% debentures 2012. This means the company will pay interest at 10% on the $50,000 borrowed each year. The capital amount of $50,000 will be repaid in 2012.

8 Finance costs 8.1 The interest expense incurred on long term borrowings will be shown as an expense called

'finance costs' in the income statement.

8.2 It will be accounted for as follows: Dr Finance costs (I/S) Cr Bank

9 Income taxes 9.1 Companies must pay income tax on their profits. This tax is payable after the end of the

financial year and so the financial statements will include an accrual for the directors' best estimate of the tax due on the profit for the period.

9.2 The tax is shown as an expense in the income statement and a current liability in the statement of financial position and will be accounted for as follows: Dr Income tax expense (I/S) Cr Current tax payable (SOFP)

9.3 Often the actual amount of tax paid will be different from the amount that was recorded in the financial statements. This over or under provision is simply adjusted in the next financial statements.

Page 396: f3 Course Notes

20: INTRODUCTION TO COMPANY ACCOUNTING

396

Lecture example 5 Preparation question

Lauren Ltd has a year end of December. When preparing its financial statements for the year ended 31 December 20X5, Lauren Ltd estimated that its income tax payable would be $62,000. Lauren Ltd settled this tax liability on 30 September 20X6, paying $65,000. The tax estimate for the year ended 31 December 20X6 is $43,000. Required (1) Record the tax entries for the years ended 31 December 20X5 and 20X6 in the ledger

accounts. (2) Prepare the tax note which relates to the income statement for the year ended 31 December

20X6.

Solution (1) Income tax expense (I/S) $ $ Current tax payable (SOFP) $ $ (2) Tax note for the year ended 31 December 20X6

Page 397: f3 Course Notes

20: INTRODUCTION TO COMPANY ACCOUNTING

397

10 Comparison The following table shows a comparison between a sole trader and a limited liability company.

Sole trader Company Ownership The proprietor owns the business. There are often a large number of

owners, who are called shareholders or members.

Liability The proprietor has unlimited legal liability regarding the business.

Members/shareholders have limited liability. This means that they are only liable to the extent of their investment in the business.

Legal status The business and the proprietor share legal identity (although the business is a separate business entity for reporting purposes).

A company is a separate legal entity.

Management The proprietor usually owns and manages the business.

Members/shareholders do not usually manage the business, but appoint a Board of Directors to run the company on their behalf.

Profits The proprietor takes 'drawings' out of the business. Any cash amounts taken as a salary are not an expense of the business but drawings.

Members/shareholders receive profits in the form of dividends. The remainder of the profits are retained in the company. The directors receive a salary from the company and this is an expense in the income statement.

Taxation Business profits are taxed in the hands of the proprietor, using individual's tax rates.

Income tax is paid on the company profits.

Statement of financial position

The middle of the statement of financial position is split into 'opening capital', 'profits' and 'drawings'.

The middle of the statement of financial position is split into 'share capital' and 'reserves'.

Legal requirements

There are no legal requirements specific to a sole trader.

There are extensive legal requirements governing limited companies.

Other The business is closed to outside investors.

Investors can invest in a company.

Page 398: f3 Course Notes

20: INTRODUCTION TO COMPANY ACCOUNTING

398

11 Chapter summary Section Topic Summary

1 Introduction Companies use the same method of bookkeeping to record transactions. There are however some differences in the terminology and the formats used.

2 Proforma financial statements

The format in which companies must produce their financial statements is prescribed by the accounting standard IAS 1.

3 Share capital An entity may issue two main types of shares. Ordinary or equity shareholders have voting rights and therefore have control over the company. Preference shareholders are really just providers of finance to the business and have limited rights.

4 Share capital: accounting treatment

In a limited liability company the shareholders own the business. A company may raise finance by issuing new share capital. Where shares are issued at a premium to their nominal value, the premium is recorded in the share premium account. A bonus issue is where the company issues shares for no cash consideration. With a rights issue, shares are issued for cash but the price charged is slightly lower than the current market price.

5 Reserves A company may have several different types of reserve such as a share premium account, a revaluation reserve and retained earnings.

6 Dividends Shareholders may receive a dividend as a return on their investment; these are accounted for as a deduction to retained earnings.

7 Long term borrowings A company may also raise finance by issuing debt such as loan notes or debentures.

8 Finance costs It will have to pay interest on any debt that it issues and this will be shown as 'finance costs' in the income statement.

9 Current tax Companies pay corporation tax on their profits. 10 Comparison Sole traders and partnerships are very similar in their

nature whilst companies are quite different. You must ensure that you are happy with both the differences and similarities.

Quick Quiz

Page 399: f3 Course Notes

20: INTRODUCTION TO COMPANY ACCOUNTING

399

12 Double Entry Summary for Chapter 20 12.1 Adjustment to record dividends:

12.2 Adjustment to record finance costs:

12.3 Adjustment to record the income tax expense:

Dr Retained earnings (SOFP) Cr Dividends payable (SOFP)

Dr Finance costs (I/S) Cr Bank (SOFP)

Dr Income tax expense (I/S) Cr Current tax payable (SOFP)

Page 400: f3 Course Notes

20: INTRODUCTION TO COMPANY ACCOUNTING

400

Chapter Summary

Reserves Income taxes

Introduction to company accounting

Long term borrowings

Finance costs

Shares

Accounting treatment

Rights issue Issue at a premium Bonus issue

• Interest is shown as a finance cost in the income statement

• Share premium account • Revaluation reserve • Other reserves • Retained earnings: - cumulative undistributed profits

• Debt finance, for example: - debentures - loan notes

• Expense in the income statement and a liability at the year end

• Dr Income tax expense Cr Current tax payable • Any under/ over provision is adjusted in

the next year's financial statements

Four main types of share capital: - authorised - issued - called up - paid up

• Ordinary shares: - equity share - voting rights - no right to a dividend • Preference shares: - receive a fixed rate of dividends - no voting rights

'When shares are issued at a premium to their nominal value, the excess should be credited to the share premium account' • Dr Cash Cr Share capital Cr Share premium

'Shares are issued for no cash consideration' • Always done at nominal value • Dr Reserves (SPA) Cr Share capital • Advantages - enables company to use the share premium

account - price of shares will fall making them more

affordable to new investors • Disadvantage: - rationale is not always understood by

shareholders

'Shares issued to existing shareholders for cash' • Issued at rights price which is below current

market price • Dr Cash Cr Share capital Cr Share premium account • Advantages - cost effective way for company to raise finance - if all rights are taken up shareholders will

maintain their percentage shareholding • Disadvantages - 'bad press' for the company if all rights are not

taken up - effect on future dividend policy

• Dividends on ordinary shares and most preference shares are an appropriation of profits and are debited to retained earnings • Dividends relating to redeemable preference shares are a finance cost in the income statement

Dividends

Page 401: f3 Course Notes

401

Chapter 20: Questions

Page 402: f3 Course Notes

20: QUESTIONS

402

20.1 A company has an authorised share capital of 1,000,000 50c ordinary shares and an issued share capital of 800,000 50c ordinary shares.

If an ordinary dividend of 5% is declared what is the amount payable to shareholders? $

(1 mark)

20.2 If a shareholder in a limited liability company sells his shares to another private investor, for less than he paid for them, the share capital of the company will

A Remain unchanged

B Increase by the nominal value of the shares

C Increase by the amount received for the shares

D Decrease by the nominal value of the shares (2 marks)

20.3 A company’s issued share capital consists of $100,000 in 6% $1 preference shares and $50,000 in 50c ordinary shares. The directors wish to pay an ordinary dividend for the year of 5 cents per share. What is the company’s total dividend for the year?

A $8,500

B $11,000

C $5,000

D $17,000 (2 marks)

Page 403: f3 Course Notes

403

Chapter 20: Answers

Page 404: f3 Course Notes

20: ANSWERS

404

20.1 $20,000

5% × (800,000 × 0.50)

20.2 A

20.3 B ($100,000 × 6%) + ($50,000 × 2 × 5c)

END OF CHAPTER

Page 405: f3 Course Notes

405

21

Syllabus Guide Detailed Outcomes Having studied this chapter you will be able to: • Recognise how the accounting equation and business entity convention underlie the statement of financial

position. • Understand the nature of reserves and report them in a company statement of financial position. • Prepare extracts of a statement of financial position from given information. • Understand why the heading 'retained earnings' appears in a company statement of financial position. • Prepare extracts of an income statement from given information. • Understand how accounting concepts apply to revenue and expenses. • Calculate revenue, cost of sales, gross profit and net profit from given information and disclose items of income

and expenditure in the income statement. • Record income taxes in the income statement of a company. • Understand the inter-relationship between the statement of financial position and income statement. • Identify items requiring separate disclosure on the face of the income statement. • Identify the components of the statement of changes in equity.

Exam Context Whilst you will not be required to produce an entire income statement, statement of financial position or statement of changes in equity you may be asked to calculate individual elements of each statement. A question on the Pilot Paper required you to demonstrate understanding of what was included in the statement of changes in equity.

Qualification Context The topics covered in this chapter are developed further in the Fundamentals level paper Financial Reporting (F7). Here you will need to produce financial statements using the format specified by IAS 1. You will also learn how accounting standards such as IFRS 5 affect the presentation of the financial statements if, for example, a company discontinues part of its operations.

Business Context Financial statements are used by a wide range of user groups to make decisions, for example whether or not to buy shares in a company. Financial statements need to be prepared in a consistent way in order for users to be able to compare different companies. The notes to the accounts will also provide a lot more detail on the headline figures shown in the income statement and statement of financial position.

Preparation of financial statements for companies

Page 406: f3 Course Notes

21: PREPARATION OF FINANCIAL STATEMENTS FOR COMPANIES

406

Overview

Notes to the accounts

Income statement Statement of financial

position

Preparation of financial statementsfor companies

Statement of changes in equity

Page 407: f3 Course Notes

21: PREPARATION OF FINANCIAL STATEMENTS FOR COMPANIES

407

1 Introduction 1.1 As stated in Chapter 20 the financial statements of a limited liability company are subject to

regulation and must follow a prescribed format.

1.2 Much of the prescribed format is determined by IAS 1 (revised). This accounting standard states what should be included in a set of financial statements and how they should be presented. A complete set of financial statements in accordance with IAS 1 (revised) comprises: (a) a statement of financial position (b) an income statement (c) a statement of comprehensive income (d) a statement of changes in equity (e) a statement of cash flows; and (f) notes, comprising a summary of significant accounting policies and other explanatory

notes.

2 Proforma financial statements 2.1 Income statement for the year ended 31 March 20X7

$'000 Revenue X Cost of sales (X) Gross profit X Other income X Distribution costs (X) Administrative expenses (X) Finance costs (X) Profit before tax X Income tax expense (X) Profit for the year X

Page 408: f3 Course Notes

21: PREPARATION OF FINANCIAL STATEMENTS FOR COMPANIES

408

2.2 Statement of financial position as at 31 March 20X7 $'000 ASSETS Non-current assets Property, plant and equipment X Other intangible assets X X Current assets Inventories X Trade receivables X Other current assets X Cash and cash equivalents X X Total assets X EQUITY AND LIABILITIES Equity Share capital X Share premium account X Revaluation reserve X Retained earnings X X Non-current liabilities Long term borrowings X Long term provisions X Current liabilities Trade payables X Short term borrowings X Current tax payable X Short term provisions X Total equity and liabilities X

Page 409: f3 Course Notes

21: PREPARATION OF FINANCIAL STATEMENTS FOR COMPANIES

409

2.3 Illustration Below are the income statement and statement of financial position for Arrow Co for the

year ended 30 September 20X6 Arrow Co Income statement for the year ended 30 September 20X6

$'000 Revenue 12,740 Cost of sales (7,040) Gross profit 5,700 Distribution costs (2,060) Administrative expenses (2,375) Finance costs (72) Profit before tax 1,193 Income tax expense (270) Profit for the year 923 Arrow Co Statement of financial position as at 30 September 20X6

$'000 ASSETS Non-current assets Property, plant and equipment 5,000 5,000 Current assets Inventories 610 Trade receivables 1,000 Cash and cash equivalents 1,170 2,780 Total assets 7,780 EQUITY AND LIABILITIES EQUITY Share capital 1,750 Share premium account 585 Revaluation reserve 1,400 Retained earnings 1,873 5,608 Non-current liabilities Long term borrowings 1,200 1,200 Current liabilities Trade payables 550 Other payables 72 Current tax payable 270 Short term provisions 80 972 Total equity and liabilities 7,780

Page 410: f3 Course Notes

21: PREPARATION OF FINANCIAL STATEMENTS FOR COMPANIES

410

The following information was accounted for when the above financial statements were produced: (1) During the year the company made a rights issue on a 1 for 6 basis. The issue was

fully subscribed and the rights price was $1.27. Prior to the rights issue Arrow Co had 3,000,000 50c ordinary shares in issue.

(2) The property, plant and equipment were revalued by $600,000 during the year. (3) A dividend of $300,000 was paid during the year.

2.4 Statement of comprehensive income Financial statements have always included an income statement and a statement of financial position Following a revision to IAS 1, financial statements should also now include a statement of comprehensive income. This statement shows all of the realised gains and losses from the income statement and the unrealised gains and losses from the statement of financial position in one statement of performance.

Income statement Statement of financial position

Realised Unrealised gains and losses gains and losses

e.g. profit for the year e.g. revaluation gains/losses

Statement of comprehensive income

The statement can be presented in one of two ways:

• as one single statement (proforma 1) • as two separate statements (proforma 2)

Page 411: f3 Course Notes

21: PREPARATION OF FINANCIAL STATEMENTS FOR COMPANIES

411

Proforma 1 – one single statement

Statement of comprehensive income for the year ended 31 March 20X7 20X7 20X6 $’000 $’000 Revenue X X Cost of sales (X) (X) Gross profit X X Other income X X Distribution costs (X) (X) Administrative expenses (X) (X) Finance costs (X) (X) Investment income X X Profit before tax X X Income tax expense (X) (X) Profit for the year X X Other comprehensive income: Gains on property revaluation X X Total comprehensive income for the year X X

Proforma 2 – two separate statements

Income statement for the year ended 31 March 20X7 20X7 20X6 $’000 $’000 Revenue X X Cost of sales (X) (X) Gross profit X X Other income X X Distribution costs (X) (X) Administrative expenses (X) (X) Finance costs (X) (X) Investment income X X Profit before tax X X Income tax expense (X) (X) Profit for the year X X

Statement of comprehensive income for the year ended 31 March 20X7 20X7 20X6 $’000 $’000 Profit for the year X X Other comprehensive income: Gains on property revaluation X X Total comprehensive income for the year X X

Page 412: f3 Course Notes

21: PREPARATION OF FINANCIAL STATEMENTS FOR COMPANIES

412

Lecture example 1 Technique demonstration Using the illustration in Section 2.3, prepare the statement of comprehensive income for the year ended 30 September 20X6: (a) showing the statement as one statement (b) showing the statement as two separate statements.

Solution (a) One single statement

Statement of comprehensive income for the year ended 30 September 20X6 $’000 Revenue 12,740 Cost of sales (7,040) Gross profit 5,700 Distribution costs (2,060) Administrative expenses (2,375) Finance costs (72) Profit before tax 1,193 Income tax expense (270) Profit for the year 923 Other comprehensive income: Gains on property revaluation Total comprehensive income for the year

(b) Two separate statements Income statement for the year ended 30 September 20X6 $’000 Revenue 12,740 Cost of sales (7,040) Gross profit 5,700 Distribution costs (2,060) Administrative expenses (2,375) Finance costs (72) Profit before tax 1,193 Income tax expense (270) Profit for the year 923

Page 413: f3 Course Notes

21: PREPARATION OF FINANCIAL STATEMENTS FOR COMPANIES

413

Statement of comprehensive income for the year ended 30 September 20X6 $’000 Profit for the year 923 Other comprehensive income: Gains on property revaluation

Total comprehensive income for the year

2.5 Statement of changes in equity Proforma Share

capital Share

premium account

Reval- uation

reserve

Retained earnings

Total equity

$’000 $’000 $’000 $’000 $’000 Balance at 31 March 20X6 X X X X X

Changes in accounting policy _ _ _ X X Restated balance X X X X X Issue of share capital X X X Dividends (X) (X) Total comprehensive income _ _ X X X Balance at 31 March 20X7 X X X X X

Lecture example 2 Technique demonstration Arrow had the following equity balances at 1 October 20X5 (the beginning of the year):

$'000 Share capital – 50c ordinary shares 1,500 Share premium account 200 Revaluation reserve 800 Retained earnings 1,250 3,750 Required Using the information from the illustration in Section 2.3, produce a statement of changes in equity for Arrow for the year ended 30 September 20X6.

Page 414: f3 Course Notes

21: PREPARATION OF FINANCIAL STATEMENTS FOR COMPANIES

414

Solution

Share capital

Share premium account

Reval- uation

reserve

Retained earnings

Total equity

$’000 $’000 $’000 $’000 $’000 Balance at 30 September 20X5

Issue of share capital

Dividends

Total comprehensive income

Balance at 30 September 20X6

Working

Page 415: f3 Course Notes

21: PREPARATION OF FINANCIAL STATEMENTS FOR COMPANIES

415

3 Notes to the accounts Notes are included in a set of financial statements to give users extra information. You should be aware of the following notes:

3.1 Property, plant and equipment (Chapter 9) Land and

buildings Machinery Office

equipment Total

$ $ $ $ Net book value at 1 April 20X6 X X X X Additions X X X X Revaluation surplus X – – X Depreciation charge (X) (X) (X) (X) Disposals (X) (X) (X) (X) Net book value at 31 March 20X7 X X X X At 31 March 20X7 Cost or valuation X X X X Accumulated depreciation (X) (X) (X) (X) Net book value X X X X At 31 March 20X6 Cost or valuation X X X X Accumulated depreciation (X) (X) (X) (X) Net book value X X X X

3.2 Intangible non-current assets (Chapter 10) Development

expenditure $ Net book value at 1 April 20X6 X Additions X Amortisation charge (X) Disposals (X) Net book value at 31 March 20X7 X At 31 March 20X7 Cost X Accumulated amortisation (X) Net book value X At 31 March 20X6 Cost X Accumulated amortisation (X) Net book value X

Page 416: f3 Course Notes

21: PREPARATION OF FINANCIAL STATEMENTS FOR COMPANIES

416

3.3 Provisions (Chapter 13) $ At 1 April 20X6 X Increase in period X Released in period (X) At 31 March 20X7 X

3.4 Contingent liabilities (Chapter 13) Unless remote, disclose for each contingent liability: (a) a brief description of its nature; and where practicable (b) an estimate of the financial effect (c) an indication of the uncertainties relating to the amount or timing of any outflow; and (d) the possibility of any reimbursement

3.5 Contingent assets (Chapter 13) Where an inflow of economic benefits is probable, an entity should disclose (a) a brief description of its nature; and where practicable (b) an estimate of the financial effect

3.6 Events after the reporting period (Chapter 22) In respect of non-adjusting events after the reporting period disclose (a) the nature of the event (b) an estimate of its financial effect (or a statement that an estimate cannot be made).

Page 417: f3 Course Notes

21: PREPARATION OF FINANCIAL STATEMENTS FOR COMPANIES

417

4 Chapter summary Section Topic Summary 1 Introduction The financial statements published by a company need

to follow the format prescribed by IAS 1 (revised). 2 Proforma financial

statements You will not be required to produce an income statement or statement of financial position but should be aware of their contents. The statement of comprehensive income is a new performance statement which brings together the realised gains and losses from the income statement and the unrealised gains and losses from the statement of financial position. The statement of changes in equity shows the movements on each of the accounts in the equity section of the statement of financial position in a separate statement.

3 Notes to the accounts The purpose of the notes to the accounts is to provide additional information of key financial statement figures.

Page 418: f3 Course Notes

21: PREPARATION OF FINANCIAL STATEMENTS FOR COMPANIES

418

Chapter Summary

Notes to the accounts

Income statement Statement of financial

position

Preparation of financial statementsfor companies

Statement of changes in equity

'Shows the income and expenses for a period under specific headings' • Points to note: - distribution costs: delivery costs - administrative expenses: general costs

that do not 'fit' under the other captions - finance costs: bank interest,

debenture/loan note interest - income tax expense: estimate of income

tax due on the profits for the period plus/minus any under/over provision in respect of prior periods

'Shows the assets and liabilities of a business at a point in time'

'Explains the movements between the equity section of the statement of financial position at the beginning and the end of the year' • Key components: – issue of share capital – dividends (on ordinary shares) – total comprehensive income for the year

• profit for the year • revaluation surplus on non-current

assets

• Examinable notes: – property, plant and equipment – intangible non-current assets – provisions – contingent liabilities – contingent assets – events after the reporting period

Page 419: f3 Course Notes

419

Chapter 21: Questions

Page 420: f3 Course Notes

21: QUESTIONS

420

21.1 Which of the following items impact on the Statement of Changes in Equity?

(i) Issue of ordinary shares (ii) Revaluation of a building (iii) Profit for the period (iv) Revaluation of a non-current asset investment

A (i)

B (i), (iii)

C (ii), (iii)

D All of the above (2 marks)

21.2 Spend Co

The following balances remain in the books of Spend Co at 30 June 20X8 after the preparation of the trading account. $ Share capital 80,000 $1 ordinary shares 80,000 40,000 8% $1 preference shares 40,000 Share premium account 10,000 Revaluation reserve 30,000 Inventories at 30 June 20X8 83,852 Trade receivables and prepayments 27,200 Trade payables and accruals 13,722 Bank balance 7,796 10% debentures 16,000 General reserve 28,000 Irrecoverable debts 340 Gross profit for the period 81,508 Wages and salaries 28,200 Insurance 1,410 Postage and telephone 620 Light and heat 1,216 Debenture interest (½ year to 31 December 20X7) 800 Directors fees 2,500 General expenses 3,108 Vehicles (cost $19,400) 6,800 Office furniture and equipment (cost $44,640) 27,440 Land and buildings at valuation 132,200 Retained earnings at 1 July 20X7 24,252 The following information is also available:

(1) The land and buildings are to be revalued at $150,000;

(2) Office furniture and equipment is to be depreciated at 15% on cost, and vehicles at 20% on cost;

(3) A bill for $348 in respect of electricity consumed up to 30 June 20X8 has not been entered in the ledger;

(4) The amount for insurance includes a premium of $300 paid in December 20X7 to cover the company against fire loss for the year 1 January 20X8 to 31 December 20X8;

Page 421: f3 Course Notes

21: QUESTIONS

421

(5) Provisions are to be made for:

$ Directors’ fees 5,000 Audit fee 1,200 The outstanding debenture interest.

(6) The directors made the following recommendations prior to the year end which have not yet been adjusted for:

(i) $12,000 should be transferred to a general reserve; (ii) the preference dividend should be accrued for payment;

Required

Prepare the income statement from the gross profit line downwards for the period ended 30 June 20X8 and a statement of financial position as at that date (ignore income tax).

Page 422: f3 Course Notes

21: QUESTIONS

422

Page 423: f3 Course Notes

423

Chapter 21: Answers

Page 424: f3 Course Notes

21: ANSWERS

424

21.1 D

21.2 Spend Co

Spend Co Income statement for the period ended 30 June 20X8

$ $ Gross profit for the period 81,508 Less expenses: Irrecoverable debts 340 Wages and salaries 28,200 Insurance (1,410 – (300 x 6/12)) 1,260 Postage and telephone 620 Light and heat (1,216 + 348) 1,564 Debenture interest (800 + 800) 1,600 Directors’ fees (2,500 + 5,000) 7,500 Audit fee 1,200 General expenses 3,108 Depreciation: Office furniture and equipment 6,696 Vehicles 3,880 55,968 Profit for the period 25,540 Spend Co Statement of financial position as at 30 June 20X8 Cost or Acc. NBV Valuation Dep'n $ $ $

NON-CURRENT ASSETS Land and buildings 150,000 – 150,000 Furniture and equipment 44,640 23,896 20,744 Motor vehicles 19,400 16,480 2,920 214,040 40,376 173,664 CURRENT ASSETS Inventories 83,852 Trade receivables and prepayments (27,200 + (300 × 6/12)) 27,350 Cash and cash equivalents 7,796

118,998 292,662

EQUITY Share capital 80,000 $1 ordinary shares 80,000 40,000 8% $1 preference shares 40,000 Share premium account 10,000 Revaluation reserve (30,000 + 17,800) 47,800 General reserve (28,000 + 12,000) 40,000 Retained earnings (Working) 34,592 252,392

NON-CURRENT LIABILITIES 10% debentures 16,000 CURRENT LIABILITIES Trade payables and accruals (13,722 + 5,000 + 1,200 + 800 + 348) 21,070 Dividends payable 3,200

24,270 292,662

Page 425: f3 Course Notes

21: ANSWERS

425

Working Retained earnings $ Retained earnings at 1 July 20X7 24,252 Profit for the period 25,540 Dividends declared 8% preference dividend (3,200) Transfer to general reserve (12,000) Retained earnings at 30 June 20X8 34,592

Page 426: f3 Course Notes

21: ANSWERS

426

END OF CHAPTER

Page 427: f3 Course Notes

427

22

Syllabus Guide Detailed Outcomes Having studied this chapter you will be able to:

• Define an event after the reporting period in accordance with International Financial Reporting Standards.

• Classify events as adjusting or non-adjusting.

• Distinguish between how adjusting and non-adjusting events are reported in the financial statements.

Exam Context Questions on this topic are likely to require you to identify adjusting and non-adjusting events from a list of options and the appropriate accounting treatment of each event. Both these types of questions were tested in the Pilot Paper.

Qualification Context The knowledge in this chapter is tested again at the Professional level paper, Corporate Reporting (P2) where you will be expected to consider how events after the reporting period may impact the way in which transactions are reported.

Events after the reporting period

Page 428: f3 Course Notes

22: EVENTS AFTER THE REPORTING PERIOD

428

Overview

Non-adjusting events

Definition

Adjusting events

Events after the reporting period

Page 429: f3 Course Notes

22: EVENTS AFTER THE REPORTING PERIOD

429

1 Definition 1.1 Events after the reporting period: events, both favourable and unfavourable, that occur

between the end of the reporting period and the date when the financial statements are authorised for issue.

1.2 There are two types of event after the balance sheet date.

2 Adjusting and non-adjusting events 2.1

Adjusting events Non-adjusting events • Events which provide evidence of

conditions which existed at the end of the reporting period.

• Events that relate to conditions which arose after the end of the reporting period

• Examples: • Examples: (1) resolution of a court case (2) bankruptcy of a major customer

(1) destruction of major asset, eg by flood or fire

(3) evidence of NRV of inventories (2) major share transactions (4) discovery of fraud or errors that show

the financial statements were incorrect (3) announcement of a plan to close part

of a business • Accounting treatment: • Accounting treatment: Change the amounts in the financial statements

Disclose non-adjusting event in a note to the financial statements

2.2 (a) Dividends proposed or declared after the end of reporting period but before the financial statements are approved should be disclosed in a note to the financial statements.

(b) A non-adjusting event that affects going concern becomes an adjusting event.

Page 430: f3 Course Notes

22: EVENTS AFTER THE REPORTING PERIOD

430

Lecture example 1 Exam standard for 2 marks Which of the following events after the reporting period would normally qualify as a non-adjusting event? 1 A fall in the market price of shares held by the entity as investments. 2 Insolvency of a trade receivable with a balance of $200,000 outstanding at the end of the

reporting period. 3 Declaration of the year-end dividend by the directors. 4 Confirmation of the amount of damages awarded to an employee who sued for unfair

dismissal after being sacked two months before the year end. A 2 only B 1 and 3 C 1, 3 and 4 D 2 and 4

Solution

3 Chapter summary Section Topic Summary 1 Definition Events after the end of the reporting period are events

which occur between the end of the reporting period and the date the financial statements are approved for issue.

2 Adjusting and non-adjusting events

There are two types: adjusting and non-adjusting. Adjusting events provide evidence of conditions that existed at the end of the reporting period. The financial statements should be changed to include this information. Non-adjusting events relate to conditions which arose after the end of the reporting period. These should be disclosed as a note to the financial statements.

Quick Quiz

Page 431: f3 Course Notes

22: EVENTS AFTER THE REPORTING PERIOD

431

Chapter Summary

Non-adjusting events

Definition

Adjusting events

Events after the reporting period

'Events, both favourable and unfavourable, that occur between the end of the reporting period and the date when the financial statements are authorised for

'Events which provide evidence of conditions which existed at the end of the reporting period' • Include in the financial statements • Examples:

– Resolution of a court case

– Bankruptcy of a major customer

– Evidence of the NRV of inventories

– Discovery of fraud or errors

'Events that relate to conditions which arose after the end of the reporting period' • Disclose in a note to the financial

statements • Examples:

- Destruction of a major asset by flood or fire

- Major share transactions

- Announcement of a plant to close part of a business

- Dividends proposed/declared after the end of the reporting period

Page 432: f3 Course Notes

22: EVENTS AFTER THE REPORTING PERIOD

432

Page 433: f3 Course Notes

433

Chapter 22: Questions

Page 434: f3 Course Notes

22: QUESTIONS

434

22.1 The following are examples of events which might occur between the end of the reporting period and the date on which the financial statements are authorised for issue:

(1) Losses on inventories as a result of a catastrophe such as a fire or flood after the year end (2) The discovery of fraud which shows that the financial statements were incorrect (3) Revaluations of property which provide evidence of an impairment in value

Which of the examples given should normally be classified as an adjusting event?

A (1), (2) and (3)

B (1) and (2)

C (1) and (3)

D (2) and (3) (2 marks)

22.2 Robin Co has a year end of 31 December 20X8, the directors were informed on 27 February 20X9 that a serious fire at one of the company's factories would stop production there for at least six months to come. On 3 March 20X9 the directors of Robin Co were informed that a major customer had gone into liquidation. The liquidator was pessimistic about the prospect of recovering anything for unsecured creditors. The financial statements for the year ended 31 December 20X8 were approved on 20 March 20X9.

In accordance with IAS 10, Events after the reporting period, how should the two events be treated in the financial statements?

Fire Liquidation

A Accounts adjusted Disclosed in notes

B Disclosed in notes Disclosed in notes

C Accounts adjusted Accounts adjusted

D Disclosed in notes Accounts adjusted (2 marks)

22.3 A Co has a year end of 31 December 20X7. During the preparation of the financial statements in March 20X8 the following issues arose:

(1) Sales of a particular inventory line were poor during the second half of 20X7. The directors had hoped that sales would pick up in 20X8 but it is now apparent that the inventory will need to be marked down below their original cost in order to sell them.

(2) On 12 February 20X8 one of the company's production plants was struck by lightening. The company will suffer a net loss of $55,000 as a result of this.

(3) Sporran Co is a valued customer which owed A Co $34,000 at the balance sheet date, although they were behind with their payments. Since the year end sales to Sporran Co were $12,000. The directors have just received notification that Sporran Co has gone into liquidation.

How should the above events be classified according to IAS 10 Events after the reporting period?

Adjusting Non-adjusting event event

A 2,3 1

B 1, 2 3

C 1,3 2

D 1, 2, 3 (2 marks)

Page 435: f3 Course Notes

435

Chapter 22: Answers

Page 436: f3 Course Notes

22: ANSWERS

436

22.1 D (1) After the end of the reporting period and therefore non-adjusting.

(2) The financial statements are incorrect, therefore clearly we must adjust. (3) The impairment is assumed to have taken place by the end of the reporting period. We

simply did not find out until later.

22.2 D The fire is a non-adjusting event as it does not affect the value of the building at 31 December 20X8. It is therefore only disclosed in a note to the financial statements unless it threatens the company's going concern in which case it would become an adjusting event.

The customer is assumed to be insolvent at 31 December 20X8. We simply did not know this and therefore it is an adjusting event and it should be adjusted for.

22.3 C

END OF CHAPTER

Page 437: f3 Course Notes

437

23

Syllabus Guide Detailed Outcomes Having studied this chapter you will be able to:

• Differentiate between profit and cash flows and understand the need for management to control cash flow.

• Recognise the benefits and drawbacks to users of the financial statements of a statement of cash flows.

• Classify the effect of transactions on cash flows and how they should be treated in a company's statement of cash flows.

• Calculate the figures needed for the statement of cash flows including cash flows from operating, investing and financing activities.

• Calculate the cash flow from operating activities using the direct and indirect method.

• Prepare extracts from statements of cash flows from given information.

Exam Context Questions on this chapter are likely to focus on whether you can identify which items should and should not go into the statement of cash flows and also on performing basic calculations. For example, you may be asked to calculate figures such as the cash generated from operations from given information or the cash paid to acquire property, plant and equipment.

Qualification Context The knowledge covered in this chapter is developed in the Fundamentals level paper Financial Reporting (F7) where you will have to produce a statement of cash flows in full. This is likely to involve more complex areas such as cash flows related to non-current assets held on finance leases. You will also need to be able to interpret a cash flow. Group statements of cash flows are examined in the Professional level paper Corporate Reporting (P2).

Business Context The ability to generate cash is key to the survival of an entity. Whilst directors may use cash budgets to estimate future cash flows, the statement of cash flows shows an historic record of how cash has been generated and where it was spent.

Cash is not subject to manipulation through an entity's choice of accounting policies. It is therefore a reliable measure of performance that is relevant to users of the financial statements.

Statements of cash flows

Page 438: f3 Course Notes

23: STATEMENTS OF CASH FLOWS

438

Overview

Cash

Cash flows

Statements of cash flows

Cash equivalents

IAS 7

Cash flows from financing activities Cash flows from

operating activities Cash flows from

investing activities

Direct method Indirect method

Page 439: f3 Course Notes

23: STATEMENTS OF CASH FLOWS

439

1 Purpose 1.1 To show the effect of a company’s commercial transactions on its cash balance.

It is thought that users of accounts can readily understand cash flows, as opposed to income statements and statements of financial position which are subject to manipulation by the use of different accounting policies. Cash flows are used as an investment appraisal method such as net present value and hence a cash flow statement gives potential investors a method with which to evaluate a business.

2 IAS 7: Statements of cash flows 2.1 IAS 7 splits cash flows into the following headings:

• Cash flows from operating activities • Cash flows from investing activities • Cash flows from financing activities

Definitions 2.2 (a) Cash (b) Cash equivalents

• cash on hand • short term, highly liquid

investments • demand deposits • readily convertible to

known amounts of cash • insignificant risk of

changes in value e.g. treasury bills

(c)

Cash flows • inflows and outflows of cash and cash equivalents

Page 440: f3 Course Notes

23: STATEMENTS OF CASH FLOWS

440

2.3 XYZ CO Statement of cash flows for the year ended 31 December 20X7 (indirect method) $000 $000 Cash flows from operating activities Profit before taxation 3,390 Adjustment for: Depreciation 450 Investment income (500) Interest expense 400 3,740 Increase in trade and other receivables (500) Decrease in inventories 1,050 Decrease in trade payables (1,740) Cash generated from operations 2,550 Interest paid (270) Income taxes paid (900) Net cash from operating activities 1,380 Cash flows from investing activities Purchase of property, plant and equipment (900) Proceeds from sale of equipment 20 Interest received 200 Dividends received 200 Net cash used in investing activities (480) Cash flows from financing activities Proceeds from issue of share capital 250 Proceeds from long-term borrowings 250 Dividends paid* (1,290) Net cash used in financing activities (790) Net increase in cash and cash equivalents 110 Cash and cash equivalents at beginning of period 120 Cash and cash equivalents at end of period 230 * This could also be shown as an operating cash flow.

Page 441: f3 Course Notes

23: STATEMENTS OF CASH FLOWS

441

3 Cash flows from operating activities 3.1 These represent cash flows derived from operating or trading activities.

An entity should report cash flows from operating activities using either: (a) The direct method, whereby major classes of gross cash receipts and payments are

disclosed (preferred method per IAS 7 – see Section 6.1), or (b) The indirect method (as above), whereby reported profit or loss is adjusted for the

effects of transactions of a non cash nature, any accruals or prepayments of operating expenses, and items relating to investing or financing cash flows.

Income taxes paid 3.2 Income taxes paid may need to be calculated from other data given to you. This is best

achieved by putting the relevant figures into a 'T' account working.

Lecture example 1 Preparation question In the statements of financial position of Tacks Co as at 31 December 20X9 and 31 December 20X8 were the following amounts for income tax payable. 31 December 20X9

$ 20X8

$ Income tax payable 156,000 168,000 The income statement tax charge for 20X9 amounted to $104,000. Required

What is the amount of income taxes paid during the year? $

Workings Income tax payable

$'000 $'000

Section 1.7.1

Page 442: f3 Course Notes

23: STATEMENTS OF CASH FLOWS

442

4 Cash flows from investing activities 4.1 The cash flows included in this section are those related to the acquisition or disposal of any

non-current assets or investments together with returns received in cash from investments, i.e. dividends and interest. This section shows the extent to which expenditures have been made for resources intended to generate future income and cash flows.

Lecture example 2 Preparation question On 31 December 20X8 the value of plant and equipment in the books of Erosion Co was as follows:

$ Plant and equipment at cost 200,000 Accumulated depreciation 80,000 Plant and equipment at net book value 120,000 On 1 January 20X9 an item of plant was sold for $8,000 which had originally cost $20,000 when new, but had a net book value of $11,000 at the time of sale. (The statement of financial position values shown above do not show that this sale has taken place.) On 31 December 20X9 the value of plant and equipment in the statement of financial position was:

$ Plant and equipment at cost 280,000 Accumulated depreciation 111,000 Plant and equipment at net book value 169,000 Required

Show the relevant entries for property, plant and equipment which would appear in a statement of cash flows for Erosion Co in 20X9.

Solution Workings

Plant & equipment – cost $'000 $'000

Accumulated depreciation

$'000 $'000

Section 1.7.2

Page 443: f3 Course Notes

23: STATEMENTS OF CASH FLOWS

443

5 Cash flows from financing activities 5.1 Financing cash flows comprise receipts from or repayments to external providers of finance

in respect of principal amounts of finance. Examples of financing cash flows are: • Cash proceeds from issuing shares • Cash proceeds from issuing debentures, loans, notes, bonds, mortgages and other

short or long term borrowings • Cash repayments of amounts borrowed • Dividends paid to shareholders In order to calculate such figures the closing statement of financial position figure for debt or share capital and share premium is compared with the opening position for the same items.

Dividends paid 5.2 The cash outflows included in dividends paid are dividends paid on the reporting company's

equity shares.

Lecture example 3 Preparation question Distribution Co statement of financial position extract for the year ended 31 December 20X9 20X9 20X8 $'000 $'000 Dividends payable 45 35 Dividends charged to retained earnings were $60,000. Required

What are the dividends paid during the year ended 31 December 20X9? $ Workings

Dividends payable $'000 $'000

Section 1.7.3

Page 444: f3 Course Notes

23: STATEMENTS OF CASH FLOWS

444

Lecture example 4 Technique question The summarised accounts of the Emma Co for the year ended 31 December 20X8 are as follows: Statement of financial position as at 31 December 20X8 20X7 $'000 $'000 Non-current assets Property, plant and equipment 628 514 Current assets: Inventories 214 210 Trade receivables 168 147 Cash 7 – 389 357 1,017 871 Equity Share capital ($1 ordinary shares) 250 200 Share premium account 70 60 Revaluation reserve 110 100 Retained earnings 314 282 744 642 Non-current liabilities 10% debentures 80 50 Current liabilities Trade payables 136 121 Income tax payable 39 28 Dividends payable 18 16 Overdraft – 14 193 179 1,017 871

Income statement for the year ended 31 December 20X8 $'000 Revenue 600 Cost of sales 319 Gross profit 281 Other expenses (including depreciation of $42,000) 186 Finance costs (interest paid) 8 Profit before tax 87 Income tax expense 31 Profit for the period 56

Page 445: f3 Course Notes

23: STATEMENTS OF CASH FLOWS

445

Movement of retained earnings $000 Balance at 31 December 20X7 282 Profit for the period 56 Dividends (24) Balance at 31 December 20X8 314 You are additionally informed that there have been no disposals of property, plant and equipment during the year. The new debentures were issued on 1 January 20X8. Required

Produce a statement of cash flows for Emma Co for the year ended 31 December 20X8.

Solution EMMA CO Statement of cash flows for the year ended 31 December 20X8

$’000 $’000 Cash flows from operating activities Profit before taxation Adjustments for: Depreciation Interest expense Increase in trade receivables Increase in inventories Increase in trade payables Cash generated from operations Interest paid Income taxes paid Net cash from operating activities Cash flows from investing activities Purchase of property, plant and equipment Net cash used in investing activities Cash flows from financing activities Proceeds from issue of shares Proceeds from issue of debentures Dividends paid Net cash from financing activities Net increase in cash and cash equivalents Cash and cash equivalents at beginning of year Cash and cash equivalents at end of year

Page 446: f3 Course Notes

23: STATEMENTS OF CASH FLOWS

446

Workings

Page 447: f3 Course Notes

23: STATEMENTS OF CASH FLOWS

447

6 Cash flows from operating activities using the direct method

6.1 As noted in Section 3.1, IAS 7 has two methods available under which the statement of cash flows can be prepared: • indirect method (seen previously) • direct method

6.2 The only difference is the direct method derives the 'cash generated from operations' figure in a different way. The operating element of the statement of cash flows should be shown as follows: $000 $000 Cash flows from operating activities Cash receipts from customers 30,150 Cash payments to suppliers and employees (27,600) Cash generated from operations 2,550 Interest paid (270) Income taxes paid (900) Net cash from operating activities 1,380

Cash received from customers 6.3 This represents cash flows received during the accounting period in respect of sales.

Cash payments to suppliers and employees 6.4 This represents cash flows made during the accounting period in respect of goods and

services and amounts paid to employees.

Page 448: f3 Course Notes

23: STATEMENTS OF CASH FLOWS

448

Lecture example 5 Technique question Required

Using the information in Lecture example 4 produce the 'cash flows from operating activities' section of the cash flow statement using the direct method.

Solution EMMA CO Statement of cash flows for year ended 31 December 20X8 (extract) $ $ Cash flows from operating activities Cash receipts from customers Cash payments to suppliers and employees Cash generated from operations Interest paid Income taxes paid Net cash used in operating activities

Page 449: f3 Course Notes

23: STATEMENTS OF CASH FLOWS

449

7 Chapter summary Section Topic Summary

1 Purpose The statement of cash flows shows the movement between a company’s cash and cash equivalents at the beginning and the end of the year.

2 Statements of cash flows (IAS 7)

Cash comprises cash on hand and on demand deposits, less bank overdrafts. Cash equivalents are short term, highly liquid investments such as current asset investments (shares) which can be converted in to known amounts of cash relatively quickly without having a major impact on the entity’s activities.

3 Cash flows from operating activities

This section of the statement of cash flows shows the cash and cash equivalents generated by and used in the entity’s main trading activities.

4 Cash flows from investing activities

This section shows the cash flows related to the acquisition and disposal of non-current assets and returns on investments such as interest and dividends received.

5 Cash flows from financing activities

Cash flows from financing activities include the monies raised from issuing shares and loans and the cash used in the repayment of loans and the payment of dividends.

6 Cash flow from operating activities using the direct method

The statement of cash flows can be produced using one of two methods: the indirect or the direct method. The direct method provides exactly the same cash flow information but calculates the cash flow from operating activities using a slightly different calculation from the indirect method.

Quick Quiz

Page 450: f3 Course Notes

23: STATEMENTS OF CASH FLOWS

450

Chapter Summary

Cash

Cash flows

Statements of cash flows

Cash equivalents

IAS 7

Cash flows from financing activities Cash flows from

operating activities Cash flows from

investing activities

Direct method Indirect method

'Cash on hand and demand deposits'

'Short-term, highly liquid investments' • Example: – current asset investments

'Inflows and outflows of cash and cash equivalents'

'Requires that a company show the movement in cash and cash equivalents between the beginning and the end of the year under three headings' • cash flows from operating activities • cash flows from investing activities • cash flows from financing activities

'Cash flows from trading activities' 'Cash flows relating to the acquisition or disposal of non-current assets and the returns on investments' • Includes – purchase of non-current assets

– proceeds from sale of non-current assets

– interest/dividends received

'Cash flows relating to the issue or repayment of long term finance' • Includes

– proceeds from share capital/debenture issue

– repayment of loans – ordinary dividends paid • Cash generated from operations

• Adjust profit before tax figure for: – non-cash items – items shown elsewhere in the

cash flow – movements in working capital • Then deduct interest and income

taxes paid

• Cash generated from operations: • Derived by calculating: – cash receipts from customers – cash payments to suppliers and employees • Then deduct interest and income taxes paid

Page 451: f3 Course Notes

451

Chapter 23: Questions

Page 452: f3 Course Notes

23: QUESTIONS

452

23.1 In a statement of cash flows which of the items below would not appear as an outflow of cash?

A The nominal value of debenture redeemed at par during the year

B The dividends paid to preference shareholders during the year

C The income statement charge for tax for the year (1 mark)

Data for Questions 23.2 and 23.3 Extracts from a company’s statements of financial position show the following items of property, plant and equipment at net book value:

The building element of the freehold property was depreciated by $6,000 and then revalued on 30 June 20X7 by $95,000. Plant and equipment, which had cost $49,000 when purchased in January 20X2 on which $35,000 of depreciation had been charged, was disposed of in November 20X6 for $8,000. Depreciation on the plant and equipment for the year amounted to $37,000. Depreciation of $55,000 has been charged on furniture and fixtures.

23.2 What is the total figure to be adjusted for in ‘cash flows from operating activities’ in respect of property, plant and equipment? $ (2 marks)

23.3 What is the total expenditure on property, plant and equipment included under ‘cash flows from investing activities’? $ (2 marks)

23.4 In a statement of cash flows, a decrease in loan stock would be shown as a cash inflow under 'cash flows from financing activities'.

A True

B False (1 mark)

23.5 These extracts have been taken from the accounts of Jeanne Co.

Statement of financial position (extracts) 31 October 31 October 20X7 20X6 Current liabilities

Dividends payable 9,750 5,750 Dividends charged to retained earnings during the year were $15,500.

What will appear as “dividends paid” in the statement of cash flows for the year ended 31 October 20X7?

A $5,750

B $11,500

C $15,500

D $21,250 (2 marks)

30 June 20X7 20X6 $ $ Property, plant and equipment

Freehold property 1,230,000 750,000 Plant and equipment 465,000 380,000 Furniture and fixtures 90,000 105,000

Page 453: f3 Course Notes

23: QUESTIONS

453

23.6 Jane Co

Income statement for the year ended 31 December 20X7

$’000 Revenue 2,553 Cost of sales 1,814 Gross profit 739 Distribution costs 125 Administrative expenses 264 Investment income 25 Finance costs 75 Profit before tax 300 Income tax expense 140 Profit for the period 160

Statement of financial position as at 31 December 20X7 20X6 $’000 $’000 Non-current assets Property, plant and equipment 380 305 Development expenditure 250 200 Investments – 25 630 530 Current assets Inventories 150 102 Trade receivables 390 315 Short-term investments 50 – Cash in hand 2 1 592 418 Total assets 1,222 948 Equity Share capital ($1 ordinary shares) 200 150 Share premium account 160 150 Revaluation reserve 100 91 Retained earnings 160 100 620 491 Non-current liabilities Long term loan 100 – Current liabilities Trade payables 127 119 Bank overdraft 85 98 Income tax payable 190 160 Dividends payable 100 80 502 457 Total equity and liabilities 1,222 948

Page 454: f3 Course Notes

23: QUESTIONS

454

The following information is available: (a) The proceeds of the sale of non-current asset investments amounted to $30,000;

(b) Furniture and fixtures, with an original cost of $85,000 and a net book value of $45,000, were sold for $32,000 during the year;

(c) The current asset investments fall within the definition of cash equivalents under IAS 7;

(d) The following information relates to property, plant and equipment:

20X7 20X6 $’000 $’000 Cost 720 595 Accumulated depreciation 340 290 Net book value 380 305

(e) 50,000 $1 ordinary shares were issued during the year at a premium of 20c per share;

(f) Dividends charged to retained earnings were $100,000 in 20X7;

(g) Development expenditure has not yet started being amortised.

Required

Prepare a statement of cash flows for the year to 31 December 20X7.

Page 455: f3 Course Notes

455

Chapter 23: Answers

Page 456: f3 Course Notes

23: ANSWERS

456

23.1 C Income tax paid is a cash flow not the income statement tax charge.

23.2 $104,000 Property, plant and equipment $’000 $’000 Bal b/d Freehold property 750 Plant & Equipment 380 Disposal – Plant & Equipment Furniture & Fixtures 105 (49 – 35) 14 Depreciation Freehold property 6 Plant & Equipment 37 Furniture & Fixtures 55 Revaluation Freehold property 95 98 ∴ Acquisitions 567 Bal c/d Freehold property 1,230 Plant & Equipment 465 Furniture & Fixtures 90 1,897 1,897 Total adjustments in the reconciliation: $’000 Depreciation 98 Loss on disposal of plant and equipment (8 – 14) 6 104

23.3 $567,000

See previous calculation

23.4 B

23.5 B Dividends payable

$ $ Balance b/d 5,750

∴ Paid 11,500 Retained earnings 15,500

Balance c/d 9,750 21,250 21,250

Page 457: f3 Course Notes

23: ANSWERS

457

23.6 Jane Co

Statement of cash flows for the year ended 31 December 20X7 $’000 $’000 Cash flows from operating activities Profit before taxation 300 Adjustments for: Depreciation (W2) 90 Loss on sale of property, plant and equipment (45 – 32) 13 Profit on sale of non-current asset investments (30 – 25) (5) Investment income (25) Finance costs 75 448 Increase in trade receivables (390 – 315) (75) Increase in inventories (150 – 102) (48) Increase in trade payables (127 – 119) 8 Cash generated from operations 333 Interest received 25 Interest paid (75) Income taxes paid (W4) (110) Net cash from operating activities 173 Cash flows from investing activities Purchase of property, plant and equipment (W1) (201) Proceeds from sale of property, plant and equipment 32 Proceeds from sale of non-current asset investments 30 Payments for development expenditure (W3) (50) Net cash used in investing activities (189) Cash flows from financing activities Proceeds from issue of ordinary share capital 60 Proceeds from long term loan 100 Dividends paid (W5) (80) Net cash from financing activities 80 Increase in cash and cash equivalents 64 Cash and cash equivalents at beginning of period (1 – 98) (97) Cash and cash equivalents at end of period (50 + 2 – 85) (33)

Workings

(W1) Property, plant and equipment – cost

$’000 $’000 Balance b/d 595 Disposals 85 Revaluation (100 – 91) 9 Additions (bal fig) 201 Balance c/d 720 805 805

(W2) Property, plant and equipment - Accumulated depreciation

$’000 $’000 Disposals (85 – 45) 40 Balance b/d 290 Balance c/d 340 ∴Depreciation charge 90 380 380

Page 458: f3 Course Notes

23: ANSWERS

458

(W3) Development expenditure

$’000 $’000 Balance b/d 200 ∴ additions 50 Balance c/d 250 250 250

(W4) Income tax payable

$’000 $’000 ∴ Income tax paid 110 Balance b/d 160 Balance c/d 190 Income statement 140 300 300

(W5) Dividends payable

$’000 $’000 ∴ Dividends paid 80 Balance b/d 80 Balance c/d 100 Retained earnings 100 180 180

END OF CHAPTER

Page 459: f3 Course Notes

459

24

Syllabus Guide Detailed Outcomes Having studied this chapter you will be able to:

• Understanding the basic function and form of accounting records in manual and computerised systems.

• Compare manual and computerised systems and identify advantages and disadvantages of computerised accounting systems.

• Understand the uses of integrated accounting software packages.

• Understand business use of computers and the nature and purpose of spreadsheets and database systems.

Exam Context Questions on this topic are likely to focus on the advantages and disadvantages of using a computerised system and the differences between a manual and a computerised system.

Qualification Context The importance of accounting systems and internal controls is tested in the Fundamentals level paper, Accountant in Business (F1).

Home study chapter –Information technology

Page 460: f3 Course Notes

24: HOME STUDY CHAPTER – INFORMATION TECHNOLOGY

460

Overview

Spreadsheets Databases

Information technology

Accounting modules

Computerised accounting packages Integrated software

Page 461: f3 Course Notes

24: HOME STUDY CHAPTER – INFORMATION TECHNOLOGY

461

1 Introduction 1.1 In today's world most businesses use accounting systems which are computerised, although

some smaller businesses may keep manual records.

1.2 The same principles of double entry are used regardless of whether an accounting system is manual or computerised.

2 Accounting packages 2.1 There are two main types of computerised accounting packages:

(a) Dedicated accounting packages, for example SAGE. (b) General software, for example spreadsheets which can be used to keep accounting

records.

2.2 Advantages and disadvantages of computerised accounting packages.

Advantages Disadvantages (1) Large amounts of data can be

processed very quickly (1) Time and cost in setting up the system

and staff training (2) Computerised systems are more

accurate (2) Need for internal controls and security

checks to ensure the accuracy of data (3) Large volumes of data can be

processed (3) Lack of 'audit trail'

(4) Little training is required (4) Staff may resist the introduction of a computerised system

(5) Computer can analyse data into tailored reports

3 Accounting modules

Definition 3.1 Accounting module – a program which deals with one part of a business' accounting

system

3.2 Examples of modules include: (a) Invoicing (b) Receivables ledger (c) Nominal ledger (d) Payroll (e) Cash book (f) Non-current asset register

Page 462: f3 Course Notes

24: HOME STUDY CHAPTER – INFORMATION TECHNOLOGY

462

Integrated software 3.3 Each module may be integrated with other modules so that when information is recorded in

one module it is automatically updated in another module. Examples: (a) The payroll module may be integrated with the nominal ledger module so that once

the payroll information is determined the associated wages expense is updated in the nominal ledger.

(b) The invoicing module may be integrated with the inventory, receivables ledger and nominal ledger modules so that once an invoice is sent the inventory levels are updated as is the customer's account in the receivables ledger.

3.4 Advantages and disadvantages of integrated software.

Advantages Disadvantages (1) An entry in one module

automatically updates all the others

(1) These systems require more memory than a stand-alone system so there is less space to store actual data

(2) Reports generated by the system can draw information from all relevant modules

(2) Each module may be limited to fewer functions than a specialised module (because one program is doing everything)

(3) Reduction in clerical time used to input information and errors

(3) An error in one part of the system will flow through to all areas

4 Databases

Definition 4.1 A database is a 'pool of data' which can be used by any number of applications.

4.2 Examples: (a) Non-current asset register (b) List of customers/suppliers (c) Price lists

Section 1.5

Page 463: f3 Course Notes

24: HOME STUDY CHAPTER – INFORMATION TECHNOLOGY

463

Lecture example 1 Idea generation What sort of information might be contained in a database file for a non-current asset register?

Solution

4.3 A database should have four major objectives. (a) It should be shared – different individuals should be able to access the

same information (b) Its integrity must be preserved – only valid alterations to information should be

made (c) It should meet the needs of different users.

For example, the accounts department may be interested predominantly in the net book value of the non-current assets but the production manager will need to know their whereabouts in order to schedule jobs efficiently

(d) The database must be able to grow and develop according to the needs of the business

5 Spreadsheets 5.1 Spreadsheets are essentially an electronic piece of paper. They are used in all parts of a

business, predominantly to perform numerical calculations.

5.2 Uses of spreadsheets by the accounting function: (a) To maintain accounting records, for example a cash book (b) To produce financial statements (c) To produce budgets/forecasts (d) To conduct variance analysis

Page 464: f3 Course Notes

24: HOME STUDY CHAPTER – INFORMATION TECHNOLOGY

464

6 Chapter summary Section Topic Summary

1 Introduction Nowadays most businesses use a computerised accounting system. However the mechanics of double entry bookkeeping are the same regardless of whether a manual or a computerised accounting system is in place.

2 Accounting packages There are two main types of accounting packages: dedicated packages such as SAGE and general software such as spreadsheets.

3 Accounting modules An accounting module is a program which deals with one part of a business’ accounting system. These modules may or may not be integrated with other modules.

4 & 5 Databases and spreadsheets

Databases and spreadsheets are electronic ways of holding and manipulating information.

.

Quick Quiz

Page 465: f3 Course Notes

24: HOME STUDY CHAPTER – INFORMATION TECHNOLOGY

465

Chapter Summary

Spreadsheets Databases

Information technology

Accounting modules

Computerised accounting packages Integrated software

• Different accounting modules may be integrated • Advantages: - one entry updates all modules - reduced input time • Disadvantages: - an error in one part of the system will flow through to all areas

• Dedicated accounting packages: - off the shelf, for example SAGE - customised • General software - for example: spreadsheets • Advantages - speed - accuracy - volume of data - computer analysis for reports • Disadvantages - time and cost - need for internal controls - lack of audit trail

'A program which deals with one part of a business' accounting systems' • Examples: - invoicing - receivables ledger - nominal ledger - payroll - non-current asset register

'A pool of data which can be used by any number of applications'. • Examples - non-current asset register - customer/supplier lists - price lists

'An electronic piece of paper' • Used - to maintain accounting records - to produce budgets/forecasts - to conduct variance analysis

Page 466: f3 Course Notes

24: HOME STUDY CHAPTER – INFORMATION TECHNOLOGY

466

Page 467: f3 Course Notes

467

Chapter 24: Questions

Page 468: f3 Course Notes

24: QUESTIONS

468

24.1 All businesses will apply the same principles of double entry bookkeeping regardless of whether they operate a manual or a computerised system.

Is this statement true or false?

A True

B False (1 mark)

24.2 If a database is to contain accurate and valid information it should only be amended by authorised personnel.

Is this statement true or false?

A True

B False (1 mark)

Page 469: f3 Course Notes

469

Chapter 24: Answers

Page 470: f3 Course Notes

24: ANSWERS

470

24.1 A

24.2 A

END OF CHAPTER

Page 471: f3 Course Notes

471

Answers to Lecture Examples

Page 472: f3 Course Notes

25: ANSWERS TO LECTURE EXAMPLES

472

Chapter 1

Answer to Lecture Example 1 Users of financial information (a) Investors – Profitability – Future prospects – Likely risk and return – Chance of capital growth – Ability to pay dividends (b) Employees – Profitability – Long-term growth – Security of their job – Likelihood of bonus – Number of employees – Ability to pay retirement benefits (c) Lenders – Whether return on finance will continue to be met – Other providers and security of their debt – Likelihood of repayment of capital amount (d) Suppliers – Likelihood of payment on time – Likelihood of payment at all – Whether they should continue to supply (e) Customers – Ability of entity to continue supplying – Profitability as a measure of value for money of goods bought (f) Government and their agencies – Statistics – Size of company – Growth rates – Average payment periods – Foreign trade – Profits made – Corporate income tax liability – Sales tax liability (g) Public – Contribution to local economy – Information about trends in the prosperity of the entity – Range of activities provided

Page 473: f3 Course Notes

25: ANSWERS TO LECTURE EXAMPLES

473

Chapter 2

Answer to Lecture Example 1 A The IASCF appoints members to the IASB, IFRIC and SAC. The SAC advises the IASB on its

agenda.

Answer to Lecture Example 2 A

Chapter 3

Answer to Lecture Example 1 Advantages of historic cost

(1) The transaction cost of $1 million is a very reliable figure which was quantified at the date of acquisition.

(2) Using current market values for the building may lead to volatility in asset values due to changing market prices.

(3) Any change in the asset's value will affect the amount of depreciation charged and therefore the entity's profits. This makes comparability more difficult.

Disadvantages of historic cost

(1) Asset values generally appreciate over time and so using historic cost will mean that the financial statements contain information which is out of date and therefore less useful for decision making.

(2) Sales revenue and costs will be shown at current prices but depreciation will be based on historic cost and therefore too low a figure. Profits will therefore look artificially high.

Answer to Lecture Example 2 (a) Historic cost is $1,000

(b) Net realisable value is

$ Selling price (100 × $11) 1,100 Less: completion costs (100 × $2) (200) 900

(c) Show inventory at the lower of cost and net realisable value = $900.

Page 474: f3 Course Notes

25: ANSWERS TO LECTURE EXAMPLES

474

Chapter 4

Answer to Lecture Example 1 Own Examples: (i) House (ii) Car (iii) Cash Owe Examples: (i) Mortgage (ii) Car loan (iii) Credit card

Chapter 5

Answer to Lecture Example 1

Transaction Debit Credit

(a) Sales for cash Cash increase asset

Sales income

(b) Sales on credit Receivables increase asset

Sales income

(c) Purchases for cash Purchases expense

Cash decrease asset

(d) Purchases on credit Purchases expense

Payables increase liability

(e) Pay electricity bill Electricity expense

Cash decrease asset

(f) Receive cash from a credit customer Cash increase assets

Receivables decrease assets

(g) Pay cash to a credit supplier Payables decrease liability

Cash decrease asset

(h) Borrow money from the bank Cash increase asset

Loan increase liability

Page 475: f3 Course Notes

25: ANSWERS TO LECTURE EXAMPLES

475

Answer to Lecture Example 2 Cash

$ $ Capital 5,000 Rent 500 Sales 2,100 Electricity 200 Car 1,000 Drawings 300

Capital $ $ Cash 5,000

Trade payables $ $ Purchases 2,000

Purchases $ $ Trade payables 2,000

Rent $ $ Cash 500

Electricity $ $ Cash 200

Car $ $ Cash 1,000

Drawings $ $ Cash 300

Trade receivables $ $ Sales 1,750

Sales $ $ Trade receivables 1,750 Cash 2,100

Answer to Lecture Example 3 Dr Cash Cr

$ $ 2/1 Sales 500 1/1 Purchases 300 10/1 Sales 500 25/1 Telephone 50 Bal c/d 650 1,000 1,000 Bal b/d 650

Page 476: f3 Course Notes

25: ANSWERS TO LECTURE EXAMPLES

476

Answer to Lecture Example 4 Cash

$ $ Capital 5,000 Rent 500 Sales 2,100 Electricity 200 Car 1,000 Drawings 300 Bal c/d 5,100 7,100 7,100 Bal b/d 5,100

Capital $ $ Bal c/d 5,000 Cash 5,000 5,000 5,000 Bal b/d 5,000

Trade payables $ $ Bal c/d 2,000 Purchases 2,000 2,000 2,000 Bal b/d 2,000

Purchases $ $ Trade Payables 2,000 Bal c/d 2,000 Bal b/d 2,000

Rent $ $ Cash 500 Bal c/d 500 Bal b/d 500

Electricity $ $ Cash 200 Bal c/d 200 Bal b/d 200

Car $ $ Cash 1,000 Bal c/d 1,000 Bal b/d 1,000

Drawings $ $ Cash 300 Bal c/d 300 Bal b/d 300

Trade receivables $ $ Sales 1,750 Bal c/d 1,750 Bal b/d 1,750

Page 477: f3 Course Notes

25: ANSWERS TO LECTURE EXAMPLES

477

Sales $ $ Bal c/d 3,850 Trade receivables 1,750 Cash 2,100 3,850 3,850 Bal b/d 3,850

Chapter 6

Answer to Lecture Example 1 Trial Balance

Debit Credit $ $ Cash 5,100 Capital 5,000 Trade payables 2,000 Purchases 2,000 Rent 500 Electricity 200 Car 1,000 Drawings 300 Trade receivables 1,750 Sales 3,850 10,850 10,850

Purchases $ $ Creditors 2,000 Bal c/d 2,000 Bal b/d 2,000 Income statement 2,000

Rent $ $ Cash 500 Bal c/d 500 Bal b/d 500 Income statement 500

Electricity $ $ Cash 200 Bal c/d 200 Bal b/d 200 Income statement 200

Sales $ $ Bal c/d 3,850 Trade receivables 1,750 Cash 2,100 3,850 3,850 Income statement 3,850 Bal b/d 3,850

Page 478: f3 Course Notes

25: ANSWERS TO LECTURE EXAMPLES

478

Answer to Lecture Example 2 Colin

Income Statement $ $ Sales (20 telephones) 600 Cost of sales Purchases (50 telephones) 1,000 Less: closing inventories (30 telephones) (600) 400 Gross profit 200

Answer to Lecture Example 3 Income Statement

$ $ Purchases 2,000 Sales 3,850 Gross profit c/d 2,100 Closing inventory 250 4,100 4,100 Rent 500 Gross profit b/d 2,100 Electricity 200 Net profit c/d 1,400 2,100 2,100 Net profit b/d 1,400

Answer to Lecture Example 4 DOUGLAS INCOME STATEMENT FOR THE MONTH OF JANUARY

$ $ Sales 3,850 Less cost of sales: Purchases 2,000 Less: closing inventories ( 250) 1,750 Gross profit 2,100 Less expenses: Rent 500 Electricity 200 (700) Net profit 1,400

Page 479: f3 Course Notes

25: ANSWERS TO LECTURE EXAMPLES

479

DOUGLAS STATEMENT OF FINANCIAL POSITION AS AT 31 JANUARY NON-CURRENT ASSET $ $ Motor vehicle 1,000 CURRENT ASSETS Inventories 250 Trade receivables 1,750 Cash 5,100

7,100 8,100

PROPRIETOR’S INTEREST $ $ Capital introduced on 1 January 5,000 Profit for the year 1,400 Less: drawings (300) Balance 31 January 6,100 CURRENT LIABILITIES Trade payables 2,000

8,100

Answer to Lecture Example 5 Drawings

$ $ Cash 300 Bal c/d 300 Bal b/d 300 Capital 300

Income statement $ $ Purchases 2,000 Sales 3,850 Gross profit c/d 2,100 Closing inventory 250 4,100 4,100 Rent 500 Gross profit b/d 2,100 Electricity 200 Net profit c/d 1,400 2,100 2,100 Capital 1,400 Net profit b/d 1,400

Capital $ $ Balance c/d 5,000 Cash 5,000 Drawings 300 Balance b/d 5,000 Balance c/d 6,100 Net profit 1,400 6,400 6,400 Balance b/d 6,100

Answer to Lecture Example 6 Assets = capital + (profit – drawings) + payables

8,100 = 5,000 + (1,400 – 300) + 2,000

Page 480: f3 Course Notes

25: ANSWERS TO LECTURE EXAMPLES

480

Chapter 7

Answer to Exercise Net

$ Sales tax

$ Gross

$ (1) Factory buys raw material 100 15.00 115.00 (2) Manufactures goods and sells to wholesaler 250 37.50 287.50 22.50 Due to sales tax authority

Answer to Lecture Example 1 Purchases Trade payables

$ Trade payables 1,000

$ Purchases 1,150

Trade receivables Sales tax control a/c

$ Sales 1,725

$ Trade payables 150

$ Trade rec. 225

Sales

$

$ Trade rec. 1,500

Chapter 8

Answer to Lecture Example 1 C Transport costs to deliver goods to customers are an example of carriage outwards and should

not be included. Administrative overheads do not relate to production and cannot therefore be included.

The depreciation of the factory machine is a production overhead and should be included.

Answer to Lecture Example 2 Net realisable value is:

$ Estimated selling price 35 Less: costs of completion (12) Less: selling costs (1) 22

Page 481: f3 Course Notes

25: ANSWERS TO LECTURE EXAMPLES

481

Answer to Lecture Example 3 (a) Closing inventories (FIFO)

Purchases Opening

inventories 10 Jan 20 Jan 25 Jan

200 300 350 250 Sales 14 Jan (200) (80) 21 Jan (220) (180) 28 Jan (80) Nil Nil 90 250 @ $11.50 @ $13.00 = $1,035 = $3,250 $4,285

Cost of sales (FIFO) $ Opening inventories (200 x $10) 2,000 Purchases 10,530 12,530 Less: closing inventories (4,285) 8,245

(b) Closing inventories and cost of sales (AVCO) Average Total Cost of Units Cost Unit Cost Cost Sales $ $ $ $ 1.1.X2 b/f 200 10.00 2,000 10.1.X2 Purchase 300 10.85 3,255 500 (W1) 10.51 5,255 14.1.X2 Sale (280) 10.51 (2,943) 2,943 220 2,312 20.1.X2 Purchase 350 11.50 4,025 570 (W2) 11.12 6,337 21.1.X2 Sale (400) 11.12 (4,448) 4,448 170 1,889 25.1.X2 Purchase 250 13.00 3,250 420 (W3) 12.24 5,139 28.1.X2 Sale (80) 12.24 (979) 979 340 4,160 8,370

(W1) 51.10$500

$5,255= (W2) 12.11$

570$6,337

= (W3) 24.12$420

$5,139=

Page 482: f3 Course Notes

25: ANSWERS TO LECTURE EXAMPLES

482

Chapter 9

Answer to Lecture Example 1 Examples include:

(a) Land and buildings (b) Plant and equipment (c) Motor vehicles (d) Furniture and fittings, computers

Answer to Lecture Example 2 B The cost capitalised should include the purchase price ($20,000) plus all directly attributable costs

(delivery and installation).

The cost of the maintenance contract should be shown as an expense in the income statement.

Answer to Lecture Example 3 Straight line method:

(a) Depreciation charge = years3

250 - $500,2$

= $750 per annum

(b) Year Cost Accumulated NBV depreciation $ $ $ 1 2,500 750 1,750 2 2,500 1,500 1,000 3 2,500 2,250 250

Answer to Lecture Example 4 Reducing balance method: NBV b/d Dep’n Dep’n Accumulated NBV c/d rate expense depreciation $ $ $ Year 1 (6,000 – 0) × 40% 2,400 2,400 3,600 Year 2 (6,000 – 2,400) × 40% 1,440 3,840 2,160 Year 3 (6,000 – 3,840) × 40% 864 4,704 1,296

Answer to Lecture Example 5 (a) Journal entry

Debit Credit $ $ Depreciation expense 750 Accumulated depreciation 750 Being annual depreciation charged on machine

Page 483: f3 Course Notes

25: ANSWERS TO LECTURE EXAMPLES

483

(b) Accounting for depreciation: Machine (SOFP)

$ Cash 2,500 2,500 Bal b/d 2,500

$ Bal c/d 2,500 2,500

Depreciation expense (I/S)

$ Year 1 Accumulated dep’n 750 Year 2 Accumulated dep’n 750 Year 3 Accumulated dep’n 750

$ Year 1 I/S 750 Year 2 I/S 750 Year 3 I/S 750

Accumulated depreciation (SOFP)

$ Bal c/d 750 Bal c/d 1,500 1,500 Bal c/d 2,250 2,250

$ Year 1 Depreciation expense 750

Year 2 Bal b/d 750 Depreciation expense 750 1,500 Year 3 Bal b/d 1,500 Depreciation expense 750 2,250

(c) Income statement (extracts): Year 1 Year 2 Year 3 $ $ $ Expenses Depreciation 750 750 750 Statement of financial position (extracts): Cost Accumulated Net Book Depreciation Value $ $ $ (Year 1) Machine 2,500 (750) 1,750 (Year 2) Machine 2,500 (1,500) 1,000 (Year 3) Machine 2,500 (2,250) 250

Page 484: f3 Course Notes

25: ANSWERS TO LECTURE EXAMPLES

484

Answer to Lecture Example 6 (a)

$ Sales proceeds 3,000 NBV at end of year 2 (2,160) 840

(b) Machine (SOFP)

$ Bal b/d 6,000

$ (a) Disposal account 6,000

Accumulated depreciation (SOFP)

$ (b) Disposal account 3,840

$ Bal b/d 3,840

Disposal account (I/S)

$ (a) Machine 6,000 Balance = profit 840 on disposal (I/S) 6,840

$ (c) Cash 3,000 (b) Accumulated dep’n 3,840

6,840

Answer to Lecture Example 7 (a) The profit on disposal is still $840, the only difference is that the proceeds were not received in

cash, but in the form of a part exchange allowance.

(b) Cash paid for the new machine is $7,000 ($10,000 – $3,000)

Old machine (SOFP)

$ Bal b/d 6,000

$ (a) Disposal account 6,000

Accumulated depreciation (SOFP)

$ (b) Disposal account 3,840

$ Bal b/d 3,840

New machine (SOFP)

$ (c) Disposal account 3,000 Cash 7,000 10,000 Bal b/d 10,000

$ Bal c/d 10,000 10,000

Page 485: f3 Course Notes

25: ANSWERS TO LECTURE EXAMPLES

485

Disposal account (I/S)

$ (a) Machine 6,000 Profit on disposal (I/S) 840 6,840

$ (c) New machine (part exchange) 3,000 (b) Accumulated depreciation 3,840

6,840

Answer to Lecture Example 8 (a) The double entry is

$ $ Dr Non-current asset – building (150 – 100) 50,000 Dr Accumulated depreciation – building 20,000 Cr Revaluation reserve (β) 70,000

Building (SOFP)

$ Bal b/d 100,000 Revaluation reserve 50,000 150,000 Bal b/d 150,000

$ Bal c/d 150,000 150,000

Accumulated depreciation (SOFP)

$ Revaluation reserve 20,000

$ Bal b/d 20,000

Revaluation reserve (SOFP)

$ Bal c/d 70,000 70,000

$ Building 50,000 Accumulated depreciation 20,000 70,000

Bal b/d 70,000

(b) Depreciation charge is years40

000,150$ = $3,750

Answer to Lecture Example 9 Review of useful life:

Year Depreciation charge

Accumulated depreciation

NBV

$ $ $ 20X1

5000,40

= 8,000 8,000 32,000

20X2

5000,40

= 8,000 16,000 24,000

20X3

2000,24

= 12,000 28,000 12,000

20X4 2000,24 = 12,000

40,000 0

40,000

Page 486: f3 Course Notes

25: ANSWERS TO LECTURE EXAMPLES

486

Answer to Lecture Example 10 Change in method of depreciation:

Depreciation charge

Accumulated depreciation

NBV

$ $ $ 20X1 40,000 × 25% 10,000 10,000 30,000 20X2 30,000 × 25% 7,500 17,500 22,500 20X3

3500,1 - 500,22

7,000 24,500 15,500

20X4 7,000 31,500 8,500 20X5 7,000 38,500 1,500 38,500

Chapter 10

Answer to Lecture Example 1 (1) Market research would take place at an early stage in any development process. Its purpose is to

gather information about whether there may be interest in a potential product. At this point in time an entity cannot be certain that the expenditure will lead to profits and so the costs are research costs. $20,000 should be shown as an expense in the income statement.

(2) A machine is a tangible non-current asset and is accounted for under IAS 16 regardless of its use. The $100,000 should be capitalised as a tangible non-current asset and depreciated over its useful life of 10 years.

(3) Material costs and design and manufacture salaries are part of the development process. They should be capitalised as an intangible non-current asset provided that all of the 'PIRATE' criteria are met. The costs should be amortised in 20X9 once the car is available to be sold on the market.

Answer to Lecture Example 2

Income statement extracts Expenses X1

$ X2 $

X3 $

X4 $

X5 $

Research expenditure 35,000 – – – 38,000 Amortisation of development expenditure – – 40,000 40,000 40,000

Statement of financial position extracts Non-current assets X1

$ X2 $

X3 $

X4 $

X5 $

Development expenditure 55,000 120,000 120,000 120,000 120,000 Amortisation – – (40,000) (80,000) (120,000) Net book value 55,000 120,000 80,000 40,000 –

Page 487: f3 Course Notes

25: ANSWERS TO LECTURE EXAMPLES

487

Chapter 11 Answer to Lecture Example 1

(a) $ Electricity expense Cash paid: 10.3.X7 96 12.6.X7 120 14.9.X7 104 10.12.X7 145 465 December expense missing ( 1

3 × $168) 56

521 $ Rent expense Cash paid: 1.2.X7 375 6.4.X7 1,584 1,959 Less: expense relating to Jan – March × ( 3

12 × $1,584) (396) 1,563

(b) & (c)

Electricity accrual is $56

$ $ Dr Electricity expense (I/S) 56 Cr Accruals (SOFP) 56 Being: electricity expense accrued at 31 December 20X7.

Rent prepayment is $396

$ $ Dr Prepayments (SOFP) 396 Cr Rent expense (I/S) 396 Being: rent expense prepaid at 31 December 20X7.

Answer to Lecture Example 2 Electricity expense (I/S) $ 10.3.X7 Cash 96 12.6.X7 Cash 120 14.9.X7 Cash 104 10.12.X7 Cash 145 31.12.X7 Accruals 56 521

$ 31.12.X7 Transfer to income statement 521 521

Page 488: f3 Course Notes

25: ANSWERS TO LECTURE EXAMPLES

488

Rent expense (I/S) $ 1.2.X7 Cash 375 6.4.X7 Cash 1,584 1,959

$ 31.12.X7 Transfer to income statement 1,563 31.12.X7 Prepayments 396 1,959

Accruals (SOFP) $ 31.12.X7 Bal c/d 56 56

$ 31.12.X7 Electricity 56 56 1.1.X8 Bal b/d 56

Prepayments (SOFP) $ 31.12.X7 Rent 396 396 1.1.X8 Bal b/d 396

$ 31.12.X7 Bal c/d 396 396

Answer to Lecture Example 3 Working

Electricity expense (I/S)

$ 12.3.X8 Cash 168 9.6.X8 Cash 134 12.9.X8 Cash 118 12.12.X8 Cash 158 31.12.X8 Accrual ( 1

3 × $189) 63 641

$ 1.1.X8 Accrual reversed 56 31.12.X8 To Income statement 585 641

Accruals (SOFP)

$ 1.1.X8 Accrual reversed 56 31.12.X8 Bal c/d 63 119

$ 1.1.X8 Bal b/d 56 31.12.X8 Electricity accrual (W) 63 119 1.1.X9 Bal b/d 63

Answer to Lecture Example 4 B

$ Insurance expense July X6 – August X6 ( 12

2 × $24,000) 4,000 Sept X6 – June X7 ( 12

10 × $30,000) 25,000 29,000 Prepayment 1 June X7 paid ( 1

4 × $30,000) 7,500 Less: June X7 ( 1

3 × $7,500) (2,500) 5,000

Page 489: f3 Course Notes

25: ANSWERS TO LECTURE EXAMPLES

489

Chapter 12

Answer to Lecture Example 1 (a) The balance c/d on the trade receivables account at the end of the year is $50,000. (b) The bad debt expense shown in the I/S is $15,000

Workings Trade receivables (SOFP)

$ 31.12.X7 Bal b/d 65,000 65,000

$ 31.12.X7 Bad debt expense (Ali $7,000) 15,000 (Tyson $8,000) 31.12.X7 Bal c/d 50,000 65,000

Bad debt expense (I/S)

$

31.12.X7 Trade receivables 15,000

$

31.12.X7 To I/S 15,000

Answer to Lecture Example 2 Allowance for receivables:

(a) The allowance for receivables shown on the statement of financial position is $3,500 (b) The doubtful debts expense shown in the I/S is $3,500

Working

Allowance for receivables (SOFP)

$

Bal c/d 3,500

$

Doubtful debts expense 3,500

Doubtful debts expense (I/S)

$

Allowance for receivables 3,500

$

I/S 3,500

Income statement extract $ Expenses Bad debts (see Lecture Example 1) (15,000) Doubtful debts expense (3,500) Statement of financial position extract $ Current assets Trade receivables 50,000 Less: allowance for receivables (3,500) 46,500

Page 490: f3 Course Notes

25: ANSWERS TO LECTURE EXAMPLES

490

Answer to Lecture Example 3 (a) The allowance for receivables shown in the statement of financial position is $1,334 (b) The bad and doubtful debts expense shown in the income statement is $1,674.

Trade receivables (SOFP)

$ Bal b/d 47,440 47,440 Bal b/d 47,100

$ Bad & doubtful debts expense 340 Bal c/d 47,100 47,100

Allowance for receivables (SOFP)

$ Bal c/d Specific 400 General (W) 934 1,334 1,334

$ Bad and doubtful debts expense 1,334 1,334 Bal b/d 1,334

Bad and doubtful debts expense (I/S)

$ Trade receivables 340 Allowance for receivables 1,334 1,674

$

I/S 1,674 1,674

Working

(W) General allowance: $ Trade receivables (net of bad debts written off) 47,100 Less: specific allowance (400) 46,700 × 2% = $934

Answer to Lecture Example 4 Bad debts recovered:

Trade receivables (SOFP)

$

1.1.X8 Bal b/d 50,000

$

Bad debt expense (I/S)

$

I/S 7,000

$

Cash 7,000

Page 491: f3 Course Notes

25: ANSWERS TO LECTURE EXAMPLES

491

Cash (SOFP)

$

Bad debt expense 7,000

Answer to Lecture Example 5 Specific allowance recovered: Trade receivables (SOFP)

$ Bal b/d 50,000 50,000

$ (a) Cash 3,500 Bal c/d 46,500

50,000

Allowance for receivables (SOFP)

$

(b) Doubtful debts expense 3,500

$

Bal b/d 3,500

Bad and doubtful debts expense (I/S)

$

I/S 3,500

$

(b) Allowance for doubtful debts 3,500

Answer to Lecture Example 6 $ $ Dr Allowance for receivables 3,500 Cr Trade receivables 3,500

Answer to Lecture Example 7 Changes in general allowance:

The doubtful debts expense in 20X8 is $500 [(30,000 x 5%) – (20,000 x 5%)]

Long method

Allowance for receivables (SOFP)

$ (a) Doubtful debts expense 1,000 (20,000 × 5%)

Bal c/d 1,500

2,500

$ 1.1.X8 Bal b/d ($20,000 × 5%) 1,000 (ii) 31.12.X8 Doubtful debts expense ($30,000 × 5%) 1,500 2,500

Page 492: f3 Course Notes

25: ANSWERS TO LECTURE EXAMPLES

492

Doubtful debts expense (I/S)

$ (ii) Allowance for receivables 1,500

1,500

$ (a) Allowance for receivables 1,000 I/S 500 1,500

Short method Allowance for receivables (SOFP)

$ 31.12.X8 Bal c/d ($30,000 × 5%) 1,500

1,500

$ 31.12.X7 Bal b/d ($20,000 × 5%) 1,000 Doubtful debts expense 500 (increase in allowance) 1,500

Doubtful debts expense (I/S)

$

Allowance for receivables 500

$

I/S 500

Answer to Lecture Example 8 A $13,000

Allowance for receivables

Income statement

$ $ (1) Write off recovered (2,000) (2) Write off in 20X8 18,000 (3) Change in allowance: At 30.9.X7 24,000 At 30.9.X8 21,000 Decrease required 3,000 (3,000) 13,000

Chapter 13

Answer to Lecture Example 1 (a) A provision should be made using expected values:

($1m × 20%) + ($6m × 5%) = $0.5m

Dr Warranty cost expense (I/S) $0.5m Cr Provisions (SOFP) $0.5m

(b) In 20X8 the provision needs to increase by $0.25m ($0.75m – $0.5m). Entry is:

Dr Warranty cost expense (I/S) $0.25m Cr Provisions (SOFP) $0.25m

Page 493: f3 Course Notes

25: ANSWERS TO LECTURE EXAMPLES

493

(c) In 20X9 the provision needs to decrease by $0.45m ($0.75m – $0.3m). Entry is

Dr Provisions (SOFP) $0.45m Cr Warranty cost expense (I/S) $0.45m

Chapter 14

Answer to Lecture Example 1 (1) Books of prime entry

Sales day book

Date Customer Amount 10 Jan X6 Customer A 150 10 Jan X6 Customer B 200

350

Purchase day book

Date Supplier Amount 15 Jan X6 Supplier Y 100 15 Jan X6 Supplier Z 1,300

1,400

Cash receipts book

Date Narrative Total Sales Receivables 21 Jan X6 Customer B 200 200

200 200

Cash payments book

Date Narrative Total Purchases Payables 21 Jan X6 Supplier Y 100 100

100 100

Memorandum ledgers

Receivables ledger

Customer A $ $ 10.1.X6 Sales 150 Bal c/d 150 150 150 Bal b/d 150

Customer B $ $ 10.1.X6 Sales 200 21.1.X6 Payment received 200 200 200

Page 494: f3 Course Notes

25: ANSWERS TO LECTURE EXAMPLES

494

Payables ledger Supplier Y

$ $ 21.1.X6 Payment made 100 15.1.X6 Purchases 100 100 100

Supplier Z $ $ Bal c/d 1,300 15.1.X6 Purchases 1,300 1,300 1,300

(2)&(3) Nominal ledger

RLCA (SOFP) $ $ 31.1.X6 Sales 350 31.1.X6 Bank 200 Bal c/d 150 350 350 Bal b/d 150

PLCA (SOFP) $ $ 31.1.X6 Bank 100 31.1.X6 Purchases 1,400 Bal c/d 1,300 1,400 1,400 Bal b/d 1,300

Bank (SOFP) $ $ 31.1.X6 RLCA 200 31.1.X6 PLCA 100 Bal c/d 100 200 200 Bal b/d 100

Sales (I/S) Purchases (I/S) $ $ $ $ 31.1.X6

RLCA 350 31.1.X6

PLCA 1,400

I/S 350 I/S 1,400 350 350 1,400 1,400

(4) Reconciliation

Balance per list of balances $ Receivables ledger Customer A 150 Customer B – 150 Balance per RLCA 150 Balance per list of balances $ Payables ledger Supplier Y – Supplier Z 1,300 1,300 Balance per PLCA 1,300

Page 495: f3 Course Notes

25: ANSWERS TO LECTURE EXAMPLES

495

Answer to Lecture Example 2 (a) Sales (I/S) RLCA (SOFP)

$

$ 1.1.X7 RLCA 10,000

$ 1.1.X7 Sales 10,000

$

(b) Bank (SOFP) RLCA (SOFP)

$ 4.1.X7 RLCA 9,000

$

$ 1.1.X7 Sales 10,000 10,000

$ 4.1.X7 Bank 9,000 Discounts 1,000 allowed 10,000

Discounts allowed (I/S)

$ 4.1.X7 RLCA 1,000

$

(c) Bank (SOFP) RLCA (SOFP)

$ 4.1.X7 RLCA 10,000

$

$ 1.1.X7 Sales 10,000 10,000

$ 4.1.X7 Bank 10,000 10,000

Answer to Lecture Example 3 (a) Purchases (I/S) PLCA (SOFP)

$ PLCA 5,000

$

$

$ Purchases 5,000

(b) Bank (SOFP) PLCA (SOFP)

$

$ Bank 4,750

$ Bank 4,750 Discounts received 250 5,000

$ Purchases 5,000 5,000

Page 496: f3 Course Notes

25: ANSWERS TO LECTURE EXAMPLES

496

Discounts received (I/S)

$

$ PLCA 250

(c) Bank (SOFP) PLCA (SOFP)

$

$ PLCA 5,000

$ Bank 5,000

$ Purchases 5,000

Answer to Lecture Example 4 B

$ List price 50,000 Less: trade discount (12%) (6,000) Record purchase at this value 44,000 Less: settlement discount (4%) (1,760) Calculate sales tax on this value 42,240 Sales tax at 15% 6,336

Answer to Lecture Example 5 (a) RLCA

$ $ Balance b/d 614,000 Bank 311,000 Sales 302,600 Discounts allowed 3,400 Contras (PLCA) 8,650 Bad debts 32,000 Bal c/d 561,550 916,600 916,600

(b) Reconciliation

RLCA $ $ Bal b/d (part (a)) 561,550 (i) Sales (SDB undercast) 3,600 Bal c/d 565,150 565,150 565,150 $ Balance per list of balances 563,900 (ii) Credit balance included as a debit (2 × $450) (900) Customer balance omitted 2,150 1,250 565,150

$50,336

Page 497: f3 Course Notes

25: ANSWERS TO LECTURE EXAMPLES

497

Chapter 15

Answer to Lecture Example 1 Adjustment of cash book balance

Cash account $ $ Balance b/d 204 Standing order (3i) 35 Bank interest (3ii) 18 Bank charges (3iii) 14 Balance c/d 173 222 222

Bank reconciliation statement

$ Balance per bank statement at 31 March 20X8 2,618 Unrecorded lodgements 723 Outstanding cheques (3,168) Balance per cash book at 31 March 20X8 173

Answer to Lecture Example 2 B (1) is a bank error, (4) is an outstanding cheque (2), (3) and (5) have all been processed correctly

by the bank but need recording in the cash account.

Chapter 16

Answer to Lecture Example 1 (a) Journal entries

Dr Cr $ $ (1) Rent and rates 350 Trade receivables 350 (2) Discounts allowed 500 Trade receivables 500 (3) Trade receivables 2,620 Cash at bank 2,620 (4) Suspense account 1,900 Cash at bank 1,900 (5) Stationery and postage 1,460 Suspense account 1,460 (6) Capital 18,000 Suspense account 18,000

(b) Suspense account $ $ Brought forward (102,800 – 85,240) 17,560 Stationery and postage (5) 1,460 Cash at bank (4) 1,900 Capital (6) 18,000 19,460 19,460

Page 498: f3 Course Notes

25: ANSWERS TO LECTURE EXAMPLES

498

Answer to Lecture Example 2 Adjustment of profits statement for the year ended 30 April 20X7

Increases Decreases $ $ $ Draft profit Adjustments 12,300 Rent (1) 350 Discounts allowed (2) 500 Stationery (5) 1,460 Total adjustments 2,310 (2,310) Revised profit 9,990

Answer to Lecture Example 3 B

Increases Decreases $ $ $ Draft profit 112,400 Adjustments: (1) sales returns (2 × $2,700) 5,400 (2) depreciation (W) 1,250 – 6,650 (6,650) Adjusted profit 105,750 (W ) Depreciation charge was

33 13 % × ($15,000 × 2/4) = $2,500

Depreciation charge should have been $15,000 ÷ 4 years = $3,750 Incremental depreciation to be charged $1,250

Chapter 17

Answer to Lecture Example 1 (a) (1)

Dr Cr $ $ Dr Inventories (SOFP) 647 Cr Closing inventories (I/S) 647 Being: adjustment to record year end closing inventories.

(2) $ $ Dr Drawings (12 × $10) 120 Cr Wages 120 Being: correction of cash drawings posted as wages.

Page 499: f3 Course Notes

25: ANSWERS TO LECTURE EXAMPLES

499

(3) $ $ Dr Depreciation expense (I/S) 601 Cr Accumulated depreciation: Motor vehicles ($1,740 × 25%) 435 Furniture and fittings ($829 × 20%) 166

Being: adjustment to record depreciation for the year

(4) $ $ Dr Bad debt expense 37 Cr Trade receivables 37 Being: write off of irrecoverable customer balance.

(5) $ $ Dr Bank (2 × $180) 360 Cr Suspense account 360 Being: adjustment to correct cash receipt from trade receivables.

(6) $ $ Dr Drawings 63 Cr Purchases 63 Being: adjustment for goods drawn from business (removed at cost value)

(7) $ $ Dr Rent expense (600 – 500) 100 Cr Accruals 100 Being: accrual of rent expense. $ $ Dr Prepayments ($180 × 6/12) 90 Cr Electricity expense 90 Being: prepayment of electricity expense.

(8) $ $ Dr Discounts allowed (I/S) 73 Cr Suspense account 73 Being: adjustment for discounts allowed omitted.

(b) Suspense account

$ $ Bal b/d 433 (5) Bank 360 (8) Discounts allowed 73 433 433

Page 500: f3 Course Notes

25: ANSWERS TO LECTURE EXAMPLES

500

(c) Mugg Income statement for the year ended 31 December 20X7

$ $ Sales 15,542 Less: cost of sales Opening inventories 510 Purchases (9,876 – 63) 9,813 10,323 Less: closing inventories 647 9,676 Gross profit 5,866 Discounts received 129 5,995 Less expenses: Rent (500 + 100) 600 Electricity (240 – ( 6

12 × 180)) 150 Insurance 120 Wages (1,634 – 120) 1,514 Repairs 635 Depreciation 601 Travel and entertaining 192 Bad debts 37 Discounts allowed 73 3,922 Profit for the period 2,073 Mugg Statement of financial position as at 31 December 20X7

Cost

Accumulated depreciation

NBV

$ $ $ Non-current assets Motor vehicles 1,740 870 870 Furniture and fixtures 830 332 498 2,569 1,202 1,368 Current assets Inventories 647 Trade receivables (672 – 37) 635 Prepayments 90 Cash and bank balances (5 + 762 + 360) 1,127 2,499 3,867 Capital $ Capital as at 1 January 20X7 2,377 Profit for the period 2,073 Less: drawings (1,200 + 63 + 120) (1,383) 3,067 Current liabilities Trade payables 700 Accruals 100 800 3,867

Page 501: f3 Course Notes

25: ANSWERS TO LECTURE EXAMPLES

501

Chapter 18 Answer to Lecture Example 1

% $ Sales 100 476,000 x 60% COS 60 285,600 GP 40 190,400

Answer to Lecture Example 2

% $ Sales 130 221,000 x 100/130 COS 100 170,000 GP 30 51,000

Purchases: $ Cost of sales Opening inventory 43,000 + Purchases 174,500 – Closing inventory 47,500 170,000

Answer to Lecture Example 3

B Cost structure: 25% mark up.

$ Sales = 125% = 985,000 ∴ COS = 100% = 788,000 Gross profit 25% 197,000 Cost of sales

$ Opening inventories 620,000 Purchases 700,000 1,320,000 Less: cost of sales (788,000) Closing inventories should be 532,000 Closing inventories is (180,000) ∴ inventory lost in fire 352,000

Page 502: f3 Course Notes

25: ANSWERS TO LECTURE EXAMPLES

502

Answer to Lecture Example 4 Trade payables

$ Till 430 Bank 167,224 Balance c/d 43,825 211,479

$ Bal b/d 38,450 Purchases* 173,029 211,479

* Dr Purchases (I/S) Cr Trade payables

$173,029 $173,029

Answer to Lecture Example 5 Cash

$ Bal b/d 50 Receipts from Trade receivables (1) 39,204 39,254

$ General expenses 4,500 Drawings 6,250 Bankings 28,454 Bal c/d 50 39,254

Trade receivables

$ Bal b/d 1,447 Sales* (2) 39,685 41,132

$ Cash (deduced from cash a/c) 39,204

Bal c/d 1,928 41,132

Answer to Lecture Example 6 $4,050

Cost structure:

$ Sales = 100% = 23,750 ∴ COS = 80% = 19,000 Gross profit 20% 4,750

Cash

$ $ Balance b/d 1,000 Wages 5,200 Stationery 500 Sales 23,750 Electricity 1,200 Bankings 12,800 ∴ drawings 4,050 Bal c/d 1,000 24,750 24,750

Page 503: f3 Course Notes

25: ANSWERS TO LECTURE EXAMPLES

503

Chapter 19

Answer to Lecture Example 1 (a)

(i) Capital accounts

Tick Cast Balance $ $ $ Bal c/d 50,000 30,000 20,000 50,000 30,000 20,000

Tick Cast Balance $ $ $ Bank 50,000 30,000 20,000 50,000 30,000 20,000 Bal b/d 50,000 30,000 20,000

(ii)

Appropriation account for the year ended 31 December 20X4 $ $ Salary – Balance 15,000 Interest on capital (12%) Tick 6,000 Cast 3,600 Balance 2,400 12,000 Profit share Tick (5/10) 11,500 Cast (3/10) 6,900 Balance (2/10) 4,600 23,000 50,000

$ Profit b/d from the income statement 50,000 50,000

(iii) Current accounts

Tick Cast Balance $ $ $ Drawings 6,000 4,000 8,800 Bal c/d 11,500 6,500 13,200 17,500 10,500 22,000

Tick Cast Balance $ $ $ Salary 15,000 Interest on capital 6,000 3,600 2,400 Profit share 11,500 6,900 4,600 17,500 10,500 22,000

Page 504: f3 Course Notes

25: ANSWERS TO LECTURE EXAMPLES

504

(iv) TICK, CAST AND BALANCE

Statement of financial position as at 31 December 20X4 (extract) $ $ Capital accounts Tick 50,000 Cast 30,000 Balance 20,000 100,000 Current accounts Tick 11,500 Cast 6,500 Balance 13,200 31,200 131,200 (b) Alternative answer to parts (ii) and (iii) (including interest on drawings)

(ii) Appropriation account for the year ended 31 December 20X4

$ $ Salary – Balance 15,000 Interest on capital (12%) Tick 6,000 Cast 3,600 Balance 2,400 12,000 Profit share Tick (5/10) 11,700 Cast (3/10) 7,020 Balance (2/10) 4,680 23,400 50,400

$ Profit b/d 50,000 Interest on drawings Tick (6,000 x 10% x 6/12) 300 Cast (4,000 x 10% x 3/12) 100 50,400

(iii) Current accounts

Tick Cast Balance $ $ $ Drawings 6,000 4,000 8,800 Interest on drawings 300 100 – Balance c/d 11,400 6,520 13,280 17,700 10,620 22,080

Tick Cast Balance $ $ $ Salary 15,000 Interest on capital 6,000 3,600 2,400 Profit share 11,700 7,020 4,680 17,700 10,620 22,080 Bal b/d 11,400 6,520 13,280

Page 505: f3 Course Notes

25: ANSWERS TO LECTURE EXAMPLES

505

Answer to Lecture Example 2 X Y Z Total $ $ $ $ Salaries 15,000 – 8,000 23,000 Interest on capital 400 400 400 1,200 Profit share (6:3:1) 25,320 12,660 4,220 42,200 (β) Total profit for the year (W) 40,720 13,060 12,620 66,400 (W)

$ Profit before loan interest 67,000 Loan interest ($10,000 × 12% × 6/12) (600) 66,400

Answer to Lecture Example 3 B M S A $ $ $ Salary 20,000 10,000 PSR (1st half) 90,000 30,000 30,000 PSR (2nd half) 110,000 66,000 44,000 200,000 116,000 84,000

$ Profit for year 400,000 Add bank expense relating to first half of year 40,000 440,000 1st Half $ $ Profit ($440,000 x 6/12) 220,000 Expense relevant to first half of year (40,000) 180,000 Salary S (40,000 x 6/12) (20,000) A (20,000 x 6/12) (10,000) 150,000 PSR M 60% (90,000) S 20% (30,000) A 20% (30,000) (150,000) – 2nd Half $ $ Profit ($440,000 x 6/12) 220,000 Salary – 220,000 PSR M 50% (110,000) S 30% (66,000) A 20% (44,000) (220,000) –

Page 506: f3 Course Notes

25: ANSWERS TO LECTURE EXAMPLES

506

Answer to Lecture Example 4 Capital account

A B C A B C $'000 $'000 $'000 $'000 $'000 $'000 Bal b/d 80 97 – Goodwill (1:1:1)

20 20 20 Goodwill (5:7)

25 35 –

Bal c/d 85 112 80 Bank – – 100 105 132 100 105 132 100

Goodwill

$'000 $'000 Capital account (210 – 150) 60 Capital account 60

C has effectively paid in capital of $100,000 but $20,000 of this was used to pay for C's share of the goodwill the partnership had built up. C's closing investment in the partnership is therefore $80,000.

Chapter 20

Answer to Lecture Example 1 Rab Co $ $ Dr Cash (200,000 × 80c) 160,000 Cr Share capital (200,000 × 50c) 100,000 Cr Share premium account (200,000 × 30c) 60,000 Statement of financial position (extract) as at 1 June 20X0 Equity $ Share capital – 50c ordinary shares (50,000 + 100,000) 150,000 Share premium account 60,000 210,000

Page 507: f3 Course Notes

25: ANSWERS TO LECTURE EXAMPLES

507

Answer to Lecture Example 2 Bonus Issue

New share capital: 500,37$c504000,300 =×

Double entry: $ $ Dr Share premium account 37,500 Cr Share capital 37,500 Statement of financial position $ Share capital – 50c ordinary shares (150,000 + 37,500) 187,500 Share premium account (60,000 – 37,500) 22,500 Retained earnings 200,000 410,000

Answer to Lecture Example 3 Rights Issue $ $

New share capital: c505000,375 × 37,500

Share premium: 1$5000,375 × 75,000

$ $ Dr Cash 112,500 Cr Share capital 37,500 Cr Share premium account 75,000 Rab Co Statement of financial position (extract) $ Share capital – 50c ordinary shares 225,000 Share premium account 97,500 Retained earnings 230,000 552,500

Answer to Lecture Example 4 ABC Co Reconciliation of movement in retained earnings for year ended 31 December 20X6 $ $ Retained earnings at beginning of year 125,000 Profit for the period 50,000 Dividends – preference 6,000 – ordinary 10,000 (16,000) Retained earnings at end of year 159,000

Page 508: f3 Course Notes

25: ANSWERS TO LECTURE EXAMPLES

508

Answer to Lecture Example 5 (1)

Income tax expense (I/S)

$ 31.12.X5 Current tax payable 62,000 30.9.X6 Current tax payable 3,000 31.12.X6 Current tax payable 43,000

$ 31.12.X5 Income statement 62,000

Current tax payable (SOFP)

$ 31.12.X5 Balance c/d 62,000 62,000 30.9.X6 Bank 65,000 31.12.X6 Bal c/d 43,000 108,000

$ 31.12.X5 Income tax expense 62,000 62,000 1.1.X6 Balance b/d 62,000 30.9.X6 Income tax expense 3,000 31.12.X6 Income tax expense 43,000 108,000 1.1.X7 Balance b/d 43,000

(2) Tax note for the year ended 31 December 20X6

$ Tax charge for the year 43,000 Under provision in respect of prior periods 3,000

46,000

Chapter 21

Answer to Lecture Example 1 (a) One single statement

Statement of comprehensive income for the year ended 30 September 20X6 $’000 Revenue 12,740

Cost of sales (7,040) Gross profit 5,700 Distribution costs (2,060) Administrative expenses (2,375) Finance costs (72) Profit before tax 1,193 Income tax expense (270) Profit for the year 923 Other comprehensive income: Gains on property revaluation 600 Total comprehensive income for the year 1,523

Page 509: f3 Course Notes

25: ANSWERS TO LECTURE EXAMPLES

509

(b) Two separate statements

Income statement for the year ended 30 September 20X6 $’000 Revenue 12,740 Cost of sales (7,040) Gross profit 5,700 Distribution costs (2,060)

Administrative expenses (2,375) Finance costs (72) Profit before tax 1,193 Income tax expense (270)

Profit for the year 923 Statement of comprehensive income for the year ended 30 September 20X6 $’000 Profit for the year 923 Other comprehensive income: Gains on property revaluation 600 Total comprehensive income for the year 1,523

Answer to Lecture Example 2 Share

capital Share

premium account

Reval- uation

reserve

Retained earnings

Total equity

$’000 $’000 $’000 $’000 $’000

Balance at 30 September 20X5 1,500 200 800 1,250 3,750

Issue of share capital 250 385 635

Dividends (300) (300)

Total comprehensive income ___ ___ 600 923 1,523

Balance at 30 September 20X6 1,750 585 1,400 1,873 5,608

Working

Rights issue:

Issue is on a 1 for 6 basis, therefore issue 3,000,000 ÷ 6 = 500,000 shares at $1.27 each.

Record as: Dr Bank (500,000 × $1.27) 635,000 Cr Share capital (500,000 × 50c) 250,000 Cr Share premium (500,000 × 77c) 385,000

Page 510: f3 Course Notes

25: ANSWERS TO LECTURE EXAMPLES

510

Chapter 22

Answer to Lecture Example 1 B 1 and 3 are non-adjusting events as the condition did not exist at the end of the reporting period.

Chapter 23

Answer to Lecture Example 1 Income taxes paid

Income tax payable

$'000 Income tax paid 116 Bal c/d 156 272

$'000 Bal b/d 168 I/S 104 272

Answer to Lecture Example 2 Property, plant and equipment

Plant and equipment – cost

$'000 Bal b/d 200 Addition 100 300

$'000 Disposal 20 Bal c/d 280 300

Accumulated depreciation

$'000 Disposal 9 Bal c/d 111 120

$'000 Bal b/d 80 ... Charge 40 120

Profit/loss on disposal: $ Net book value of asset sold 11,000 Sales proceeds (8,000) Loss on sale (3,000) The entries in the statement of cash flows for 20X9 would be: $ (i) Cash flows from operating activities (extract) Adjustments for Depreciation 40,000 Loss on sale of plant 3,000 43,000 (ii) Cash flows from investing activities (extract) Purchase of property, plant and equipment (100,000) Proceeds from sale of plant 8,000 (92,000)

Page 511: f3 Course Notes

25: ANSWERS TO LECTURE EXAMPLES

511

Answer to Lecture Example 3 Dividends paid

Dividends payable

$'000 Dividends paid 50 Bal c/d 45 95

$'000 Bal b/d 35 Retained earnings 60 95

Answer to Lecture Example 4 Emma Co Statement of cash flows for the year ended 31 December 20X8

$’000 $’000 Cash flows from operating activities Profit before taxation 87 Adjustments for: Depreciation 42 Interest expense 8 137 Increase in trade receivables (168 – 147) (21) Increase in inventories (214 – 210) (4) Increase in trade payables (136 – 121) 15 Cash generated from operations 127 Interest paid (8) Income taxes paid (W2) (20) Net cash from operating activities 99 Cash flows from investing activities Purchase of property, plant and equipment (W1) (146) Net cash used in investing activities (146) Cash flows from financing activities Proceeds from issue of shares (250 + 70 – 200 – 60) 60 Proceeds from issue of debentures 30 Dividends paid (W3) (22) Net cash from financing activities 68 Net increase in cash and cash equivalents 21 Cash and cash equivalents at beginning of year (14) Cash and cash equivalents at end of year 7 Workings

(W1) Property, plant and equipment at NBV $’000 Bal b/d 514 Revaluation during the year 10 (110 – 100) Additions 146 670

$’000 Depreciation 42 Bal c/d 628 670

Page 512: f3 Course Notes

25: ANSWERS TO LECTURE EXAMPLES

512

(W2) Income tax payable $’000 Income tax paid 20 Bal c/d 39 59

$’000 Bal b/d 28 Income tax expense (I/S) 31 59

(W3) Dividends payable

$’000 Dividends paid 22 Bal c/d 18 40

$’000 Bal b/d 16 Dividend for year 24 40

Answer to Lecture Example 5 Emma Co Statement of cash flows for the year ended 31 December 20X8 (extract)

$'000 $'000 Cash flow from operating activities Cash receipts from customers (W1) 579 Cash payments to suppliers and employees (W2) (452) Cash generated from operations 127 Interest paid* (8) Income taxes paid* (20) Net cash from operating activities 99 * These amounts are the same amounts as in Lecture Example 4.

Workings

(W1) Trade receivables $'000 $'000 Bal b/d 147 ∴ cash received 579 Revenue (I/S) 600 Bal c/d 168 747 747

(W2) Trade payables $'000 $'000 Bal b/d 121 ∴ cash paid 452 Expenses (W3) 467 Bal c/d 136 588 588

(W3) $'000 $'000 Cost of sales 319 Add: closing inventories 214 Less: opening inventories (210) Purchases 323 Other expenses 186 Less: depreciation (42) 144 467

Page 513: f3 Course Notes

25: ANSWERS TO LECTURE EXAMPLES

513

Chapter 24

Answer to Lecture Example 1 Information that may be included in a database file for a non-current asset register:

(1) Code/item number to identify asset (2) Details of category of asset (motor vehicles, machine etc) (3) Serial number of the asset (4) Details of physical location of the asset (5) Person responsible for the asset (6) Cost of the asset (7) Date of purchase (8) Depreciation policy for the asset (9) Accumulated depreciation charged to date (10) Net book value of the asset (11) Insurance details

Page 514: f3 Course Notes

25: ANSWERS TO LECTURE EXAMPLES

514

END OF ANSWERS TO LECTURE EXAMPLES